Sunteți pe pagina 1din 94

Anul XV, Nr.

Iulie Decembrie 2013

RECREAII MATEMATICE
REVIST DE MATEMATIC PENTRU ELEVI I PROFESORI

e i = 1

Asociaia Recreaii Matematice IAI - 2013

Semnificaia formulei de pe copert.

ntr-o form concis, formula e = 1 leag cele patru ramuri fundamentale ale matematicii: ARITMETICA - reprezentat de 1; GEOMETRIA reprezentat de ; ALGEBRA reprezentat de i ; ANALIZA MATEMATIC reprezentat de e.

Membrii fondatori :
Temistocle BRSAN Ctlin CALISTRU Alexandru CRUU Constantin COCEA Adrian CORDUNEANU Gheorghe IUREA

Membri onorifici :
Acad. Constantin CORDUNEANU Prof.univ. Vasile OPROIU Acad. Radu MIRON Cercet.pr. Dan TIBA

Redactor ef :

Temistocle BRSAN

Redactori principali : Gabriel POPA coordonator al rubricilor de probleme


Gheorghe IUREA, Petru ASAFTEI, Maria RACU

Comitetul de redacie :
Alexandru CRUU Constantin CHIRIL Eugenia COHAL Adrian CORDUNEANU Mihai CRCIUN (Pacani) Paraschiva GALIA Paul GEORGESCU Gheorghe ILIE Gabriel MRANU Dan POPESCU (Suceava) Neculai ROMAN (Mirceti) Ioan ERDEAN (Ortie) Marian TETIVA (Brlad) Lucian TUESCU (Craiova) Adrian ZANOSCHI Titu ZVONARU (Comneti)

Materialele vor fi trimise la una dintre adresele: t-birsan@yahoo.com , profgpopa@yahoo.co.uk COPYRIGHT 2008, ASOCIAIA RECREAII MATEMATICE Toate drepturile aparin Asociaiei Recreaii Matematice. Reproducerea integral sau parial a textului sau a ilustraiilor din aceast revist este posibil numai cu acordul prealabil scris al acesteia. TIPRIT LA BLUE SIM&Co IAI Bd. Carol I, nr. 3-5 Tel. 0332 111021, 0721 571705; e-mail: simonaslf@yahoo.com ISSN 1582 1765

Anul XV, Nr. 2

Iulie Decembrie 2013

RECREAII MATEMATICE
REVIST DE MATEMATIC PENTRU ELEVI I PROFESORI

e i = 1
Revist cu apari ie semestrial

EDITURA RECREAII MATEMATICE

IA I - 2013

Profesorul Constantin Corduneanu la aniversarea a 85 de ani

Constantin Corduneanu este binecunoscut, apreciat si se bucur a de un deosebit prestigiu n lumea academic a matematic a grat ie prodigioasei sale activit a ti stiint ice, didactice si prezent ei active n manifest arile comunit a tii matematice internat ionale: conferint e, simpozioane si congrese. S-a n ascut n Ia si la 26 iulie 1928, a copil arit si a parcurs clasele primare n satul Pot angeni, com. Movileni, jud. Ia si, sat n care p arint ii s ai erau nv a tatori. Studiile secundare le-a nceput n 1940 la Liceul Militar din Ia si si le-a terminat dup a Al Doilea R azboi Mondial, n 1947, la Predeal, unde liceul se mutase ntre timp. Ultimele dou a clase lea promovat ntr-un singur an, regulamentul scolar permit and acest lucru. A fost colaborator al Gazetei Matematice si a participat la concursurile organizate de aceasta; n ultimul an de liceu a fost premiat la Concursul Gazetei Matematice, clas andu-se pe locul nt ai. Despre aceast a scoal a si dasc alii s ai, are cuvinte de ad anc a recuno stint a: Am avut sansa de a studia n acea scoal a de elit a, unde profesorul de istorie era doctor la Oxford, cel de chimie era de asemenea doctor n chimie si inginer chimist, cel de geograe era simultan si asistent universitar, cel de francez a era doctor n lologie, cu stagiu de specializare n Frant a, devenind apoi seful catedrei de limb a francez a la Universitatea din Ia si. Profesorul de matematic a fusese si cadru universitar (conferent iar universitar). Profesorii s ai l-au sf atuit ca, n noile condit ii politice din tar a, s a renunt e la o carier a militar a si s a continue cu studii de matematic a. S-a nscris la Facultatea de S tiint e din Ia si, sect ia matematic a, la care a promovat primii doi ani de studii ntr-un singur an. A absolvit facultatea n 1950, iar n ultimul an de studii a fost ret inut ca preparator. Fiind un student eminent, scurt and perioada de studii prin comasarea unor ani, sprijinit de profesorii s ai n momente critice ale viet ii, a reu sit s a scape de prigoana contra celor cu origine nu tocmai ,,s an atoas a. Cu aceea si ad anc a recuno stint a se refer a si la perioada studiilor universitare: In anii mei de student ie am avut sansa extraordinar a de a nimeri n scoala de matematic a a lui Alexandru Myller. A r amas pentru totdeauna ata sat de Ia sul tineret ii sale, de Seminarul Matematic n care s-a format ca specialist, de atmosfera de nalt a tinut a stiint ic a a acestuia si de profesorii s ai. In anul 2010, la conferint a dedicat a jubileului centenarului Seminarului Matematic ,,A. Myller va expune n plen un istoric al acestei institut ii. 89

Datorit a talentului, pasiunii si d aruirii sale, parcurge n put ini ani treptele ierarhiei didactice: n 1955 este lector universitar titular, n 1962 conferent iar prin concurs, iar n anul 1968 este promovat ca profesor titular. A sust inut n 1956 la Universitatea din Ia si teza de doctorat Probleme globale pentru ecuat iile diferent iale de ordinul I si II sub conducerea prof.dr. Ilie Popa si n fat a unei comisii formate din academicienii M. Nicolescu, Gr. Moisil si N. Teodorescu. In aceast a perioad a a funct ionat si la Institutul Politehnic din Ia si (1950-1954) si a avut norm a de cercetare la Institutul de Matematic a al Academiei-liala Ia si. In perioada 1964-1967 a contribuit la organizarea activitat ii n Institutul Pedagogic din Suceava (acum Universitatea ,,S tefan cel Mare), ind rector n ultimul an. In anii academici 1967-68 si 1973-74 a fost profesor vizitator la Universitatea de Stat din Rhode Island (SUA). Intre anii 1968-1972 a fost decan al Facult a tii de Matematic a, iar ntre 1972-1973 si 1974-1975 a avut funct ia de prorector al Universit a tii din Ia si. Printr-un decret din prim avara anului 1975, Institutele de Matematic a cu sediile n Bucure sti, Ia si si Cluj Napoca treceau de sub controlul Academiei Rom ane n subordinea universit a tilor, care aveau obligat ia s a distribuie la alte unit a ti socialiste pe tot i cercet atorii ncadrat i n ele. In acel moment, profesorul C. Corduneanu era prorector cu problemele cercet arii stiint ice si studiilor doctorale si, n consecint a, i-a revenit ca obligat ie de serviciu aplicarea decretului. Mai t arziu, ntr-un interviu, profesorul si aminte ste: Majoritatea celor transferat i, prin aceast a ingrat a sarcin a de serviciu, erau fo stii mei student i si erau dintre cei mai buni! Desint area Institutului de Matematic a, c at si climatul politic tensionat din tara noastr a, l-au decis s a emigreze. In 1977 este invitat s a expun a o serie de conferint e la Centrul Internat ional de Fizic a Teoretic a (UNESCO), Trieste (Italia). Folosind acest prilej, emigreaz a la nceputul anului 1978 n SUA. Dup a scurte sederi ca profesor vizitator la Universitatea de Stat din Rhode Island (ian.-iun. 1978) si la Universitatea de Stat Tennesse (Knoxville, 1978-1979), ocup a prin concurs o pozit ie de profesor la Universitatea de Stat din Texas, ora sul Arlington, pozit ie din care se va pensiona n 1996, cu titlul de Profesor Emerit. Activitatea didactic a si de ndrumare a profesorului C. Corduneanu este bogat a si de lung a durat a: 47 de ani, dintre care n Rom ania 26 de ani si jum atate. A contribuit la preg atirea a circa 4000 de student i n Rom ania si 3000 n SUA, iar num arul tinerilor care si-au trecut doctoratul sub conducerea sa este de peste 20. A fost si este nconjurat cu recuno stint a, respect si iubire de cei c arora cu generozitate le-a oferit idei, ndrum ari, ncuraj ari si sfaturi n modul cel mai simplu si resc. Activitatea stiint ic a a prof. C. Corduneanu s-a desf a surat f ar a ntrerupere si continu a si n prezent. Rezultatele obt inute i-au adus recunoa sterea comunit a tii academice internat ionale a matematicienilor. A abordat variate domenii de cercetare: teoria global a a sistemelor diferent iale, teoria admisibilit a tii pentru sisteme de ecuat ii diferent iale si integrale, teoria ecuat iilor cu operatori Voltera abstract i (cauzali), teoria oscilat iilor si undelor (cazul aproape-periodic), c at si aplicat iile acestor teorii la teoria sistemelor (generale sau de reglare automat a). La Ia si a nint at Seminarul de teoria calitativ a a ecuat iilor diferent iale si integrale, ind init iatorul acestei direct ii de cercetare n acest centru universitar. Ca o apreciere a cercet arilor sale, a primit multe distinct ii si premii. In Rom ania a fost distins cu Premiul Ministerului Educat iei 90

(1963), cu Premiul ,,Gh. Laz ar al Academiei Rom ane (1965), iar mai t arziu Prim aria municipiului Ia si i-a conferit Premiul ,,Vasile Pogor (2002). Universitatea de Stat din Texas i-a atribuit Distinguished Research Award (1995), Uniunea Matematicienilor Cehi i-a acordat Medalia de Merit n Matemamatici (2001) etc. In anul 1974 a fost ales membru corespondent al Academiei Rom ane (repus n drepturi ca membru corespondent al Academiei n 1990). Activitatea didactic a si de cercetare stiint ic a este doar o parte din multele direct ii n care prof. C. Corduneanu si-a adus aportul; prolul prezentei reviste nu permite dec at o n sirare pe scurt: autor al unor monograi ap arute n cele mai prestigioase edituri din lume, membru n redact iile unor importante reviste din tar a si str ain atate, membru al unui num ar mare de societ a ti si asociat ii de matematic a, a conferent iat la numeroase universit a ti si institute de cercetare din toate continentele (cu except ia Australiei), participant activ la conferint e si congrese de pretutindeni s. a. Extragem dintr-un interviu un bilant propriu foarte sumar: Sunt bucuros c a pe parcursul carierii mele stiint ice am reu sit s a scriu sase c art i, ap arute n 12 edit ii, toate de circulat ie internat ional a. Am tinut conferint e n peste 60 de universit a ti si academii din diferite t ari, am participat la peste o sut a de conferint e si congrese de specialitate. Tot at atea prilejuri de satisfact ie personal a. In 1981 prof. C. Corduneanu a fondat revista Libertas Mathematica sub egida Academiei Rom ano-Americane de S tiint e si Arte (ARA). Tot el a editat si tip arit revista timp de 31 de ani, p an a n anul 2012, c and prof. Vasile Staicu (Portugalia) a preluat publicarea acesteia. Revista si propune s a capteze energiile matematicienilor rom ani din diaspora si s a introduc a rezulatatele lor remarcabile n mediul academic internat ional. Mai multe volume au fost dedicate (ca volume memoriale sau aniversare) unor distin si matematicieni: Grigore Moisil, Tiberiu Popoviciu, Nicholas Georgescu Roegen, Radu Ro sca s. a. Rubrica Miscellanea include o list a cu numele si adresele matematicienilor de origine rom an a din diaspora (ajuns a acum la circa 500 nume) menit a s a faciliteze contactele dintre ace stia. Ment ion am si faptul c a n perioada 1995-1998 prof. C. Corduneanu a fost pre sedinte al ARA, iar din 1998 este Pre sedinte de Onoare al ARA. Dup a evenimentele din 1989, prof. C. Corduneanu revine n mod constant n Rom ania, de c ateva ori pe an, la locuint a proprie redob andit a ntre timp. Aici si continu a activitatea sau si propune noi proiecte: e c a este vorba de o conferint a n Ia si sau alt centru universitar din tar a, e c a se ngrije ste de aparit ia unui num ar de Libertas Mathematica, e c a preg ate ste aparit ia unei noi c art i etc. Universitatea ,,Ovidius din Constant a si Universitatea ,,Al.I. Cuza din Ia si i-au conferit titlul de Doctor Honoris Causa n 1994, Universitatea ,,Transilvania din Bra sov n 1999, iar Universitatea ,,S tefan cel Mare din Suceava n 2003. Este Honorary Doctor al Universit a tii din Ecaterinburg (Rusia), 2010. Cu ad anc a recuno stint a pentru aportul adus n cre sterea prestigiului internat ional al matematicii rom ane sti si cu admirat ie fat a de exemplul personal de demnitate uman a, ur am prof. Constantin Corduneanu mult i ani de viat a fericit a, cu s an atate, putere deplin a de creat ie si multe mpliniri!

Prof. univ. Temistocle BIRSAN


91

Grupuri nite cu proprietatea (P)


Marius T arn auceanu
1

Abstract. In this paper we characterize the nite groups that have the property (P). Keywords: nite abelian groups, automorphisms. MSC 2010: 20K01, 20K30.

1. Introducere. Problema 2 de la faza judet ean a a Olimpiadei de Matematic a 2013, clasa a XII-a, are urm atorul enunt : Problem a. Un grup (G, ) are proprietatea (P) dac a (P ) f Aut(G), g, h Aut(G) astfel nc at f (x) = g (x) h(x), x G.

S a se arate c a: (a) Orice grup cu proprietatea (P) este abelian. (b) Orice grup abelian nit de ordin impar are proprietatea (P). (c) Niciun grup nit de ordin 4n + 2, n N, nu are proprietatea (P). Solut ia prezentat a n barem se ncheie cu o remarc a interesant a, anume c a exist a grupuri de ordin 4n care au proprietatea (P) - e.g., grupul lui Klein Z2 Z2 - si grupuri de ordin 4n care nu o au - e.g., grupul aditiv Z4 . Cerint ele problemei, mpreun a cu remarca anterioar a, conduc la ntrebarea reasc a: Care sunt grupurile ce satisfac proprietatea (P)? Pentru cazul nit suntem n m asur a s a d am un r aspuns la aceast a ntrebare. Mai precis, vom proba urm atorul rezultat. Teorema 1.1. Un grup nit G are proprietatea (P) dac a si numai dac a G = G1 G2 , unde G1 este un 2-grup abelian de tipul Z21 Z22 Z2k cu |{j | j = i }| 2, i = 1, 2, ..., k, iar G2 este un grup abelian de ordin impar. In particular, putem decide care din grupurile nite de ordin 4n cu n impar au proprietatea (P). Corolarul 1.2. Grupurile nite de ordin 4n, n 1 (mod 2), care au proprietatea (P) sunt de tipul Z2 Z2 G,
1 Lector

dr., Universitatea ,,Al.I. Cuza din Ia si, tarnauc@uaic.ro

92

iar cele care nu au aceast a proprietate sunt de tipul Z4 G, unde G este un grup abelian de ordin n. De asemenea, ment ion am c a exist a si grupuri innite ce satisfac proprietatea (P) (spre exemplu (Q, +)), o clasicare a acestora ind mult mai greu de realizat. 2. Preliminarii. Principalul rezultat ca va utilizat este teorema de structur a a grupurilor abeliene nite (a se vedea, spre exemplu, [3]). Teorema 2.1. Fie G un grup abelian nit. Atunci exist a si sunt unice numerele naturale m, d1 , d2 , ..., dm , astfel nc at G = Zd1 Zd2 Zdm , unde di > 1, i = 1, 2, ..., m, si d1 |d2 |...|dm . Cu notat iile din Teorema 2.1, consider am descompunerile n produse de factori primi ale numerelor d1 , d2 , ..., dm :
ik i1 i2 di = p 1 p2 pk , i = 1, 2, ..., m.

T in and cont c a pentru ecare i are loc izomorsmul Zdi i1 Zpi2 = Zp 1 2 Zp ik ,obt inem
k

(1) unde

G = G1 G2 Gk ,

Gj = Zp1j Zp2j Zpmj si 1j 2j ... mj , j = 1, 2, ..., k.


j j j

Cu alte cuvinte, orice grup abelian nit este produs direct (sau, echivalent, sum a direct a) de p-grupuri abeliene. Urm atoarea teorem a arat a c a studiul grupului automorsmelor unui grup abelian nit se reduce la p-grupuri abeliene (a se vedea, spre exemplu, Lema 2.1 din [2]). Teorema 2.2. Fie H si K dou a grupuri nite de ordine relativ prime. Atunci particular, dac Aut(H K ) a G este un grup abelian nit de = Aut(H ) Aut(K ).In tipul (1), atunci (2) Aut(G) = Aut(G1 ) Aut(G2 ) Aut(Gk ).

Un rezultat mai puternic dec at precedentul l constituie Teorema 3.2 din [1]. Aceasta indic a forma automorsmelor unui produs direct de grupuri nite ce nu au factori direct i comuni. 93

Teorema 2.3. Fie H si K dou a grupuri nite f ar a factori direct i comuni. Atunci Aut(H K ) este izomorf cu grupul multiplicativ
Q

f v

u | f Aut(H ), g Aut(K ), uHom(K, Z (H )), v Hom(H, Z (K )) . g

particular, dac In a grupurile H si K sunt abeliene, atunci Aut(H K ) este izomorf cu grupul multiplicativ
Q

f v

u | f Aut(H ), g Aut(K ), uHom(K, H ), v Hom(H, K ) . g

Incheiem acest paragraf cu o observat ie simpl a, dar extrem de util a. Observat ia 2.4. Un grup abelian (G, +) satisface proprietatea (P) dac a si numai dac a automorsmul identic 1G poate scris sub forma (3) 1G = g + h cu g, h Aut(G).

Intr-adev ar, dac a (3) are loc, atunci pentru orice automorsm f al lui G avem f = f 1G = f g + f h si f g, f h Aut(G). Putem acum proba principalul nostru rezultat. Ment ion am c a nu am inclus n demonstrat ie vericarea propriet a tilor (a)-(c), pentru care poate consultat a solut ia problemei considerate. 3. Demonstrat ia Teoremei 1.1. Presupunem mai nt ai c a G satisface proprietatea (P). Atunci el este abelian si, conform cu (1), admite o descompunere de tipul G = G1 G2 ,unde G1 este un 2-grup abelian, iar G2 este un grup abelian de ordin impar. Din Teorema 2.2 deducem c a Aut(G) = Aut(G1 ) Aut(G2 ), ceea ce arat a c a G1 si G2 satisfac, de asemenea, proprietatea (P). Este sucient s a indic am structura lui G1 . Avem G1 = Z21 Z22 Z2k , unde 1 1 2 ... k . Dac a, prin absurd, exist a i {1, 2, ..., k } astfel nc at j = i , j =1, 2, ..., i1, si K i+1, ..., k , atunci G1 poate scris sub forma G1 = H K,unde H = Z2i = Z21 Z2i1 Z2i+1 Z2k . In plus, remarc am c aH si K nu au factori direct i comuni, a sadar Aut(G1 ) =
Q

f v

u | f Aut(H ), g Aut(K ), uHom(K, H ), v Hom(H, K ) g


Q Q Q

din Teorema 2.3. Atunci 1G1 = 1H 0 0 1K = f1 v1 u1 f + 2 g1 v2 u2 , g2

94

unde fi Aut(H ), gi Aut(K ), ui Hom(K, H ), vi Hom(H, K ), i = 1, 2. Rezult a c a 1H = f1 + f2 , ceea ce constituie o contradict ie (automorsmele lui H sunt de tipul x qx cu q impar, suma a dou a astfel de automorsme neind un automorsm). Reciproc, este sucient s a ar at am c a un 2-grup abelian de tipul G = Z21 Z22 Z2k cu |{j | j = i }| 2, i = 1, 2, ..., k, sau, echivalent, de tipul G = (Z21 )
r1

(Z22 )

r2

(Z2s )

rs

cu ri 2, i = 1, 2, ..., s,

are proprietatea (P). De asemenea, conform Observat iei 2.4, ne putem reduce la cazul s = 1. Avem astfel de probat c a pentru un 2-grup abelian G = (Z2 ) = Z2 Z2 Z2 cu r 2
r ori r

automorsmul identic 1G poate scris ca suma a dou a automorsme ale lui G. T in and cont c a automorsmele grupului abelian G coincid cu automorsmele Z2 -modulului r (Z2 ) , iar acestea se identic a cu matricele inversabile de ordin r peste Z2 , trebuie s a ar at am c a (4) Ar , Br GLr (Z2 ) astfel nc at Ir = Ar + Br .

Vom verica aceast a armat ie prin induct ie dup a r. Pentru r = 2 consider am A2 = iar pentru r = 3 consider am
T

1 1

1 0

si B2 =

0 1 , 1 1 0 1 1 1 0 0 1 0 1

A3 =

1 1 1

1 1 0

1 0 0

si B3 =

Fie r 4. Presupunem c a (4) este adev arat a pentru orice r satisf ac and 2 r < r si o veric am pentru r. Avem Ir = Ar + Br , unde matricele Ar =
Q

Ar2 0

0 A2

si Br =

Br2 0

0 B2

sunt ambele inversabile, ceea ce ncheie demonstrat ia. Bibliograe 1. J.N.S. Bidwell, M.J. Curran, D.J. McCaughan Automorphisms of direct products of nite groups, Arch. Math., 86 (2006), 481-489. 2. C. Hillar, D. Rhea Automorphisms of an abelian p-group, Amer. Math. Monthly, 114 (2007), 917-922. 3. I.D. Ion, N. Radu Algebr a, Editura Didactic a si Pedagogic a, Bucure sti, 1991. 95

Cercurile mixtliniare adjuncte nscrise asociate unui triunghi


Ion PATRAS CU 1
Abstract. In this note there are dened the inscribed adjoint mixtlinear circles associated to a triangle. The main results are Theorem 1 and 2. E.g., the triangle determined by the centers of the inscribed adjoint mixtlinear circles and the triangle tangential to the given triangle are orthological. Keywords: inscribed adjoint mixtlinear circles, Apollonius circle, Brocard s angle, orthological triangle. MSC 2010: 67G40.

In acest articol denim cercurile mixtliniare adjuncte asociate unui triunghi si evident iem c ateva propriet a ti ale lor. Denit ie. Numim cerc mixtilinar adjunct nscris al unui triunghi dat un cerc tangent (interior) cercului circumscris triunghiului ntr-un v arf al s au si tangent la latura opus a v arfului considerat. Observat ie. Evident, pentru orice triunghi, avem trei cercuri mixtliniare adjuncte nscrise. Mai observ am c a avem si trei cercuri mixtliniare adjuncte ex nscrise, care sunt tangente exterior cercului circumscris triunghiului dat si ndeplinesc celelalte condit ii din denit ia de mai sus. Ne vom ocupa numai de cercurile mixtliniare nscrise; le vom numi, dup a v arful triunghilui ABC prin care trec, A-mixtliniar adjunct nscris etc. Propozit ia 1. Punctul de tangent a cu BC al cercului A-mixtliniar adjunct nscris >. este piciorul bisectoarei interioare a unghiului A Demonstrat ie. Not am cu La centrul cercului A-mixtliniar adjunct nscris si cu D contactul acestuia cu latura BC . Fie S intersect ia tangentei n v arful A la cercul circumscris triunghiului ABC cu latura BC ( n gur a m(B ) > m(C )). In mod obi snuit, cu 0 not am centrul cercului circumscris. a In OAC avem: m(OAC ) = 90 m(B ). Apoi, m(OAD) = 1 m(ASLa ) = m(ASC ) = 2 1 1 [m(AC ) m(AB )] = [m(B ) 4 2 m(C )]. Ca urmare, combin and rezul1 tatele obt inute, m(CAD) = m(OAC ) + m(OAD) = 90 m(B ) + [m(B ) m(C )] = 2 1 > > m(A), adic a D este piciorul bisectoarei interioare a unghiului A. 2

.L .O N

1 Profesor,

Colegiul Nat ional ,,Frat ii Buze sti, Craiova

96

Analog se demonstreaz a proprietatea n cazul triunghiurilor obtuzunghice. Dac a triunghiul ABC este isoscel sau echilateral, demonstrat ia este imediat a. Propozit ia 2. Cercul A-mixtliniar adjunct nscris intersecteaz a laturile AB si AC n extremit a tile unei coarde paralele cu BC . Demonstrat ie. Not am cu M si N punctele de intersect ie a cercului A-mixtliniar 1 >) adjunct nscris cu AB , respectiv AC. Avem: m(CDN ) = m(DAN ) = m(A si, pe 2 1 >). Rezult de alt a parte, m(DN M ) = m(M AD) = m(A a c a CDN DN M , ceea ce 2 implic a M N BC. Observat ie. Armat ia M N BC decurge si din omotetia cercurilor A-mixtliniar adjunct nscris si circumscris triunghiului, centrul de omotetie ind v arful A. Propozit ie. Raza rA a cercului A-mixtliniar nscris este dat a de formula: (1) rA = 4p(p a)R . (b + c)2

Demonstrat ie. Solut ia I. Aplic and teorema sinusurilor n AM N si ABC , obt inem: M N = 2rA sin A si a = 2R sin A, de unde rA = R M N. a a AN . Ca urmare, b

Deoarece AM N ABC , avem c a MN = rA =

R AN. b

Puterea punctului C fat a de cercul A-mixtliniar adjunct nscris se scrie: CN CA = ab 2 CD si cum CD = (consecint a a teoremei bisectoarei), rezult a c a CN = b+c a2 b 4p(p a)b . Cum AN = b CN , decurge c a AN = . Inlocuind aceast a 2 (b + c) (b + c)2 expresie g asit a, vom avea rA = R 4p(p a)b 4p(p a)R = b (b + c)2 (b + c)2

si, deci, formula dorit a. Solut ia II. In La AD avem: La A = La D = rA , AD = la (=lungimea bisectoarei 1 din A) si m(La AD) = [m(B ) m(C )]. Atunci, 2 rA = AD 2 cos La AD 97 = la . C 2 cos B 2

2bc A A Folosind formule ca la = cos , cos = b+c 2 2 culelor se obt ine (1).

p(p a) etc., dup a efectuarea calbc

Observat ie. Pentru raza rA ret inem si formulele: (2) rA = cos A bc 2 =R 1 C b + c cos B 2
 Q

a b+c

2

Vom aminti acum c ateva not iuni si rezultate necesare n continuare. Se nume ste cerc adjunct al unui triunghi ABC un cerc ce trece prin dou a v arfuri ale sale si A n unul dintre aceste v arfuri este tangent laturii respective. Un tri unghi are sase cercuri adjuncte. Vom nota cu CA cercul adjunct ce cont ine v arfurile C si A si este . tangent n A laturii AB . Cer curile adjuncte AB , BC , CA au B un punct comun ; analog, cerC curile BA, CB , AC au un punct comun . Punctele si se numesc punctele lui Brocard: este punctul direct al lui Brocard, iar este punctul retrograd. Punctele si sunt izogonal conjugate: AB = BC = CA = si AC = CB = BA = . Unghiul se nume ste unghiul lui Brocard. Cititorul poate g asi alte informat ii n [1].

S a observ am leg atura urm atoare: cercul A-mixtliniar adjunct nscris asociat triunghiului ABC este cerc adjunct pentru triunghiurile ADB si ADC . Ca urmare, avem: Propozit ia 4. Intr-un triunghi ABC n care D este piciorul bisectoarei interioare >, punctele A, D a unghiului A si punctele directe ale lui Brocard corespunz atoare triunghiurilor ADB si ADC sunt conciclice. Urm atoarele dou a teoreme indic a propriet a ti remarcabile ale cercurilor mixtliniare adjuncte nscrise. Teorema 1. Triunghiul La Lb Lc determinat de centrele cercurilor mixtliniare adjuncte nscrise si triunghiul tangent ial Ta Tb Tc sunt ortologice. Centrele lor de ortologie sunt O si centrul radical al cercurilor mixtliniare adjuncte nscrise. Demonstrat ie. Perpendicularele duse din La , Lb , Lc pe laturile corespunz atoare ale triunghiului tangent ial cont in razele OA, OB , respectiv OC ale cercului circumscris. Prin urmare, O este centrul de ortologie al triunghiurilor La Lb Lc si Ta Tb Tc .

98

Tangentele duse din Ta la cercul circumscris, Ta B si Ta C, sunt egale. Ele sunt tangente si la cercurile B -mixtliniar Tb adjunct nscris, respectiv C -mixtliniar adjunct nscris. In consecint a, Ta are puteri egale fat a de aceste cercuri si ca atare apart ine axei radicale a lor. Cum axa radical a a dou a cercuri este A perpendicular a pe linia centrelor acestora, urmeaz a c a perpendiculara din Ta pe La Lc trece prin centrul radi- Tc La . cal al cercurilor mixtliniare adjuncte Lb. O . nscrise, deci acest punct este centru Lc de ortologie al triunghiurilor La Lb Lc B C si Ta Tb Tc .

Fiind date trei cercuri de centre diferite, se nume ste cerc Apollonius al lor ecare dintre cercurile tangente celor trei cercuri date. Evident, cercul circumscris triunghuiului ABC este cerc Apollonius al cercurilor mixtliniare adjuncte nscrise asociate triunghiului.

Ta

Teorema 2. Cercul Apollonius tangent interior cercurilor mixtliniare adjuncte nscrise are cu acestea punctele de tangent a T1 , T2 , respectiv T3 . Dreptele AT1 , BT2 si CT3 sunt concurente. Demonstrat ie. Demonstrat ia face apel la teorema lui D Alembert (de ex., [2]): trei cercuri necongruente si ale c aror centre nu sunt coliniare au cele sase centre de omotetie situate c ate trei pe patru drepte. Intr-adev ar, A este centrul de omotetie direct a a cercului circumscris (O) si a celui A-mixtliniar adjunct nscris (La ); T1 este centrul de omotetie direct a a cercului Apollonius tangent interior cercurilor mixtliniare adjuncte nscrise si a cercului (La ), iar J este centrul de omotetie direct a a cercului Apollonius si a celui circumscris (O). Conform teoremei lui D Alembert, rezult a c a punctele A, J, T1 sunt coliniare. Analog se arat a c a B, J, T2 sunt coliniare si C, J, T3 sunt coliniare. In consecint a, punctul J este punctul de concurent a a dreptelor AT1 , BT2 si CT3 . Bibliograe 1. R.A. Johnson Advanced Euclidean Geometry, Dover Publications, Inc., Mineola, New York, 2007. 2. N.N. Mihaileanu Lect ii complementare de geometrie, Editura Didactic a si Pedagogic a, Bucure sti, 1976.

99

C ateva aplicat ii ale inegalit a tii Ionescu - Weitzenb ock


Dumitru M. BATINET U-GIURGIU 1 , Neculai STANCIU
2

Abstract. The purpose of the present note is to establish some applications of the inequality Ionescu- Weitzenb ock. Keywords: Ionescu-Weitzenb ock inequality, Bergstr om s inequality, Radon s inequality. MSC 2010: 51M16.

ION IONESCU (BIZET ) (1870-1946) s-a n ascut la 22 noiembrie/4 decembrie 1870 n satul Stoienoaia, comuna Creat a-Le sile, judet ul Ilfov. In anul 1889 a reu sit primul la S coala Nat ional a de Poduri si S osele din Bucure sti, unde obt ine diploma de inginer cu nota 18,42 (nota maxim a ind 20). In anul 1895 nint eaz a Gazeta Matematic a mpreun a cu Victor Balaban, Vasile Cristescu, Mihail Roco, Ion Zotta, Emanoil Davidescu, Mauriciu Kinbaum, Nicolae Nicolescu, Tancred Constantinescu si Andrei Ioachimescu. In 31 august 1909, la o sedint a a redact iei Gazetei Matematice, tinut a la via lui Ion Ionescu se decide nint area Societ a tii Gazeta Matematic a. A fost unul din St alpii Gazetei Matematice, al aturi de Vasile Cristescu, Andrei Ioachimescu si Gheorghe T it eica. Din 1921 se dedic a nv a ta m antului de la S coala Politehnic a din Bucure sti, profesor de baz a al acestei institut ii. A condus lucr arile de proiectare si construct ie a numeroase poduri, printre care podul peste Borcea (prima port iune a podului peste Dun are), podul de la Bobolia pe Valea Prahovei etc. In 1900, la Giurgiu se impunea realizarea unui pod peste canalul Sf. Gheorghe, care s a lege portul situat pe malul Dun arii cu celelalte regiuni printr-o leg atur a dubl a sosea-cale ferat a. Coordoneaz a proiectarea si construct ia podului peste un brat al Dun arii din bazinul de la Giurgiu. Condit iile de teren f aceau aproape imposibil a aceast a realizare, dar Ion Ionescu a g asit o solut ie except ional a; podul dublu, cale ferat a si sosea, are o form a curb a, unic a la vremea aceea n lume. Prin aceast a lucrare, inaugurat a n 1905, Ion Ionescu-Bizet a r amas n istoria ora sului Giurgiu. Podul este folosit si n prezent, dup a o utilizare de peste 100 de ani, si poart a numele de Podul Bizet . A elaborat proiectul pentru santierul naval de la Turnu-Severin. Este autorul unui studiu de deviere spre Prut a apelor Siretului n vederea construct iei unei centrale hidroelectrice si a transform arii Prutului ntr-un canal navigabil ntre Galat i si Ia si. A executat harta hidrograc a a bazinului Dun arii. Pentru meritele sale, a fost ales n 1919 membru corespondent al Academiei Rom ane, sect ia stiint ic a. Al aturi de Gheorghe Laz ar, Gheorghe Asachi, Ion Heliade R adulescu, Spiru Haret, Petrache Poenaru si Gheorghe Duca, a fost ctitor al nv a ta m antului tehnic rom anesc. A ncetat din viat a la 17 septembrie 1946. A l asat prin testament Societ a tii Gazeta Matematic a (ast azi numit a SSMR), casa sa din Str. R asuri, nr. 25.

In [1] am ar atat c a Ion Ionescu a publicat cu 22 de ani naintea lui Roland Weitzenb ock inegalitatea a2 + b2 + c2 4S 3, adev arat a n orice triunghi. Intradev ar, n Gazeta Matematic a vol.III, nr. 2 (oct. 1897), p. 52, sub semn atura lui Ion Ionescu apare problema:
1 Profesor, 2 Profesor,

Colegiul Nat ional ,,Matei Basarab, Bucure sti S coala General a ,,George Emil Palade, Buz au

100

273. S a se arate c a nu exist a nici un triunghiu pentru care inegalitatea 4S 3 > a2 + b2 + c2 s a e satisf acut a. Abia n 1919, n articolul Uber eine Ungleichung in der Dreiecksgeometrie publicat n revista Mathematische Zeitung, vol. 5, nr.1-2, pp. 137-146, apare inegalitatea care n mod curent poart a numele lui R. Weitzenb ock. Aceast a inegalitate a fost numit a de noi n [1] inegalitatea Ionescu-Weitzenb ock. In prezenta not a vom da c ateva aplicat ii ale sale. orice triunghi ABC , are loc inegalitatea: Aplicat ia 1. In

a3 b3 c3 4 3 (1) + + S. bR+cr cR+ar aR+br R+r Solut ie. Avem, aplic and inegalitatea lui Bergstr om: U=
:
ciclic

: : a3 a4 (a2 )2 = 2 2 2 bR+cr abR + acr (a + b )R + (a2 + c2 )r

( 2 2
ciclic

ciclic

ciclic

a2 )2
2
ciclic

( r(a2 + c2 ) =2

a2 )2
2
ciclic

a2 .

R(a2 +

ciclic b2 ) +

ciclic

2(R + r)

a2

=
2

ciclic

R+r

Mai departe, aplic am inegalitatea Ionescu- Weitzenb ock, adic a rezult a c aU


4S 3 R+r ,

a2 4S 3, si

ciclic

ceea ce era de demonstrat.

In acela si fel se stabile ste si urm atorul rezultat: m3 m3 m3 3 3 a c b + + S. R mb + r mc R mc + r ma R ma + r mb R+r Solut ie. Avem, utiliz and inegalitatea lui Bergstr om si formula bine-cunoscut a 3 2 2 2 2 2 2 ma + mb + mc = (a + b + c ) : 4 2 : : m3 (m2 a a) V = = R mb + r mc R ma mb + r ma mc (2)
ciclic ciclic

orice triunghi ABC , are loc inegalitatea: Aplicat ia 2. In

:
ciclic

R ( m2 a+ 2(
2
ciclic m2 a+

2 ( m2 a) 2 mb ) + r(m2 a

( + m2 c)
2
ciclic

2 m2 a)

2
ciclic

(m2 a+

ciclic m2 b) +

2 (m2 a + mc )

ciclic

2 m2 a)

m2 a =

2R

2
ciclic

2r

2
ciclic

m2 a

R+r

3 a2 + b2 + c2 , 4 R+r

101

unde folosim inegalitatea Ionescu-Weitzenb ock, i.e. a2 + b2 + c2 4S 3, si deducem c a: 1 3 3 3 V 4S 3 = S. 4 R+r R+r Urm atoarele generaliz ari ale inegalit a tilor (1) si (2) se obt in la fel, dar folosind inegalitatea lui Radon: Aplicat ia 3. Dac a x R+ , atunci n orice triunghi ABC are loc inegalitatea: ax+2 bx+2 cx+2 4 3 (3) + + S. (b R + c r)x (c R + a r)x (a R + b r)x (R + r)x Aplicat ia 4. Dac a x R+ , atunci n orice triunghi ABC are loc inegalitatea: +2 +2 +2 mx mx mx 3 3 a c b (4) + + S. (R mb + r mc )x (R mc + r ma )x ( R ma + r mb ) x (R + r)x Bibliograe 1. D.M. B atinet u-Giurgiu, Neculai Stanciu Inegalit a ti de tip Ionescu-Weitzenb ock, Arch. Math., 86 (2006), 481489.

Utiliz and ca model urm atoarele scheme:

1 2 3 6 6 6

2 4 3 9 24 72

5 1 3 8 15
120

s a se completeze, nlocuind stelut ele cu numere naturale nenule, schemele:

* * * 4 * *

* * * * 7 *

* * * * * 15

(R aspuns la pag. 118) 102

C ateva generaliz ari si ran ari ale inegalit a tii 2 2 2 x +y +z xy + yz + zx


Mih aly BENCZE1
Abstract. In this paper we present some generalizations and renements for classical inequlity x2 + y 2 + z 2 xy + yz + zx. Keywords: classical inequalities. MSC 2010: 97H30.

Propozit ia 1. Dac a x>0 si n N, atunci are loc inegalitatea (1) (x2 + 1)n (xn 1)2 + 2n xn ,

cu egalitate dac a si numai dac a x = 1. Demonstrat ie. Fie funct ia f : RR denit a prin f (x) = (x2 + 1)n (xn 1)2 n n 2 x . Vom scrie f n dou a moduri, dup a cum urmeaz a ( n calcule se tine seama de faptul c a 2n xn =
n 2 k=0 n : k=0 n n k x ):

f (x) =

n 2(nk) x (x2n 2xn + 1) 2n xn = k (x


2(nk)

n : n

k=0

x )x
n

2n

+ 2x 1 =
n

n 1 : k=1

n (x2(nk) xn ); k

f (x) =

n : n

k=0

x2k (x2n 2xn + 1) 2n xn =

n 1 : k=1

n (x2k xn ). k

Prin adunare, obt inem: 2f (x) =


n 1 : k=1

n (xnk xk )2 k

si, deci, f (x) 0, adic a inegalitatea (1). Corolarul 1.1. inegalit a tile (2)
:
ciclic

Dac a ak > 0, k = 1, m, atunci pentru orice n N au loc


n :
ciclic

2 a2 1 + a2 2

(a1 a2 )n +

: 1 : n n 2 ( a a ) (a1 a2 )n , 1 2 2n
ciclic ciclic

cu egalitate dac a si numai dac a a1 = a2 = = am . a1 Demonstrat ie. In (1) lu am x = si obt inem: a2



2 a2 1 + a2 2

(a1 a2 )n +

1 n 2 (a an 2) . 2n 1

1 Profesor,

Colegiul Nat ional ,,Aprily Lajos, Bra sov

103

Scriind si analoagele acesteia si adun and cele m inegalit a ti obt inute (convenim ca am+1 a1 ), deducem prima inegalitate din (2), iar cea de-a doua este evident a. Corolarul 1.2. Dac a ak > 0, k = 1, m, atunci au loc inegalit a tile (3)
m : k=1

a2 k

:
ciclic

a1 a2 +

: 1 : (a1 a2 )2 a1 a2 , 2
ciclic ciclic

cu egalitate dac a si numai dac a a1 = a2 = = am . Observat ie. Evident, (3) reprezint a o generalizare si o ranare a inegalit a tii 2 2 clasice a2 + a + a a a + a a + a a . 1 2 2 3 3 1 1 2 3 Propozit ia 2. Dac a x, y > 0, n N si fn (x, y ) = xy n1 + y n ), atunci (4)

x + y n

1 (xn + xn1 y + + n+1

fn (x, y )

xn + y n , 2

cu egalitate dac a si numai dac a x = y. Demonstrat ie. Proced am prin induct ie complet a at at pentru prima inegalitate din (4) c at si pentru cea de-a doua. x+y x + y
x + y 2 Pentru n = 1 si n = 2 se obt in inegalit a tile adev arate si 2 2 n 2
1 2 x+y fn (x, y ), (x + xy + y 2 ) (echivalent a cu (x y )2 0). Presupun and c a 3 2 vom ar ata c a inegalitatea are loc si pentru n + 1. Intr-adev ar, deoarece

x + y n+1

x + y

x + y

1 (xn + xn1 y + + xy n1 + y n ), n+1

este sucient s a ar at am c a 2 1 1 (xn + xn1 y + + xy n1 + y n ) (xn+1 + xn y + + xy n + y n+1 ), n+1 n+2

ceea ce revine la 2(xn y + xn1 y 2 + + xy n ) n(xn+1 + y n+1 ). Aceast a inegalitate rezult a observ and mai nt ai c a xnk y k+1 + xk+1 y nk xn+1 + n+1 nk y (echivalent a cu inegalitatea evident a (x y nk )(xk+1 y k+1 ) 0), apoi scriind-o pe aceasta pentru k = 0, 1, , n 1 si n nal adun and inegalit a tile obt inute. A sadar, prima inegalitate din (4) este adev arat a. A doua inegalitate din (4) se veric a imediat pentru n = 1 si n = 2; ntr-adev ar, 2 2 x+y 1 2 x + y x+y si (x + xy + y 2 ) (echivalent a cu (x y )2 0). S a avem: 2 2 3 2 104

trecem la etapa inductiv a. Presupunem c a are are loc fn (x, y )


 

xn + y n . Avem: 2


fn+1 (x, y ) =

1 x xn + xn1 y + + xy n1 + y n (n + 1) + y n+1 n+2 n+1 Q n x + yn 1 x (n + 1) + y n+1 = n+2 2 (n + 1)xn+1 + (n + 1)xy n + 2y n+1 . = 2(n + 2) (n + 1)y n+1 + (n + 1)yxn + 2xn+1 . 2(n + 2)

In acela si mod g asim si inegalitatea fn+1 (y, x) Dup a adunarea lor, obt inem: 2fn+1 (x, y ) (n + 3)(xn+1 + y n+1 ) + (n + 1)xy (xn1 + y n1 ) . 2(n + 2)

R am ane s a ar at am c a n+1 (n + 3)(x + y n+1 ) + (n + 1)xy (xn1 + y n1 ) xn+1 + y n+1 , 2(n + 2) care este echivalent a cu xy (xn1 + y n1 ) xn+1 + y n+1 , deci cu (x y )(xn y n ) 0, care este adev arat a. Propozit ia este complet demonstrat a. Corolarul 2.1. Dac a ak > 0, k = 1, m, atunci (5)
m : k=1

an k

:
ciclic

fn (a1, a2 )

:
a1 + a2 n
ciclic

:
ciclic

( a1 a2 )n ,

pentru orice n N , cu egalitate dac a si numai dac a a1 = a2 = = am (cu am+1 a1 ). Demonstrat ie. Observ am c a
m : k=1

a2 k

: an + an 2 1
ciclic

si aplic am Propozit ia 2. acelea Corolarul 2.2. In si condit ii, avem: (6)


m : k=1

a2 k

:
ciclic

f2 (a1, a2 )

:
a1 + a2 2
ciclic

:
ciclic

a1 a2 .

2 2 Observat ie. (6) ofer a noi ranari inegalit a tii clasice a2 1 + a2 + a3 a1 a2 + a2 a3 + a3 a1 .

Bibliograe 1. Octogon Mathematical Magazine, (1993-2012). 105

Caracterizarea unor propriet a ti de perpendicularitate n care sunt implicate punctele O, I, H, G, O9


Andi Gabriel BROJBEANU1
Abstract. In this Note, a couple of conditions of perpendicularity for some lines determined by the vertices of a nonisosceles triangle and the points O, I, H, G, O9 (the center of the nine point circle) are characterized. The obtained results are formulated in Propositions 1-4. Keywords: circumcenter, incenter, orthocenter, centroid, the nine point circle. MSC 2010: 51M04.

In aceast a Not a avem n vedere numai triunghiuri neisoscele. Pentru un triunghi ABC de acest fel, punctele O, I, H, G si O9 (centrul cercului celor nou a puncte) sunt distincte. Amintim c a punctele O, H, G, O9 sunt coliniare, iar dreapta pe care se a a se nume ste dreapta lui Euler. Se stie c a punctul I apart ine dreptei lui Euler a unui triunghi dac a si numai dac a triunghiul este isoscel. A sadar, n condit ia convenit a, punctul I nu apart ine dreptei lui Euler a triunghiului. Mai amintim c a HG = 2OG si c a O9 este mijlocul segmentului HO (a se vedea, de exemplu, [1, 2]). Intre elementele unui triunghi au loc urm atoarele egalit a ti binecunoscute: (1) (2) (3) ab + bc + ca = p2 + r2 + 4Rr, a2 + b2 + c2 = 2(p2 r2 4Rr), abc = 4Rrp.

Pentru distant ele ntre punctele O, I, H, G, O9 au loc formulele: (4) (5) (6) (7) (8) OI 2 = R2 2Rr (Euler), OH 2 = 9R2 + 8Rr + 2r2 2p2 , IH 2 = 4R2 + 4Rr + 3r2 p2 , 1 IG2 = (p2 + 5r2 16Rr), 9 1 IO9 = (R 2r). 2

Formula (8) se demonstreaz a n mod obi snuit utiliz and teorema medianei n IOH relativ la IO9 . Vom folosi ca instrument de lucru urm atoarea
1 Elev,

cl. a X-a, Colegiul Nat ional ,,Constantin Carabella, T argovi ste

106

Lem a. Fie A, B, C, D patru puncte oarecare n plan. Atunci AB CD AC 2 + BD2 = AD2 + BC 2 . Acest rezultat se stabile ste u sor cu teorema lui Pitagora si poate privit ca o extindere a acesteia (la care se reduce dac a unul dintre punctele C sau D coincide cu unul dintre A sau B ). Propozit ia 1. Intr-un triunghi ABC neisoscel, avem: 1) IO9 BC 2a = b + c; 2) IG BC 3a = b + c. Demonstrat ie. 1) Conform Lemei, avem: IO9 BC IB 2 + O9 C 2 = IC 2 + O9 B. Aplic and de dou a ori teorema bisectoarei, se obt in relat iile: IB 2 = ac(p b) , p IC 2 = ab(p c) . p

Pentru calculul termenilor O9 B si O9 C vom utiliza teorema medianei. Ca urmare, IO9 BC ac(p b) R2 + CH 2 OH 2 ab(p c) R2 + BH 2 OH 2 + = + p 2 4 p 2 4 2 a BH 2 CH 2 = [c(p b) b(p c)] p BD2 CD2 = 2a(c b) c2 b2 = 2a(c b) 2a = b + c. (D noteaz a piciorul n alt imii cobor ate din v arful A). 2) Avem: IG BC IB 2 + GC 2 = IC 2 + GB 2
Q
2 2 ac(p b) ab(p c) 2 2 + mc = + mb p 3 p 3 2 2 c 2 b2 ac + a2 + b2 = ab + (c2 + a2 ) 9 2 9 2 2 3 2 a(b c) = (b c2 ) 9 2 3a = b + c

(mb , mc noteaz a lungimile medianelor din B , respectiv C ). Propozit ia 2. Intr-un triunghi ABC neisoscel este adev arat a armat ia: IO9 IG 2a = b + c sau 2b = c + a sau 2c = a + b. 107

Demonstrat ie. Conform teoremei lui Pitagora,


2 IO9 IG IO9 + IG2 = O9 G2 .

1 T in and seama de formulele (5), (7), (8) si faptul c a O9 G = O9 O OG = HO 2 1 1 HO = HO, rezult a c a 3 6 IO9 IG 1 1 1 (R 2r)2 + (p2 + 5r2 16Rr) = (9R2 + 8Rr + 2r2 2p2 ) 4 9 36 9r2 18Rr + p2 = 0.

Prin calcul si utiliz and formulele (1) si (3), avem: (2p 3a)(2p 3b)(2p 3c) = 8p3 12p2 (a + b + c) + 18p(ab + bc + ca) 27abc = = 8p3 24p3 + 18p(p2 + r2 + 4Rr) 27 4Rrp = = 2p(9r2 18Rr + p2 ). Combin and rezultatele precedente, vom obt ine: IO9 IG (2p 3a)(2p 3b)(2p 3c) = 0 (b + c 2a)(c + a 2b)(a + b 2c) = 0 2a = b + c sau 2b = c + a sau 2c = a + b. Propozit ia 3. Intr-un triunghi ABC neisoscel sunt adev arate armat iile: 1) IO AG 2bc = ca + ab, 2) IO9 AG 2(b c)2 = (c a)2 + (a b)2 . Demonstrat ie. 1) Utiliz and din nou Lema, avem: 1 IO AG IA2 + OG2 = IG2 + OA2 IA2 + OH 2 = IG2 + R2 9 bc(p a) 1 1 + (9R2 + 8Rr + 2r2 2p2 ) = (p2 + 5r2 16Rr) + R2 p 9 9 1 2 2 2 bc 4Rr + (9R + 24Rr 3r 3p ) = R2 9 3bc = p2 + r2 + 4Rr 3bc = ab + bc + ca 2bc = ca + ab. 2) Not am cu A1 mijlocul laturii BC . Atunci,
2 2 IO9 AG IO9 AA1 IA2 + O9 A2 1 = IA1 + O9 A

bc(p a) + p

R 2

a2 IB 2 + IC 2 2 4 108

AH 2 + AO2 OH 2 . 2 4

Cum AH = 2R cos A, obt inem c a AH 2 = 4R2 cos2 A = 4R2 (1 sin2 A) = 4R2 a2 . T in and seama de (5), (3) si (2), avem: IO9 AG bc(p a) R2 ac(p b) + ab(p c) a2 5R2 a2 OH 2 + = + p 4 2p 4 2 4 2 2 2 2 2 2 R ac + ab a 5R a 9R + 8Rr + 2r 2p2 bc 4Rr + = 4Rr + 4 2 4 2 2 4 4bc = 2ac + 2ab 3a2 + 2(p2 r2 4Rr) 4bc = 2ac + 2ab 3a2 + (a2 + b2 + c2 ) 2(b2 + c2 2bc) = (a2 + c2 2ac) + (a2 + b2 2ab) 2(b c)2 = (c a)2 + (a b)2 . Propozit ia 4. Intr-un triunghi ABC neisoscel avem IO AI bc = 6Rr. Demonstrat ie. Intr-adev ar, IO AI IO2 + IA2 = OA2 bc(p a) = R2 p 2Rr + bc 4Rr = 0 R2 2Rr + bc = 6Rr. Proced and n acela si mod, se pot g asi si alte rezultate de acest fel. Nu ntotdeauna proprietatea geometric a se caracterizeaz a cu o condit ie de form a simetric a, simpl a. Propunem spre vericare urm atoarele armat ii: 1. IH AG (b c)2 = a(2a b c); A B C 2. IO AI 6Rr = bc 3 sin = 2 cos cos . 2 2 2 Bibliograe 1. T. Lalescu Geometria triunghiului, Editura Tineretului, Bucure sti, 1958. 2. D. Sachelarie Geometria triunghiului. Anul 2000, Matrix Rom, Bucure sti, 2000.

109

La cisso de, podaire de la parabole


Adrien REISNER1
Abstract. The cissoid (C ) is a pedal curve of a parabola (P ) with respect to the point A of (P ). The polar equation of (C ) is = (cos sin )2 p . 2 cos 9p2 of (C ) is a parabola P . 4

Pp
3

, p

The inverse curve with center A and radius k = Keywords: pedal curve, cissoid, inversion. MSC 2010: 14H50.

2 2 o` u p est une longueur constante. D esignons par (P ) la parabole ayant F pour foyer et O pour sommet. Le point A appartient ` a cette parabole. Tout point M de (P ) est rep er e par son ordonn ee egale ` a pt, o` u t est un r eel variable. La perpendiculaire du point Q A sur la tangente en M ` a (P ) coupe cette tangente en H . La tangente au 1 2 1 point M pt , pt ` a (P ) admet pour equation: x ty + pt2 = 0, la perpendiculaire 2 2 1 ` a cette tangente issue de A a pour equation : tx + y pt + p = 0. Finalement, le 2 point H admet pour coordonn ees: (1) H : x = p t p t3 + t 2 , y= . 2 1+t 2 1 + t2

Dans un rep` ere orthonorm e (Ox, Oy ) soitent les points F

,0

et A

, p ,

Dans notre cas on a la d enition suivante: D enition 1. Lorsque M d ecrit la parabole (P ), l ensemble (C ) des points H est appel e podaire de (P ) pour le p ole A. Son equation param etrique est donn ee par (1). On construit la courbe (C ) sans dicult e: dx t2 1 =p , dt (1 + t2 )2 dy p (t + 1)(t3 t2 + 3t + 1) = ; dt 2 (1 + t2 )2

dy t3 t2 + 3 t + 1 = (t = 1). dx 2(t 1) Le tableau de variations est le suivant:


1 TELECOM

ParisTech; e-mail: adrien reisner@yahoo.fr

110

t
dx dt

0 + +

1 0 p 0
p 2

t0 0 +

x y
dy dt

1 0 p 2 0

+ + + 0 +

La courbe (C ) pr esente un point stationnaire pour t = 1: c est le point A. La tangente en A est parall` e le ` a la premi` ere bissectrice et A est un point de rebroussed2 x d2 y p d3 x 3p d3 y ment de premi` ere esp` ece = 2 = , = , = 0 voir gure . dt2 dt 2 dt3 2 dt3 Th eor` eme 2. Trois points Mi (ti ) (C ), i : 1, 2, 3, sont allign es si et seulement si on a la relation: (2) t1 t2 t3 (t1 + t2 + t3 ) = 2.

D emonstration. Soit ux + vy + w = 0 l equation d une droite quelconque du plan: elle coupe (C ) en trois points dont les trois param` etres sont les racines de l equation pvt3 + 2wt2 + p(v 2u)t + 2w 2pv = 0. La courbe (C ) est une cubique (trois points communs avec une droite). Les param` etres 2w cherch es v erient: t1 + t2 + t3 = et pv 2w t1 t2 t3 = + 2, i.e. la relation (2). pv R eciproquement, soient trois points H1 , H2 et H3 de (C ) dont les param` etres v erient la relation (2) du Th eor` eme. La droite H1 H2 rea t coupe (C ) en H3 correspondant ` 3 ; ainsi H1 , es et par suite, compte tenu H2 et H3 sont align du Th eor` eme, on a: t1 t2 t 3 (t1 + t2 + t3 ) = 2. Or, par hypoth` ese nous avons: t1 t2 t3 (t1 + t2 + t3 ) = 2. On en d eduit imm ediatement eor` eme est d emontr e. t3 = t 3 ; donc les points H3 et H3 sont confondus, et ainsi le th 2 4 Corollaire 3. Le point t = 2 : x = p, y = p est point d inexion de la 5 5 courbe (C ). D emonstration. La tangente d inexion ` a (C ) coupe cette courbe en trois points allign es et confondus, donc t1 , t2 et t3 ont la m eme valeur t. Les inexions se trouvent parmis les points dont les param` etres satisfont ` a: t3 3t 2 = 0. La racine double correspond au point A de rebroussement. L inexion correspond ` a la troisi` eme racine 4 2 t = 2 de cette equation. On trouve le point (x = p, y = p). 5 5 111

Soit H (t0 ) (C ). La tangente en H ` a (C ) recoupe (C ) au point K (): K est appel e le tangentiel de H . On a le Corollaire 4. Si les trois points Mi (ti ) (C ), i : 1, 2, 3, sont align es, alors il en est de m eme de leurs tangentiels. D emonstration. La tangente en H (t0 ) ` a (C ) coupe la courbe (C ) en trois points ` partir de la relation (2) du Th allign es: H, H et K . A eor` eme 2, on en d eduit:
t2 u = 0 (2t0 + ) 2 = 0 d o`

2 avec (t0 = 1). t0 1

On en d eduit alors avec des notations evidentes: 1 2 3 (1 + 2 + 3 ) 2 = 2 Le corollaire en r esulte imm ediatement. Cas particuliers. 1) Le tangentiel du point ` a l inni de (C ) est le point D( = 0) situ e sur l asymptote Oy de (C ) (t donne = 0). 2) Si H tend vers le point E (t = 1) de (C ) situ e sur l axe Ox, son tangentiel K s eloigne ` a l inni sur (C ) ( t ) ce qui correspond au parall elisme de la tangente en E et de l asymptote. Remarque. Si un point est confondu avec son tangentiel, on doit avoir t = on trouve le point de rebroussement A(t = 1) et le point d inexion t = 2. 2 ; t1 2 t1 t2 t3 + t1 + t2 + t3 . (t1 1)(t2 1)(t3 1)

Soit R = (AX, AY ) un rep` ere orthonorm e dont l origine est A et dont les axes sont parall` eles ` a Ox et Oy . Th eor` eme 5. L equation cart esienne de la courbe (C ) dans le rep` ere orthonorm e R est p X (X 2 + Y 2 ) + (X Y )2 = 0. 2 D emonstration. Dans le rep` ere R nous avons: X =x On en d eduit t = p p (1 + t)2 p t(1 + t)2 = , Y = y + p = . 2 2 1 + t2 2 1 + t2

Y qui correspond au fait que AH est perpendiculaire ` a la tangente X 1 en M ` a la parabole, cette tangente ayant une pente egale ` a . En eliminant t on t obtient l equation cherch ee. Remarque. La courbe (C ) est une cubique circulaire ((C ) passe par les points cycliques: points de coordonn ees homog` enes (1, i, 0)). Elle poss` ede un rebroussement en A et la tangente de rebroussement admet pour equation X Y = 0. La courbe (C ) est une cisso de. 112

L equation polaire de la cisso de (C ) s ecrit: (3)


Soit Inv A,

9p2 4

p cos + (cos sin )2 = 0. 2 l inversion de p ole A et de puissance

9p2 . On a alors: 4

9p2 Th eor` eme 6. La courbe inverse de (C ) dans l inversion Inv A, est la 4 Q Q 1 7 5 7 p, p . parabole de foyer F p, p et de sommet S 16 16 32 32 D emonstration. La courbe inverse s obtient ` a partir de l equation polaire (3) de 9p2 1 et on revient en coordonn ees la cisso de en conservant et en changeant en 4 kX kY 9p2 cart esiennes. On peut aussi changer X en 2 et Y en 2 o` uk= . 2 2 X +Y X +Y 4 On obtient ainsi 9p X + (X Y )2 = 0 2 soit, en revenant aux axes initiaux:
Q 9p
p 3p x + xy 2 2 2
2

= 0.

C est une parabole (P ) dont l axe est parall` ele ` a la premi` ere bissectrice; elle passe en A (tangente X = 0 ). Cette parabole passe par O (o` u la pente de la tangente est Q 1 5 1 ). Le point B o` u la tangente a une pente nulle a pour coordonn ees B p, p . 2 8 8 Le foyer F de (P ) est la projection sur AB du point de concours des deux tangentes en Q A et en B puisque celles-ci sont perpendiculaires. Les coordonn ees de F sont: 1 7 ees: F p, p . Enn, le sommet S de la parabole (P ) a pour coordonn 16 16 Q 5 7 p, p ; c est l intersection de la parabole avec son axe qui est issu de F et S 32 32 est parall` ele ` a la premi` ere bissectrice: la pente de la tangente en S est 1. Remarque. De fa con g en erale, la pente de la tangente en un point de la parabole est donn ee par Q Q dy 9p 3p 1 = 0, + x0 y 0 4 2 dx dy = 1 est bien l axe de la parabole voir la gure . dx Bibliograe 1. G. Cagnac, E. Ramis, J. Commeau Nouveau Cours de Math ematiques Speciales. Tome 4. Applications de l Analyse ` a la G eom etrie, Masson et Cie, Paris, 1963. 113

Another proof of the I. P atra scu s theorem


Florentin SMARANDACHE
1

Abstract. In this note the author presents a new proof for the theorem of I. P atra scu. Keywords: median, symmedian, Brocard s points. MSC 2010: 97G40.

In [1], Ion P atra scu proves the following Theorem. The Brocard s point of an isosceles triangle is the intersection of a median and the symmedian constructed from the another vertex of the triangle s base, and reciprocal. We ll provide below a dierent proof of this theorem than the proof given in [1] and [2]. We ll recall the following denitions: Denition 1. The symmetric cevian of the triangle s median in rapport to the bisector constructed from the same vertex is called the triangle s symmedian. Denition 2. The points , from the plane of the triangle ABC with the property BA AC CB , respectively AB BC CA, are called the Brocard s points of the given triangle. Remark. In an arbitrary triangle there exist two Brocard s points. Proof of the Theorem. Let ABC an isosceles triangle, AB = AC, and the Brocard s point, therefore BA AC CB = . We ll construct the circumscribed circle to the triangle B C. Having BA CB and CA BC , it results that this circle is tangent in B, respectively in C to the sides AB , respectively AC . We note M the intersection point of the line B with AC and with N the intersection point of the lines C and AB . From the similarity of the triangles ABM , AM , we obtain (1) M B M = AM 2 .

Considering the power of the point M in rapport to the constructed circle, we obtain (2) M B M = M C 2.

From the relations (1) and (2) it results that AM = M C , therefore, BM is a median. If CP is the median from C of the triangle, then from the congruency of the triangles ABM, ACP we nd that ACP ABM = . It results that the cevian CN is a symmedian and the direct theorem is proved.
1 University

of New Mexico, Gallup Campus, U.S.A.

114

A w P N W w B w C M

We ll prove the reciprocal of this theorem. In the triangle ABC is known that the median BM and the symmedian CN intersect in the Brocard s point . We ll construct the circumscribed circle to the triangle B C . We observe that because (3) BA CB,

this circle is tangent in B to the side AB . From the similarity of the triangles ABM, AM it results AM 2 = M B M . But AM = M C , it results that M C 2 = M B M . This relation shows that the line AC is tangent in C to the circumscribed circle to the triangle B C , therefore (4) BC CA.

By adding up relations (3) and (4) side by side, we obtain ABC ACB , consequently, the triangle ABC is an isosceles triangle. References 1. I. P atra scu O teorem a relativ a la punctual lui Brocard, Gazeta Matematic a, nr. 9/1984, LXXXIX, 328-329. 2. I. P atra scu Asupra unei teoreme relative la punctual lui Brocard, Revista Gamma, nr. 1-2/1988, Bra sov. 115

Comentarii pe marginea unor probleme


Mihai MONEA 1 , Stelut a MONEA
2

Abstract. We make some methodic remarks on solving geometry problems and we insist on the existence of the geometric objects before proving their properties. Keywords: Erd os-Mordell inequality, Brocard angle. MSC 2010: 97D50.

Punctul de plecare al acestei Note este Problema 26624 din nr. 6-7-8/ 2012, Gazeta Matematic a, seria B: Problema 26624 ([2]). Fie ABC un triunghi si M un punct n interiorul acestuia astfel nc at m( M AB ) = m( M BC ) = m( M CA) = 30 . S a se arate c a (1) M A2 + M B 2 + M C 2 16S 2 . 9R2

Se pune n mod resc ntrebarea dac a construct ia din ipoteza problemei poate realizat a, adic a: exist a M care s a ndeplineasc a cerint ele problemei? In revistele destinate cu prec adere elevilor apar frecvent probleme n care se trece u sor peste cerint a existent ei obiectului geometric pus n discut ie. Consecint ele apar imediat. Chiar dac a nu suntem n cazul cel mai grav n care obictul nu exist a, apare totu si riscul ca n rezolvarea problemei s a pornim pe c ai mai complicate si nu pe cele re sti si simple. In cazul problemei precedente, vom constata mai jos c a existent a unui punct M ce veric a condit iile cerute impune triunghiului ABC s a e echilateral, iar nsu si punctului M s a e centrul triunghiului. Odat a stabilite acestea, se va constata imediat c a relat ia (1) are loc cu semnul egal. S a presupunem c a problema are ca scop testarea ,,vigilent ei rezolvitorilor. Atunci adev aratul rezolvitor va urma etapele: 1) (partea delicat a) arat a ca ABC este echilateral si M O (centrul triunghiului), 2) (partea banal a) veric a relat ia (1) n care se ia semnul de egalitate. Pe c and, rezolvitorul supercial va stabili ntr-un fel sau altul c a are loc relat ia cerut a, obt in and egalitate pentru cazul n care ABC este echilateral. Excludem situat ia n care o problem a cu unele imperfect iuni vede lumina tiparului. F ar a alte comentarii... Revenind la problema enunt at a mai sus, vom stabili mai nt ai c a ABC este echilateral pe dou a c ai diferite. Instrumentele teoretice ce ne vor ajuta n acest scop sunt: inegalitatea Erd os-Mordell (de ex., [1], 12.13, p.105.) si m asura unghiului
1 Profesor, 2 Profesor,

Colegiul Nat ional ,,Decebal, Deva Colegiul Nat ional ,,Decebal, Deva

116

Brocard trece n al unui triunghi (de ex., [4], 17.13, p. 155). Reamintim aceste rezultate n lemele urm atoare: Lema 1. Fie P un punct interior triunghiului ABC . Dac a x, y, z reprezint a distant ele de la P la laturile triunghiului atunci este adev arat a inegalitatea (2) PA + PB + PC 2 (x + y + z ) .

Egalitatea are loc dac a si numai dac a triunghiul este echilateral iar P este centrul s au. Lema 2. 30 si este 30 dac a si numai dac a ABC este echilateral. Demonstrat ie. Fie punctul lui Brocard (direct) si D, E.F proiect iile sale pe laturile BC, CA, si respectiv AB. Avem: m( BC ) = m( CA) = m( AB ) = si deci au loc relat iile: D = B sin , E = C sin , F = A sin . Aplic and inegalitatea Erd os-Mordell obt inem: A + B + C = 2(B + C + A) sin , de unde 2 sin 1, adic a 30 . Conform, lemei 1 are loc si partea a doua a armat iei. Vom da acum dou a solut ii pentru Problema 26624. Solut ia I. Evident, un triunghi echilateral cu punctul M n centrul s au ndepline ste condit iile problemei. Ar at am c a acesta este singurul caz posibil. Intr-adev ar, dac a exist a punctul M cu proprietatea dat a si not am cu U, V, W picioarele perpendicularelor duse din M pe laturile AB, BC si CA, atunci M A = 2M U si analoagele. Prin adunare obt inem M A + M B + M C = 2M U + 2M V + 2P W , adic a egalitate n (2) si, conform Lemei 1, ABC este echilateral si M este centrul s au. Ca urmare, avem: M A2 + M B 2 + M C 2 = 3R 2 = 3 dar a2 = a2 ; 3

16S 2 16 3a4 3 = = a2 , 2 9R 9 16 a2 deci relat ia (1) are loc cu egalitate. Solut ia II. Din condit iile problemei si Lema 2 rezult a nemijlocit c a ABC este echilateral si M coincide cu centrul s au. Se continu a rezolvarea ca n solut ia precedent a. Prezent am nc a o problem a care se ncadreaz a n contextul n care ne a am. Problema 97 ([3]). Intr-un triunghi ABC, medianele AA , BB , CC fac cu laturile AB , BC si, respectiv CA unghiuri de 30 . Demonstrat i c a triunghiul este echilateral. Solut ia autorului. Dac a unul dintre unghiurile triunghiului are 60 , atunci mediana respectiv a este bisectoare, iar triunghiul va isoscel cu un unghi de 60 , deci echilateral. Dac a toate unghiurile sunt diferite de 60 , construim BA AA , CB BB si AC CC , unde A AA , B BB , C CC . Atunci, avem: a c = BA < BA = , 2 2 a b = CB < CB = , 2 2 117 b c = AC < AC = , 2 2

de unde obt inem c a b c < < < , 2 2 2 2 ceea ce nu este posibil. Solut ie alternativ a. Rezultatul este o consecint a a Lemei 2. Mai mult, condit ia ca AA , BB , CC s a e mediane nu este necesar a. Este important a doar concurent a lor. Bibliograe . Djordjevi 1. O. Bottema, R.Z c, R.R. Jani c, D.S. Mitrinovi c, P.M. Vasi c Geometric Inequalities, Wolters-Noordhoe Publishing, Groningen, 1969. 2. Problema 26624, Gazeta Matematic a, seria B, nr. 6-7-8/2012, p.364 (solut ie n nr. 1/2013, p.23). 3. Problema 97, Revista Minus, nr. 2/2008 (solut ie n nr. 1/2009, p.28). 4. T. Lalescu Geometria triunghiului, Editura Tineretului, Bucure sti, 1958.

R aspuns la ,,recreat ia de la pag. 102:

1 1 2 4 2 4

1 1 7 9 7 63

1 1 3 5 3 15

Fiecare schem a se scrie dup a regula: pe r andul doi si de la st anga la dreapta sunt scrise suma si produsul numerelor aate pe r andul nt ai, iar pe r andul al treilea este scris cel mai mic multiplu comun al numerelor de pe al doilea r and. Evident, numerele scrise pe primul r and se pot permuta (suma si produsul lor ind acelea si).

Vizitat i pagina web a revistei Recreat ii Matematice:

http://www.recreatiimatematice.ro
118

Asupra unei clase de ecuat ii/inecuat ii


Dan POPESCU1
Abstract. The interested reader nds some methodic aspects and comment concerning some selected problems given at mathematical contests. Keywords: real number, complex number, exponential equation. MSC 2010: 97D40.

In aceast a Not a sunt str anse ntr-o clas a un num ar de probleme propuse elevilor la edit iile din ultimii ani ale olimpiadelor de matematic a, ncep and cu etapa local a. In mare spus, este vorba de rezolvarea unor ecuat ii/inecuat ii exponent iale care, spre deosebire de cele studiate la orele de matematic a, au ca necunoscut a si date numere reale si/sau numere complexe. In abordarea acestor ecuat ii se trece frecvent din domeniul real n cel complex, se apeleaz a la interpret ari geometrice sau se utilizeaz a ajut ator funct ii reale monotone. A sadar, rezolvarea problemelor de acest tip necesit a mbinarea cuno stint elor din mai multe capitole. Consider am c a elevii care particip a la diferite concursuri scolare vor g asi n aceast a Not a un material folositor preg atirii lor. Problema 1. Fie a, b R si z C/R, astfel nc at |a b| = |a + b 2z |. a) S a se arate c a ecuat ia |z a|x + |z b|x = |a b|x cu necunoscuta x R, are solut ie unic a. b) S a se rezolve inecuat ia |z a|x + |z b|x |a b|x , cu necunoscuta x R. (Vasile St anescu Problema 26669, GM-B-10/2012 si ONM-2013, etapa judet ean a). Solut ia I (barem olimpiad a). a) Fie z1 = z a si z2 = z b. Rezult a c a z 1 , z2 , z1 + z2 C\R si, n condit ia problemei, |z1 z2 | = |z1 + z2 |. Avem |z1 z2 |2 = |z1 + z2 |2 (z1 z2 )(z 1 z 2 ) = (z1 + z2 )(z 1 + z 2 ) z1 z 2 + z 1 z2 = 0, de unde 2 | + z1 z 2 + z 1 z2 = |z1 |2 + |z2 |2 , adic |z1 + z2 |2 + |z1 |2 + |z2 a |z1 + z2 |2 = |z1 z2 |2 = 2 2 |z1 | + |z2 | , ceea ce permite scrierea ecuat iei sub forma
Q

|z1 | |z1 z2 |

|z2 | |z1 z2 |

=1

sau, echivalent,
T
x

|z1 | |z1 |2 |z2 |2 Funct ia f : R R, denit a prin


T

|z2 | |z1 |2 + |z2 |2

= 1.

f (x) =
1 Profesor,

|z1 | |z1 |2 + |z2 |2

|z2 | |z1 |2 |z2 |2

Colegiul Nat ional ,,S tefan cel Mare, Suceava

119

este strict descresc atoare si ia valoarea 1 pentru x = 2. Ca urmare, ecuat ia dat a are x = 2 ca unic a solut ie. b) Mult imea solut iilor inecuat iei este intervalul [2, ). Solut ia II. De fapt, o variant a a primei solut ii. Se face apel la interpretarea geometric a a not iunii de num ar complex. Cu notat iile de mai sus, avem c a |z1 z2 | = |z1 + z2 |, precum si faptul c a z1 + z2 , z1 z2 C\R. Fie, n planul euclidian raportat la sistemul de coordonate carteziene (O; i, j ), punctele M1 si M2 cu axele z1 si, respectiv, z2 . Dac a M este imaginea geometric aa sumei z1 + z2 , relat ia |z1 + z2 | = |z1 z2 | arat a faptul c a OM1 M M2 este dreptunghi, deci vectorii de pozit ie OM 1 si OM sadar, exist a R , astfel 2 sunt ortogonali. A ca z1 = iz1 . Cum |z1 z2 | = |z1 | 1 + 2 , ecuat ia se scrie n forma
Q

1 1 + 2

| | 1 + 2

= 1.
x

x 1 | | + 2 1+ 1 + 2 g (2) = 1. Atunci ecuat ia are solut ia unic a x = 2.

Funct ia g : R R, g (x) =

este strict descresc atoare si

Problema 2 (enunt modicat). Se consider a numerele complexe z1 sQ i z2 , nc at z 1 z2 = 0. S a se rezolve ecuat ia |z1 |x + |z2 |x = |z1 z2 |x , x R, stiind c a e = 0. z2 (Dan Popescu - ONM-2013, etapa local a, Suceava) Solut ie. Cazul z1 = 0 ind nesemnicativ, e z1 = 0. Avem:
Q

z1 z2

=0

z1 z1 + = 0 z1 z 2 + z 1 z2 = 0. z2 z2

Ca urmare, obt inem |z1 z2 |2 = (z1 z2 )(z 1 z 2 ) = z1 z 1 z1 z 2 z2 z 1 + z2 z 2 = 2 2 |z1 | + |z 2 | . De aici si din faptul c a z1 = 0 si z2 = 0, rezult a c a |z1 | < |z1 z2 |, si analog, |z2 | < |z1 z2 |. Astfel, ecuat ia se scrie echivalent
Q

|z1 | |z1 z2 |

+
x

|z2 | |z1 z2 | |z2 |

=1
x

sau

|z1 | |z1 |2 + |z2 |2

|z1 |2 + |z2 |2

=1

sau f (x) = 1, cu funct ia f : R R denit a ca n problema precedent a. Cum aceast a funct ie este strict descresc atoare si f (2) = 1, obt inem c a 2 este singura solut ie a ecuat iei n cazul z1 , z2 C . Problema 3. S a se determine mult imea M a punctelor din planul Euclidian de ax z C , stiind c a |e||z| + |m z ||z| = |z ||z| . (S erban Olteanu - ONM-2009 etapa local a, Giurgiu) 120

Solut ie. In cazul e z = 0, adic a z = bi, mult imea este M = {(0, b) : b R }. Totodat a, dac a m z = 0, mult imea va M = {(a, 0) : a R }. Fie acum z = a + bi, a, b R . Ecuat ia se scrie echivalent:
Q

a 2 a + b2

a2 +b2

b 2 a + b2

a2 +b2

= 1.

In cazul
Q

a2 + b2 < 2, se deduce inegalitatea


a2 +b2

a a2 + b2

b a2 + b2

a2 +b2

>

a a2 + b2

b a2 + b2

= 1.

La fel, condit ia a2 + b2 < 2 este inacceptabil a. Astfel, n acest caz, mult imea M este cercul centrat n origine si cu raza 2. Problema 4. Fie z1 , z2 , z3 C care ndeplinesc condit iile |z3 |2 = |z1 |2 + |z2 |2 si z1 + z2 + z3 = 0. S tiind c a a (0, 1) (1, ), s a se determine x (0, ) din egalitatea xloga |z1 +z2 | = xloga |z1 +z3 | + xloga |z2 +z2 | . Solut ie. T in and cont de ipoteze, avem: xloga |z1 +z2 | = aloga |z1 +z2 |loga x = |z1 + loga x z2 | = |z3 | etc. Ecuat ia dat a se scrie, echivalent,
loga x

|z3 |loga x = |z2 |loga x + |z1 |loga x , iar aceast a ecuat ie are solut ia unic a loga x = 2, adic a x = a2 . Problema 5. Fie a, b, c, d numere complexe distincte dou a c ate dou a nc at e e ba = da

bc = 0. S a se determine x R, dac a |a b|x + |a d|x |b d|x dc |b c|x + |d c|x . (Enunt propus n Complex Numbers from A to...Z , Birkh auser Boston, 2006, autori Titu Andreescu si Dorin Andrica). Solut ie. Dac a A, B, C, D sunt punctele planului cu axele a, b, c, d, condit iile ba bc e = e = 0 exprim a faptul c a m( BAD) = m( BCD) = 90 . Astfel, da dc |a b| = AB , |a d| = AD si |b d| = BD si prima inegalitate devine AB x + ADx x BD , adic a, echivalent, x 2. Similar, a doua inegalitate conduce la x 2. In concluzie, x = 2 este solut ia problemei. In nal, propunem cititorului o problem a legat a de cont inutul Notei: Problema 6. S a se determine x R, dac a |1 + ab|2x + |a b|2x = (1 + |a|2 + 2 x |b| + |ab| ) , unde a, b sunt numere complexe. (Dan Popescu). Indicat ie. Mai nt ai se analizeaz a cazurile ab = 0 si a = b, iar apoi se apeleaz a identitatea u sor de dovedit |1 + ab|2 + |a b|2 = 1 + |a|2 + |b|2 + |ab|2 , a, b C.
2

121

Rezolvarea unor ecuat ii si inecuat ii integrale


1 Florin STANESCU

Abstract. It is shown how the classical integral inequalities can be used, through the cases when they become equalities, for solving some types of integral equations or inequations. Chebyshev s integral inequality and Jensen s integral inequality are pre-eminently used. Keywords: monotone function, convex function, Cebyshev s integral inequality, Jensen s integral inequality, integral equation. MSC 2010: 97I70.

In aceast a Not a prezent am un procedeu de rezolvare aplicabil unui anumit tip de ecuat ii sau inecuat ii integrale sau unui tip de probleme ce apar adesea n concursurile scolare si reviste. Procedeul const a n urm atoarele: pentru rezolvarea unei ecuat ii/inecuat ii integrale se utilizeaz a o inegalitate integral a clasic a care se dovede ste a egalitate n condit iile problemei si, n consecint a, suntem condu si la solut ia problemei. Amintim inegalit a tile integrale care vor utilizate cu prec adere mai jos: Inegalitatea lui Ceb a sev: Fie f, g : [a, b] R dou a funct ii monotone. Atunci: a) (ba)
a b

f (x)g (x)dx
a

f (x)dx
a

g (x)dx, dac af si g au aceea si monotonie;


b

b) (b a)
a

f (x)g (x)dx
a

f (x)dx
a

g (x)dx, dac af si g sunt de monotonii

diferite. In acestea, avem egalitate dac a si numai dac a una dintre funct iile f, g este constant a (eventual cu except ia unei mult imi num arabile). Inegalitatea lui Jensen. Fie f : I J , g : J R dou a funct ii continue. Atunci: b b 1 1 a) g f (x)dx g (f (x))dx, dac a g este convex a; ba a ba a b b 1 1 b) g f (x)dx g (f (x))dx, dac a g este concav a. ba a ba a Dac a funct ia g este strict convex a (respectiv strict concav a), atunci la punctul a) (respectiv punctul b)) avem egalitate dac a si numai dac a funct ia f este constant a. Problema 1. Determinat i funct iile f : [0, 2] R derivabile de dou a ori, cu derivata f cresc atoare pe [0, 2] si care veric a ecuat ia
2 1
1 Profesor,

f (x)dx
0

f (x)dx =

f (2) f (0) . 2

S coala ,,S erban Cioculescu, G ae sti

122

Solut ie. S tim c a o funct ie : I R este convex a pe intervalul I dac a si numai dac a x, y, z, t I , x < y < z < t, avem: f (z ) f (x) f (t) f (y ) . zx ty Deoarece f este cresc atoare, rezult a c a f este convex a pe [0, 2]; ca urmare, dac a x, y [0, 1], x < y , ceea ce nseamn a c a x < y < x + 1 < y + 1, avem: f (x + 1) f (x) f (y + 1) f (y ), adic a funct ia x f (x + 1) f (x) este cresc atoare pe [0, 1]. Conform inegalit a tii lui Ceb a sev, putem scrie:
1 0

x[f (x + 1) f (x)]dx
0

x dx
0

[f (x + 1) f (x)]dx

(1)

f (2) 2f (1) + f (0) . 2

Pe de alt a parte, cu ajutorul formulei de integrare prin p art i si tin and seama de ecuat ia din enunt , avem:
1 0

x[f (x + 1) f (x)]dx = x[f (x + 1) f (x)] = f (2) f (1)


1 2

0 0

[f (x + 1) f (x)]dx
1

f (t)dt +
0

f (x)dx

= f (2) f (1)

f (2) f (0) 2 f (2) 2f (1) + f (0) = . 2

A sadar, n (1) vom avea egalitate peste tot. Rezult a c a una dintre funct iile c arora leam aplicat inegalitatea lui Ceb a sev este constant a, ceea ce revine la f (x +1) f (x) = k (constant a real a), x [0, 1]. Urmeaz a c a f (x + 1) = f (x), x [0, 1] si deci f (0) = f (1) = f (2). De aici si din faptul c a f este cresc atoare pe [0, 2], deducem c a f este constant a pe [0, 2]. In concluzie, funct iile f : [0, 2] R care ndeplinesc cerint ele problemei au forma f (x) = ax2 + bc + c, a, b, c R. Problema 2. Determinat i funct iile f : [1, 1] R derivabile de dou a ori si cu derivata f continu a pe [1, 1], cresc atoare pe [1, 0] si descresc atoare pe [0, 1] si care ndeplinesc condit ia f (1) f (1) = 2[f (1) 2f (0) + f (1)]. Solut ie. Consider am funct ia g : [1, 1] R dat a de g (x) = x + 1, x [1, 0] x + 1, x [0, 1] 123

si observ am c a ea este continu a pe intervalul [1, 1], cresc atoare pe [1, 0] si descresc atoare pe [0, 1]. In condit iile problemei, putem aplica inegalitatea lui Ceb a sev pe intervalele [1, 0] si [0, 1] funct iilor g si f :
1 1

g (x)f (x)dx = =

0 1 0 1 0 1

g (x)f (x)dx +
0

g (x)f (x)dx
1 0 1 0

g (x)dx

0 1

f (x)dx +

g (x)dx
0

f (x)dx

(x + 1)dx [f (0) f (1)] +

(x + 1)dx [f (1) f (0)]

f (1) f (1) . 2

Pe de alt a parte, integr and prin p art i obt inem:


1 1

g (x)f (x)dx =

0 1

g (x)f (x)dx +
0 0 1

g (x)f (x)dx
1 1

= (x + 1)f (x)

0 1

f (x)dx + (x + 1)f (x)

+
0 0

f (x)dx

= f (0) [f (0) f (1)] f (0) + [f (1) f (0)] = = f (1) 2f (0) + f (1). Combin and rezultatele obt inute si av and n vedere relat ia din problem a, avem: f (1) 2f (0) + f (1) =
1 1

g (x)f (x)dx

f (1) f (1) = f (1) 2f (0) + f (1), 2

deci inegalitatea lui Ceb a sev devine egalitate pe ecare din cele dou a intervale si, ca urmare, funct ia continu a f este constant a pe [1, 1]. In consecint a, pentru funct ia f g asim forma: f (x) = ax2 + bx + c, x [1, 1] (a, b, c R arbitrare). Se veric a direct c a aceste funct ii satisfac cerint ele problemei. Problema 3. Se consider a f : [1, 1] R o funct ie continu a si strict convex a, iar a, b [1, 1] astfel nc at
b

a
0

f (x)dx b
0

f (x)dx = 2ab f

ba . 4

Ar atat i c a ab = 0. Solut ie. Presupunem, prin absurd, c a ab < 0 (analog se rat ioneaz a si pentru ab > 0). Egalitatea din enunt poate scris a sub forma
Q

ba 4

1 2

1 a

f (x)dx +
a

1 b

b 0

f (x)dx .

124

Denim funct ia g : [1, 1] R prin g (x) = ax, x [1, 0] . bx, x [0, 1]

Ea este continu a si nu este constant a. Conform inegalit a tii lui Jensen, putem scrie:

1 2

g (x)dx
1

<

1 2

f (g (x))dx
 Q
1

f f

1 2

ax dx +
1

bx dx
0 0

ba 4

<

1 2

1 < 2 1 b

f (ax)dx +
1 b 0

f (bx)dx
0

1 a

f (x)dx +
a

f (x)dx .

Ultima inegalitate este n contradict ie cu forma n care a fost scris a condit ia din enunt . In concluzie, avem ab = 0. Problema 4. Determinat i funct iile continue f : [0, 2] R pentru care exist a 2 5 a f (x)dx = 2 sin . a R astfel nc at f (x3 + x) = sin ax, x [0, 1] si 8 0 3 Solut ie. Funct ia g : [0, 1] [0, 2] denit a prin g (x) = x + x este bijectiv a, continu a si strict cresc atoare. Prima condit ie se scrie f (g (x)) = sin ax, x [0, 1], sau f (x) = sin(a g 1 (x)), x [0, 2]. A doua condit ie se va scrie
0 2

sin(a g 1 (x))dx =

2 sin

5a . Aplic and inegalitatea lui Jensen, funct ia sinus ind concav a pe [0, 2], obt inem: 8 5a 1 sin = 8 2
2 2

sin(a g
0

(x))dx sin

a 2

g
0

(x)dx .

Putem calcula
0 1 2

g 1 (x)dx cu inegalitatea lui Young (cazul de egalitate):


2 0

g (x)dx +
0 0

g 1 (x)dx = 2 1 0 0

g 1 (x)dx = 2
0

(x3 + x)dx =

5 . 4

Introduc and acest rezultat mai sus, constat am c a n urma aplic arii inegalit a tii lui Jensen obt inem o egalitate. In consecint a, funct ia ag 1 este constant a pe [0, 2] deci a = 0. Prima condit ie din enunt devine f (x3 + x) = 0, x [0, 1], adic a f (x) = 0, x [0, 2]. Bibliograe 1. N. Boboc Analiz a matematic a, Editura Universit a tii din Bucure sti, 1999. 2. M. Ganga Teme si probleme de matematic a, Editura Tehnic a, Bucure sti, 1991. 3. C. Mortici 600 de probleme de matematic a pentru concursuri, Editura Gil, Zal au, 2001. 125

Merge si a sa!
Marian TETIVA
1

Abstract. In this note, a couple of remarks, comments and solving methods regarding equation a cos t + b sin t = c are presented for both cases when a, b, c R, respectively a, b, c C. Keywords: equation, complex number, sinus, cosinus. MSC 2010: 97D40.

Nu, nu m a refer cu acest titlu la binecunoscutul cuv ant de ordine, aplicabil ori de c ate ori ceva funct ioneaz a prost, dar totu si funct ioneaz a. Nu, aici vom vorbi chiar despre o alt a metod a de a rezolva o problem a: e vorba de aarea sinusului si cosinusului unui num ar real t stiind c a ele veric a o ecuat ie de forma a cos t+b sin t = c, unde a, b, c sunt numere reale cu a2 + b2 = 0 si a2 + b2 c2 (condit ia cunoscut a de existent a a solut iilor). Pentru diverse metode de a face acest lucru se poate consulta orice manual de trigonometrie pentru ncep atori. Metoda despre care vorbim noi n continuare este la ndem ana oric arui absolvent de clasa a zecea, dar cred c a nu prea at i v azut-o folosit a n scoala elementar a. Vom utiliza numere complexe: consider am pe z = cos t + i sin t si observ am c a avem 1 1 cos t = (z + z ) si sin t = (z z ), 2 2i unde z este conjugatul complex al lui z , prin urmare ecuat ia se scrie 1 1 a (z + z ) + b (z z ) = c. 2 2i Dar z este un num ar complex de modul 1, deci z = 1/z , ceea ce nseamn a c a ecuat ia noastr a devine una de gradul al doilea (cu coecient i complec si), anume: (a bi)z 2 2cz + a + bi = 0. Av and n vedere c a a2 + b2 c2 discriminantul ecuat iei este = 4(c2 a2 b2 ) 0, deci ea are solut iile c i a2 + b2 c2 z1,2 = , a bi adic a z1 = si ac b a2 + b2 c2 bc + a a2 + b2 c2 + i a2 + b2 a2 + b2

bc a a2 + b2 c2 ac + b a2 + b2 c2 + i. z2 = a2 + b2 a2 + b2
1 Profesor,

Colegiul Nat ional ,,Gheorghe Ro sca Codreanu, B arlad

126

Amintindu-ne c a z = cos t + i sin t si identic and p art ile reale, respectiv imaginare, vedem c a valorile posibile pentru cos t si sin t sunt ac b a2 + b2 c2 bc + a a2 + b2 c2 cos t = s i sin t = a2 + b2 a2 + b2 sau ac + b a2 + b2 c2 bc a a2 + b2 c2 cos t = si sin t = . a2 + b2 a2 + b2 Desigur, n cazul a2 + b2 = c2 , obt inem, practic, o singur a valoare pentru perechea (cos t, sin t) (de fapt: dou a egale). Cititorul este invitat s a se conving a singur c a aceste valori veric a ecuat ia init ial a si c a ele reprezint a, ntr-adev ar, cosinusul, respectiv sinusul unui num ar real - cu alte cuvinte s a vad a c a suma p atratelor lor este egal a cu 1. Probabil c a nici de acum ncolo profesorii nu vor folosi aceast a metod a de rezolvare a ecuat iei a cos t + b sin t = c cu elevii lor. Totu si, trebuie s a-i recunoa stem cel put in un prim merit: acela c a ne permite s a calcul am efectiv cos t si sin t n funct ie de a, b, c, f ar a prea mare efort si ntr-un fel destul de elegant. (Facet i asta si rezolv and sistemul format din ecuat iile a cos t + b sin t = c si cos2 t + sin2 t = 1, cum se procedeaz a de obicei: nu mai e la fel de atr ag ator, nu?) Totu si, e clar c a atunci c and rezolvi prima dat a ecuat ii de acest tip nu pot i folosi numere complexe, n primul r and pentru c a nc a nu s-au studiat. Pe de alt a parte, dup a ce s-a ncheiat si capitolul despre numere complexe (cu tot cu forma lor trigonometric a) poate c a o asemenea privire peste um ar, retrospectiv a, ar avea beneciile ei - ca s a nu mai vorbim de faptul c a utilizarea numerelor complexe n probleme al c aror enunt pare s a nu aib a de a face cu ele reprezint a o metod a puternic a ( si omniprezent a n matematic a) pentru a solut iona probleme dicile. S tiu c a v-at i pus deja o ntrebare ( stiu, pentru c a este o ntrebare natural a): putem oare stabili condit ia de existent a a solut iilor ecuat iei prin aceast a metod a? R aspunsul este: da, dar... Adic a da, se poate, dar mai complicat dec at se face clasic (de exemplu folosind, ntr-o form a sau alta, inegalitatea Cauchy-Schwarz). Pentru asta s a admitem c a am avea 0 = a2 + b2 < c2 , deci ecuat ia cu necunoscuta z are discriminantul num ar real si pozitiv si, prin urmare, solut iile sale sunt c c2 a2 b2 c c2 a2 b2 z1,2 = = (a + bi). a bi a2 + b2 Acum ideea e c a noi c aut am un asemenea z care, n plus, are si modulul 1. Condit ia |z1 | = 1 sau |z2 | = 1 se scrie |c
F

c2 a2 b2 | =

a2 + b2

(pentru o alegere a semnului) si e u sor de vericat c a, dup a ridicarea la p atrat obt inem (indiferent cu care dintre semnele plus/minus am pornit) a2 + b2 = 0 - contradict ie care arat a c a ecuat ia nu poate avea solut ii dac a a2 + b2 < c2 ( si cel put in unul dintre a si b este nenul). Deci se poate: merge si a sa! 127

Mai departe mi-am pus urm atoarea ntrebare (poate deja si cititorul a f acut asta): ce se nt ampl a dac a a, b, c nu mai sunt, neap arat, numere reale? Putem g asi numere t astfel nc at a cos t + b sin t = c? S a ncerc am o alt a abordare ( ntruc at, dac a am vrea s a proced am ca mai sus, neam mpotmoli la rezolvarea ecuat iei, neav and niciun control asupra discriminantului) si anume utilizarea conjugatului - o tehnic a obi snuit a pentru numere complexe. Din a cos t + b sin t = c rezult a (prin conjugare, si tin and seama de faptul c a numerele cos t si sin t sunt reale) a cos t + b sin t = c. Acum s a interpret am aceste dou a ecuat ii ca pe un sistem liniar (cu necunoscutele cos t si sin t), pe care-l rezolv am cu regula lui Cramer (admit and c a ea este aplicabil a). G asim cos t = bc bc ac ac si sin t = , ab ab ab ab

dac a presupunem c a ab = ab ( n particular aceast a metod a nu se poate aplica dac a a si b sunt ambele numere reale, ca n cazul de manual studiat anterior). Desigur num arul t exist a dac a si numai dac a valorile obt inute pentru cos t si sin t sunt numere reale si suma p atratelor lor este egal a cu 1. Se poate u sor vedea c a aceste numere sunt reale, deoarece ecare este egal cu conjugatul s au (w R w = w). Astfel am demonstrat valabilitatea urm atorului enunt : Problema 1. Fie a, b, c numere complexe astfel nc at ab = ab. Exist a un num ar real t care veric a a cos t + b sin t = c dac a si numai dac a (bc bc)2 + (ac ac)2 = (ab ab)2 . L as am (iar!) n grija cititorului interesat detaliile acestei demonstrat ii (e un enunt cu dac a si numai dac a!) precum si s a studieze ce se nt ampl a dac a ab = ab; de exemplu, dac a a, b, c sunt toate reale, condit ia este ndeplinit a, dar existent a lui t nu este asigurat a n toate cazurile, a sa cum am v azut n prima parte a acestei note. La sf ar sit v a propun (ca de obicei) nc a un exercit iu a c arui rezolvare seam an a mult cu cea de mai sus (dac a v a g andit i put in, vedet i c a si enunt urile seam an a, nu?). Observat i, de asemenea, de unde am plecat si de unde am ajuns: ntotdeauna se nt ampl a a sa, dac a vrei mai mult dec at, pur si simplu, s a rezolvi o problem a; adic a dac a t i pui ntreb ari si caut i mereu alte solut ii pentru probleme deja rezolvate (chiar aparent epuizate). Problema 2. Fie a, b, c trei numere complexe cu a = 0 si |a| = |c|. Ecuat ia az 2 + bz + c = 0 are o (singur a) r ad acin a de modul 1 dac a si numai dac a are loc egalitatea |ab bc| = ||a|2 |c|2 |. Observat i, de asemenea, c a enunt ul r am ane valabil si pentru a = 0 (dar nu mai e vorba despre o ecuat ie de gradul al doilea). Ce se poate oare spune despre modulele r ad acinilor ecuat iei dac a avem |ab bc| = ||a|2 |c|2 | si |a| = |c|?

128

Colegiul Nat ional ,,S tefan cel Mare H arl au

La H arl au, Alexandru L apu sneanu nint a n 1551 un Colegiu, prima unitate de nv a t am ant laic a de grad mediu din Moldova 1 . A fost o scoal a domneasc a cu internat, unde educat ia era bine controlat a si asigura o cultur a nalt a. Programa scolar a avea la baz a cunoa sterea limbii latine, a scriitorilor clasici latini, matematicile, stiint ele n general, iar nv a t am antul religios redus. In 1856 ia int a S coala de b aiet i ,,S tefan cel Mare din H arl au, care a funct ionat n actualul local al prim ariei p an a n 1881. Intre anii 1881-1944 va funct iona f ar a ntrerupere ntr-un local nou, construit n 1881 n gr adina bisericii ,,Sf. Dumitru din H arl au - ctitorie a lui Petru Rare s. La nceput a funct ionat cu trei nv a t atori, iar elevii erau mp art it i n patru clase (1856). S coala a f acut fat a nv a t am antului public primar din H arl au aproape sapte decenii, p an a n anul 1949. A sa cum arat a V.M. Li sman1 , unii absolvent i ai ultimelor generat ii, pe care monograstul i-a cunoscut, evocau r avna si d aruirea unor cadre didactice precum: Grigore Rozan, Ion Popovici-Puiu, Neculai C. Ursoi, Ion I. Onu, I. D. Dr agan, Toma C. Zaharia si alt ii. Din 1949 devine gimnaziu mixt, iar n anul 1965, S coala medie va deveni Liceu de cultur a general a conform H.C.M. nr. 903/1965. Din 1974 se nume ste Liceu realumanist, cu 12 clase. Str ans legat de procesul de industrializare are loc transformarea liceelor realumaniste n licee industriale; ca urmare, liceul nostru devine n 1977 liceu industrial,
Iorga, Istoria rom anilor prin c al atori, I, p. 163. M. Li sman, H arl au (Monograe), sub egida Comitetului de Cultur a si Educat ie Socialist a, Ia si, 1972, p. 70.
1 Vasile 1 Nicolae

129

patronat de Ministerul Industriei Chimice. Va avea la nceput clase cu prol de chimie industrial a, mecanic a si mai t arziu se vor ad auga noi proluri ca: industrie u soar a, industria lemnnului si agrobiologie. In 1990, ca urmare a dorint ei cadrelor didactice ale scolii si re nnod and o perioad a de mpliniri, liceul devine Liceul Teoretic ,,S tefan cel Mare prin ordinul nr. 45816/1990 al Ministerului Inv a t am antului si S tiint ei. Numele liceului este numele marelui voievod moldav ca recunoa stere pentru ,,chipul de viteaz, bun, cuminte si sf ant al celui mai mare om ce s-a ridicat dintre rom ani1 . Se reiau clasele cu prol real de chimie-biologie, matematic a-zic a, zic a-chimie si cu prol uman: lologie, lologie-limbi str aine, istorie- stiint e sociale, iar din anul scolar 1992/1993 va lua int a o clas a cu prol de informatic a. In toamna anului 2011 liceul s-a transformat n colegiu Colegiul Nat ional ,,S tefan cel Mare, prin ordin ministerial nr. 4535 din 01.07.2011, recunosc andu-ise astfel acestei scoli si dasc alilor ei aportul avut n nv a t am antul rom anesc. Dintre absolvent ii scolii noastre, cu nume de prestigiu n domeniile n care au activat, enumer am: Vasiliu Titu (1885-1961), elev si urma s al savantului Victor Babe s, fondatorul scolii de anatomie patologic a din Cluj, doctor n medicin a si chirurgie; Gheorghe Leahu, directorul Teatrului Nat ional din Timi soara (1956-1970); Lucia T ibuleac, solist a a Operei Rom ane din Bucure sti; Ioan Emanoil Gandrabura, prof. la Universitatea ,,Al.I. Cuza Ia si, Facultatea de geograe si geologie; Natan Cohen, prof. la Universitatea ,,Ben Gurion din Negev, Israel; Carol Iancu, prof. de istorie contemporan a la Universitatea ,,Paul Valery din Montpellier, Doctor Honoris Causa al Universit a tii ,,Babe s-Bolyai Cluj si al Universit a tii ,,Al.I. Cuza Ia si si mult i alt ii. Colegiul Nat ional ,,S tefan cel Mare din H arl au organizeaz a anual Concursul nat ional ,,Micii Matematicieni, aat n anul 2013 la edit ia a VIII-a, si public a revista cu acela si nume. Aceast a manifestare este girat a de SSMR si de ISJ Ia si. Intre concursurile derulate de colegiul nostru se a a si Concursul literar ,,Cezar ascut n decembrie 1892 la Hodora, Cotnari. Petrescu, dedicat memoriei scriitorului n La concurs au participat: Colegiul Nat ional ,,M. Sadoveanu Pa scani, Liceul Teoretic ,,I. Neculce Tg. Frumos, Colegiul Tehnic de C ai Ferate ,,Unirea Pa scani, Grupul S colar ,,M. Busuioc etc. Colegiul Nat ional ,,S tefan cel Mare H arl au organizeaz a anual Concursul de web ,,HWEB, care const a n crearea unei pagini web cu tema ,,Afacerea mea pe internet si un test-gril a referitor la cuno stint ele elevilor din domeniul IT si web. Clubul de turism si ecologie ,,Ecomont a fost nint at n Liceul Teoretic ,,S tefan cel Mare H arl au la nceputul anului 1996 si a avut de-a lungul timpului numeroase activit a ti dedicate protejarii mediului, iar prin intermediul ei s-au organizat excursii de studiu si de agrement n diverse zone ale t arii. Cercul de pictur a religioas a ,,Trepte c atre cer desf a soar a n timpul vacant ei de iarn a activit a ti de pictur a religioas a si are n vedere descoperirea si cultivarea talentelor si abilit a tilor artistice ale elevilor. Cercul de art a dramatic a ,,Arlechino se a a n al cincilea an de existent a, se reune ste periodic, de trei ori pe lun a, si exerseaz a pe diverse texte dramatice sub
1 Nicolae Iorga, Istoria lui S tefan cel Mare, edit ie ngrijit a si tabel cronologic de Victor Iova, Bucure sti, Editura Minerva, 1978.

130

ndrumarea prof. Iancu Mirela. Asociat ia sportiv a scolar a ,,Olimpia a fost nint at a n anul 2010. Profesorii coordonatori sunt Butnariu Marius Doina s, Danielevici Iulian Constantin, iar pre sedinte - prof. S ac aleanu Ioan. Asociat ia si popune promovarea n r andul elevilor a urm atoarelor sporturi: atletism, baschet, badminton, fotbal, gimnastic a ritmic a, handbal, tenis de mas a, tenis de c amp, oin a, sah, volei. Clubul ,,Micilor matematicieni a fost nint at n anul 2002, mai nt ai sub denumirea de Cercul elevilor ,,Pro-matematica. Are ca scop preg atirea elevilor colegiului nostru pentru concursurile scolare, prin rezolvarea problemelor propuse n reviste de specialitate. Init iatorul acestui cerc si apoi al revistei Micii matematicieni este prof. Ioan S ac aleanu. Iar din 2006 a luat int a Concursul Micii Matematicieni, coordonat de prof. Aurel Neicu, av and ca pre sedinte pe prof. Gheorghe Oancea. Proiecte si campanii internat ionale. Ment ion am doar dou a dintre multele proiecte pe care colegiul le-a derulat. In spiritul cooper arii si al valoriz arii sistemului educat ional din Europa, n anul scolar 2011/2012 prof. Aurel Neicu a f acut o aplicat ie pentru proiectul internat ional individual Grundvig pe tema exploat arii si inov arii n domeniul mediului si al istoriei. Activit a tile proiectului ,,Dry stone in Europe s-au desf a surat n localitatea Arlanc/Frant a si au constat n reabilitarea unui vechi sit cu valent a cultural a. Tot n anul scolar 2011/2012 a fost c a stigat prin concurs un proiect internat ional Comenius cu tema ,,Mo stenirea istoric a n sprijinul construirii Europei de c atre o echip a format a din prof. Adriana G andac, prof. Aurel Neicu, prof. Pu sa Alexa. Proiectul se desf a soar a n intervalul 2012-2014 si va consta n mobilit a ti parteneriale n Frant a, Germania, Turcia, Italia, Bulgaria si Polonia. Prin munca multor generat ii de elevi si profesori Colegiul Nat ional ,,S tefan cel Mare a dob andit un renume pe care actuala generat ie de elevi si dasc ali se oblig a s a-l p astreze si s a-l sporeasc a prin munc a, talent si d aruire, acest fapt ind o obligat ie si un leg am ant de onoare.

Prof. Aurel NEICU Director al C.N. ,,S tefan cel Mare H arl au

ERRATUM
D-l Prof. Dumitru Barac semnaleaz a faptul c a n enunt ul problemei X.127 din Recreat i Matematice nr. 1/2012 trebuie ad augat a condit ia a2 > bc.

Vizitat i pagina web a revistei Recreat ii Matematice:

http://www.recreatiimatematice.ro
131

Concursul de matematic a ,,Florica T. C ampan


Edit ia a XIII-a, Ia si, 2013

Clasa I
I.1. Doar 5 numere respect a regula de alc atuire a sirului. Care este intrusul? Incercuie ste-l! 73, 28, 52, 19, 46, 55. I.2. Descoper a regula de formare a sirului, apoi completeaz a: 2, 6, 4; 3, 7, 5; 4, 8, 6; 5, . . . , . . . ; 6, . . . , . . . .

I.3. Completeaz a p atratul din gur a cu numerele 1, 2, 3 si 4 astfel nc at suma numerelor de pe ecare linie (pe orizontal) si de pe ecare coloan a (pe vertical) s a e 10. II.1. Un uture zboar a din oarea 1 n oarea 3, apoi din aceasta n oarea 5 si a sa mai departe. Dup a c ate zboruri ajunge n oarea de pe care a plecat?
9 8 7 6 5 4 1 2 3

II.2. Iepuril a are 18 ou a de ciocolat a. Iepurica are cu 3 ou a mai put in dec at Iepuril a, dar cu 10 ou a mai mult dec at Ril a. S a se ae c ate ou a de ciocolat a are Ril a! III.1. Corina a cules 13 lalele galbene si 4 lalele ro sii. Ema a cules 7 lalele, unele galbene, altele ro sii. Fetit ele pun toate lalelele n vaz a. Care este cel mai mare num ar de lalele galbene care ar putea n vaz a? III.2. De la apartamentul meu cobor 4 etaje, apoi urc 3 etaje si observ c a sunt la etajul 9. La ce etaj locuiesc?

Clasa a II-a
I.1. G asit i numerele care lipsesc din urm atorul sir: 1, 1, 2, 3, 5, 8, 13, . . . , . . . , . . .. I.2. V arsta bunicii este un num ar cuprins ntre 60 si 70 de ani. S tiind c a diferent a dintre cifra unit a tilor si cea a zecilor din acel num ar este 3, a a ce v arst a are bunica. II.1. Dintre Alin, Marin, Corina, Ioana si Raluca trebuie s a aleg membrii unei echipe formate din dou a fete si un b aiat. In c ate moduri o pot face? 132

II.2. Timp de sase zile, veverit a a c arat n scorbura sa ghinde astfel n ecare zi cu 17 ghinde mai mult dec at n ziua precedent a. S tiind c a n a patra zi a c arat 68 de ghinde, a a c ate ghinde a adunat n total, n cele sase zile. III.1. Dac a dou a p apu si si o ma sinut a cost a 8 lei, iar o p apu s a si dou a ma sinut e cost a 7 lei, c at cost a nou a p apu si si nou a ma sinut e? III.2. Pe trei rafturi sunt 75 de c art i. Dac a pe primul raft punem jum atate din c art ile de pe cel de-al doilea, atunci vom avea pe rafturi, n ordinea lor, numere consecutive de c art i. C ate c art i erau la nceput pe ecare raft?

Clasa a III-a
I. Veverit ele Lia, Mia si Kia adun a alune. Impreun a au adunat un num ar de alune egal cu cel mai mare num ar par de trei cifre. Lia si Mia au adunat un num ar de alune egal cu cel mai mare num ar de trei cifre ce are suma cifrelor 5. Mia si Kia au adunat un num ar de alune egal cu cel mai mare num ar par de trei cifre ce are suma cifrelor 10. Care din cele trei veverit e este cea mai h arnicut a? (Justicat i!) II.1. Un num ar A se va numi angelic dac a suma cifrelor lui A este egal a cu num arul cifrelor lui A. (Spre exemplu 2011 si 200013 sunt numere angelice deoarece 2+0+1+1 = 4 si 2 + 0 + 0 + 0 + 1 + 3 = 6). Scriet i toate numerele angelice care au trei cifre. II.2. Aat i suma numerelor de trei cifre care au produsul cifrelor egal cu 8. III. Petrut s-a n ascut n anul 19xy , iar tat al s au n anul 19yx. C and Petrut avea 25 de ani, tat al s au avea 52 de ani. Aat i n ce an s-a n ascut Petrut si tat al s au, stiind c a n anul 2013 au mpreun a 105 ani.

Clasa a IV-a
I (Nu v a jucat i cu chibriturile!). a) Construim p atrate din chibrituri. Primul p atrat are latura de un chibrit, al doilea din trei chibrituri, al treilea din cinci chibrituri etc. Aat i cu c ate chibrituri folosim mai mult la construct ia p atratului cu num arul 100 dec at la construct ia p atratului cu num arul 99. b) Care este suma perimetrelor primelor 100 de p atrate astfel formate? II (Ferit i-v a de aviar a!). Speriate teribil de efectele virusului gripei aviare (H5 N1 ), g ainile dintr-o gospod arie au plecat gr abite s a fac a un control am anunt it la medicul veterinar. Transpirate si obosite dup a drumul parcurs, s-au oprit ntr-un parc s a se odhneasc a. Ele constat a c a dac a s-ar a seza c ate zece g aini pe o banc a r am an sase b anci goale, iar pe o banc a ar numai trei g aini. Dac a s-ar a seza c ate sase g aini pe c ate o banc a, ar exista o singur a banc a cu numai trei g aini, iar restul b ancilor ar ocupate cu c ate sase g aini. C ate g aini are gospodarul si c ate b anci sunt n parc? III (R azboiul numerelor ,,stelare). Un num ar S se nume ste ,,stelar dac a suma cifrelor lui S este egal a cu num arul cifrelor lui (spre exemplu 2011 este num ar ,,stelar deoarece are patru cifre si 2 + 0 + 1 + 1 = 4). a) Scriet i toate numerele ,,stelare de patru cifre. b) Determinat i diferent a dintre cel mai mare si cel mai mic num ar ,,stelar care au c ate 100 de cifre. Care este suma cifrelor acestui num ar obt inut ca diferent a? 133

Clasa a V-a
I. Un album foto are paginile numerotate de la 1 la 80. Pe ecare pagin a care este multiplu de 2, dar nu si de 3, sunt c ate 2 fotograi. Pe paginile care sunt multipli de 3, dar nu si de 2 sunt 5 fotograi. In rest, paginile sunt ocupate cu c ate o fotograe. C ate fotograi apar n album? II. Un grup de copii care particip a la o aniversare constat a c a media v arstelor lor este egal a cu num arul lor. Dup a ce apare, neinvitat, Ionel, n v arst a de 15 ani, copiii constat a c a noul grup are aceea si proprietate. C a ti copii erau, init ial, la aniversare? 2013 2013 III. Dac a num arul 2 are m cifre si 5 are n cifre, s a se ae m + n.

Clasa a VI-a
39 85 133 69 145 217 I. a) Ar atat i c a + + este un num ar natural. 65 119 209 115 203 341 b) Intr-o zi din ianuarie 2013, din autogara Ia si pleac a n curs a trei autocare c atre Spania, Italia si Germania, care se re ntorc n autogar a dup a 6, 8 si respectiv 11 zile. Dup a ce stat ioneaz a ecare n autogar a c ate 4 zile are loc o nou a plecare. Aat i de c ate ori n cursul anului 2013, cele trei autocare pleac a n curs a n aceea si zi din autogara Ia si. 2y + 7 7x + 2 N si V = y N N . II. Fie mult imile U = x N 11 11 a) Ar atat i c a: U = si V = ; 8x + 29 61y 1 b) Demonstrat i c a N, oricare ar x U si N, oricare ar 11 11 y V. c) Determinat i U V. III. Punctele D, E si F sunt situate pe latura (BC ) a triunghiului ascut itunghic ABC astfel nc at AD BC, (AE ) (AF ), E [BD] si F [CD]. Punctele B si C apart in laturilor (AC ), respectiv (AB ) astfel nc at ACC BAE si ABB CAF , iar punctul O este intersect ia dreptelor BB si CC . a) Demonstrat i c a (BO) = (CO). b) Dac a punctul O este egal dep artat de v arfurile triunghiului ABC , determinat i pozit iile punctelor E si F pe (BC ). c) Dac a BB AE = {M } si CC AF = {N }, iar (AM ) (AN ), atunci ABC este un triunghi isoscel.

Clasa a VII-a
I. Ana, Dan si S tefan si-au cump arat scutere. Ast azi Ana a mers cu o or a mai mult dec at Dan si cu o vitez a cu 5 km/h mai mare dec at a acestuia. S tefan a mers cu dou a ore mai mult dec at Dan si cu o vitez a cu 10 km/h mai mare dec at a acestuia. Ana a parcurs cu 70 km mai mult dec at Dan. Aat i care este diferent a dintre distant a parcurs a de S tefan si cea parcurs a de Dan. II. O gr adin a n form a de triunghi are aria de 2 ari. Proprietarul vrea s a o mprejmuiasc a cu un gard cu trei r anduri de s arm a. Stabilit i dac a i ajung 18 dam de s arm a. Justicat i r aspunsul. 134

III. Zece elevi organizeaz a un turneu de sah dup a urm atoarele reguli: - ecare joac a exact o partid a cu ceilalt i nou a juc atori; - se acord a un punct pentru cel ce c a stig a, zero puncte pentru egalitate si se scade un punct nvinsului. La sf ar situl turneului se constat a c a mai mult de 70% din meciuri s-au terminat la egalitate. S a se arate c a exist a doi juc atori care au acela si punctaj.

Clasa a VIII-a
I (O l acust a s alt areat a). Pe un plan raportat la un reper cartezian XOY , se joac a o l acust a s arind din punct n punct dup a regula ,,dac a la un moment dat l acusta este n puctul A(a, b), atunci ea poate s ari n oricare dintre punctele de coordonate: (a 1, b 3) sau la (a 3, b 1), semnele + si pot luate n toate modurile posibile (prin urmare l acusta, din A, poate s ari n unul din cele opt puncte enumerate). Dac a la momentul init ial, l acusta este n O(0, 0) se cere: a) Stabilit i un traseu prin care l acusta ajunge n punctul M (1, 1). b) Ar atat i c a l acusta, oric ate s arituri ar face, nu ajunge n punctul N (2013, 2014). c) Poate ajunge l acusta n punctul P (2001, 2013)? Justicare. II (O problem a dulce). Radu si Andrei cump ar a ecare c ate o cutie cu bomboane de ciocolat a n form a de sfer a. Cutiile au form a cubic a cu aceea si muchie n N . Radu are n cutie bomboane cu raza r, iar Andrei bomboane cu raza 2r, r N . In cutii bomboanele sunt a sezate n straturi astfel nc at ecare strat s a cont in a num arul maxim posibil. S tiind c a 4r divide n, stabilit i care dintre cutiile cump arate c ant are ste mai mult. III (O problem a cu ,,torturi). Cofetarul Gic a inventeaz a la Cofet aria ,,Pr ajitura minunat a situat a peste drum de McDonald s, un tort de ciocolat a ce urmeaz a a utilizat la festivitatea de comemorare a ,,Regelui dulciurilor. Me sterul Gic a face un tort din 30 de cubulet e de ciocolat a pe care le a saz a piramidal, unul peste altul ca n gura al aturat a astfel nc at axele verticale ale cubulet elor s a e situate pe aceea si dreapt a. Se mai stie c a lungimile muchiilor cubulet elor sunt exprimate n decimetri prin numerele rat ionale: 39 40 11 12 13 , , ,..., si, respectiv . 10 11 12 38 39 a) Ar atat i c a n alt imea tortului construit de me sterul Gic a nu dep a se ste 32 dm. b) Dac a me sterul Gic a dispune de un calup de ciocolat a care se a a ntr-o cutie plin a n form a de paralelipiped dreptunghic cu dimensiunile de 1 dm, 1 dm si 37 dm, ar atat i c a acesta poate face tortul.

135

Solut iile problemelor propuse n nr. 1/2013


Clasele primare
P.255. Dorina are un num ar de mere egal cu cel mai mare num ar scris cu o cifr a. Eu a s avea c at Dorina, dac a a s mai avea 2 mere. C ate mere am eu? (Clasa I ) Inst. Maria Racu, Ia si Solut ie. 9 2 = 7 (mere). P.256. Doamna nv a ta toare a scris pe tabl a sase litere m, sase litere i si sase litere n. C ate bastona se a folosit doamna nv a t atoare pentru scrierea acestor litere? (Clasa I ) Dumitrit a Grigoriu, elev a, Ia si Solut ie. Litera m cont ine dou a bastona se si o zal a, litera n cont ine un bastona s si o zal a, iar litera i este format a dintr-un singur bastona s. S-au folosit 24 bastona se. P.257. Completat i c asut ele goale din sirul 1, 1, 2, 1, 2, 3, 1, 2, , , , , , , . (Clasa I ) Mihaela Buleandr a, elev a, Ia si Solut ie. C asut ele trebuie completate cu cifrele: 3, 4, 1, 2, 3, 4, 5. P.258. Aat i numerele a, b, c si d, stiind c a a 18 = b 15 = c 8 = b c 1. (Clasa a II-a) Tatiana Ignat, elev a, Ia si Solut ie. Din egalitatea b 15 = b c 1 obt inem 15 = c + 1, de unde c = 14 si c 8 = 6. Urmeaz a c a a 18 = 6, a = 24 si b 15 = 6, b = 21. P.259. Fie sirul de numere 1, 2, 3, . . . , 12. Se formeaz a grupe de c ate trei numere luate din acest sir. C ate dintre grupe pot avea suma numerelor egal a cu 30? (Clasa a II-a) Ionut Airinei, elev, Ia si Solut ie. Observ am c a n aceast a sum a nu putem avea termeni mai mici ca 7, deoarece 6 + 11 + 12 = 29 < 30. Analog, n sum a nu putem avea tot i termenii mai mari ca 9, c aci 10 + 11 + 12 = 33 > 30. R am ane c a ntr-o sum a trebuie s a avem unul dintre numerele 7, 8, 9. Cercet and cazurile posibile, obt inem sumele: 7 + 11 + 12 = 30, 8 + 10 + 12 = 30 si 9 + 10 + 11 = 30. Concluzion am c a exist a numai trei grupe de numere care ndeplinesc condit iile problemei. P.260. Bomboanele dintr-o pung a se mpart la cinci copii. Fiecare copil prime ste cel mult patru bomboane si cel mult trei copii au acela si num ar de bomboane. Care este cel mai mare num ar posibil de bomboane din pung a? (Clasa a II-a) Alexandra Tololoi, elev a, Ia si Solut ie. In pung a trebuie s a avem cel mult 4 + 4 + 4 + 3 + 3 = 18 bomboane. P.261. Scriet i num arul 66 ca sum a de numere naturale al c aror produs este tot 66. C ate solut ii are problema? (Nu se va tine seama de ordinea termenilor.) (Clasa a III-a) Monica R aileanu, elev a, Ia si 136

Solut ie. 66=2 3 11 = 6 11 = 2 33 = 3 22. Avem: 66= 1 + 1 + . . . + 1 +2+3+11;


50 de 1

66 = 1 + 1 + . . . + 1 +6+11; 66 = 1 + 1 + . . . + 1 +2+33; 66 = 1 + 1 + . . . + 1 +3+22.


49 de 1 31 de 1 41 de 1

P.262. Ar atat i c a un p atrat din carton poate mp art it, f ar a pierdere de material, n 31 de p atrate. (Clasa a III-a) Andreea Simona Simion, elev a, Ia si Solut ie. D am urm atorul algoritm de mp art ire a unui p atrat:

...
4 patrate (4+3x1) patrate (4+3x2) patrate

Deoarece 31 = 4 + 3 9, mp art irea este posibil a. P.263. Tat al termin a o lucrare n 10 ore, iar ul n 15 ore. In c ate ore termin a lucrarea cei doi, dac a lucreaz a mpreun a? (Clasa a III-a) Paula Balan, elev a, Ia si Solut ie. Reprezent am volumul lucr arii printr-un segment, pe care l mp art im n 30 de p art i egale. Intr-o or a, tat al acoper a 3 p art i, iar ul 2 p art i, n total 5 p art i. Rezult a c a lucarea va terminat a n 30 : 5 = 6 ore. P.264. Dac a extragem o coal a dintr-un ziar, observ am c a suma celor patru numere care indic a paginile este 50. Putet i spune cu ce numere este paginat a coala din mijloc? (Clasa a III-a) Codrut a Filip, elev a, Ia si Solut ie. Suma celor patru numere care indic a paginileunei coale este aceea si. Pe coala din mijloc avem 4 numere consecutive cu suma 50. Acestea sunt: 11, 12, 13, 14. P.265. Fie numerele nenule a, b, c, d astfel nc at a + b = 18, b + c = 14, c + d = 10. Calculat i suma 5a + 6b + 8c + 7d. (Clasa a IV-a) Nicolae Iv a schescu, Craiova Solut ie. 5a +6b +8c +7d = 5 (a + b)+(b + c)+7 (c + d) = 5 18+14+7 10 = = 90 + 14 + 70 = 174. P.266. Maria are ast azi de sase ori v arsta pe care o avea c and fratele ei, Alexandru, avea v arsta ei actual a. C and ea va avea v arsta de azi a fratelui ei, cei doi vor avea mpreun a 27 de ani. Ce v arst a are acum Maria? (Clasa a IV-a) Irina C apraru, Ia si Solut ie. S a gur am v arstele celor doi frat i, raport andu-ne la prezent (P):

(P) M p A p 27 ani

137

11p + 16p = 27 27p = 27 p = 1 (an). Maria are acum 6 1 an = 6 ani. P.267. Pentru un grup de copii s-au adus de trei ori mai multe mandarine dec at portocale. Fiecare copil trebuia s a primeasc a 2 portocale si 5 mandarine, ns a din gre seal a doi copii au primit c ate o mandarin a n plus. S tiind c a au r amas 19 portocale si 92 mandarine, s a se ae c a ti copii sunt n grup. (Clasa a IV-a) Mariana Nastasia, elev a, Ia si Solut ie. Dac a mandarinele ar fost mp art ite corect, atunci ar r amas 92 + 2 = 94 mandarine. Dac a not am cu ,,a num arul copiilor, atunci 3(2a + 19) = 5a + 94, de unde a = 37 (copii). P.268. Pe o foaie sunt scrise numerele de la 1 la 20. S a se arate c a nu pot formate, din cele 20 de numere, grupe de c ate patru numere astfel nc at n ecare grup a suma a dou a numere s a e triplul sumei celorlaltor dou a. (Clasa a IV-a) Andreea B zd g a, elev a, Ia si Solut ie. 1 + 2 + 3 + . . . + 20 = (1 + 20)20 : 2 = 210. Dac a numerele ar putea grupate c ate 4, satisf ac and condit ia problemei, atunci suma numerelor din ecare grup a se mparte exact la 4, deci si suma celor 20 numere se mparte exact la 4, ceea ce este fals.

Clasa a V-a
V.158. Determinat i restul mp art irii num arului 201220132014 la 36 (f ar a a efectua mp art irea). Iulian Oleniuc, elev, Ia si Solut ie. Num arul N este divizibil cu 9 (suma cifrelor nd 18), cu 2, dar nu cu 4. Cum (2, 9) = 1, rezult a c a N este divizibil cu 18, dar nu cu 36. Prin urmare, restul mp art irii lui N la 36 este 18. V.159. Determinat i numerele xyz cu proprietatea c a (x + y + z )3 = xyz . Nicolae Iv a schescu, Craiova Solut ie. Cuburile perfecte de trei cifre sunt 125, 216, 343, 512 si 729. Calcul and pentru ecare dintre ele cubul sumei cifrelor, obt inem singura solut ie (5+1+2)3 = 512. V.160. Determinat i ultimele dou a cifre ale num arului A = 30 +31 +32 +. . .+32013 . Mirela Marin, Ia si Solut ie. Avem: A = 1+3+32 (1+3+32 +33 )+36 (1+3+32 +33 )+ . . . +32010 (1+ 3+32 +33 ) = 4+9 40 (1+34 + . . . +32008 ). Cei 503 termeni din parantez a se termin a n 1, prin urmare suma lor se termin a n 3. Atunci A = 4+360 . . . 3 = 4+ . . . 80 = . . . 84, adic a ultimele dou a cifre ale lui A sunt 84. V.161. Se consider a numerele rat ionale (nenegative) a, b, c, d si e cu proprietatea c a a2 + b2 + c2 + d2 + e2 = 4. Demonstrat i c a a3 + b3 + c3 + d3 + e3 8. Mihai Cr aciun, Pa scani 2 3 2 Solut ie. Evident c a a 4, deci a 2 si atunci a 2a . Analog se arat a c a b3 2b2 , c3 2c2 , d3 2d2 si e3 2e2 . Prin adunarea acestor relat ii, obt inem c a a3 + . . . + e3 2(a2 + . . . + e2 ) = 2 4 = 8. Egalitatea se atinge atunci c and unul dintre cele cinci numere este 2, iar celelalte sunt 0. 138

V.162. Vom spune c a un num ar este istet dac a cifra unit a tilor sale este egal a cu suma cuburilor celorlaltor cifre. Dac a ordon am cresc ator numerele istet e, determinat i al cincisprezecelea termen al sirului obt inut. Silviu Boga, Ia si Solut ie. Numere istet e de dou a cifre sunt 11 si 28, cele de trei cifre sunt 101, 112, 129, 208 si 219, iar cele de patru cifre sunt 1001, 1012, 1029, 1102, 1209, 2008, 2019 si 2109. Rezult a c a al cincisprezecelea num ar istet este 2109. V.163. Putem a seza pe un cerc numerele 1, 2, 3, . . . , n (n N, n 3) astfel nc at suma oric aror trei numere al aturate s a e num ar impar? Gheorghe Iurea, Ia si Solut ie. R aspunsul este negativ; pentru a demonstra acest lucru, s a presupunem contrariul. Evident, nu este posibil ca orice dou a numere al aturate s a aib a parit a ti diferite ( ntr-o secvent a de tipul i, p, i, suma este par a). Dac a exist a dou a numere al aturate impare, ar trebui ca toate s a e impare, imposibil. Dac a exist a dou a numere al aturate pare, a s ezarea numerelor va de tipul p, p, i, p, p, i, p, p, i, . . . ; vom avea pe DnE DnE DnE DnE numere impare si n numere pare si, cum < n , ajungem la cerc 3 3 3 3 o contradict ie si n aceast a situat ie. V.164. Scriem toate numerele naturale de trei cifre pe c ate un cartona s si introducem cele 900 de cartona se ntr-o cutie. Care este num arul minim de cartona se pe care trebuie s a le extragem, f ar a a ne uita la ele, pentru a siguri c a vom obt ine cel put in sapte numere cu aceea si sum a a cifrelor? Sergiu Prisacariu, Ia si Solut ie. Sumele posibile ale cifrelor numerelor de pe cartona se sunt 1, 2, 3, . . . , 27. Exist a un singur cartona s cu suma cifrelor 1 (anume 100) si doar unul cu suma cifrelor 27 (anume 999). Avem n cutie c ate trei cartona se cu suma cifrelor 2 (anume 101, 110 si 200) sau 26 (anume 899, 989 si 998). Exist a c ate sase cartona se cu suma cifrelor 3 (anume 111, 102, 120, 201, 210 si 300) sau 25 (anume 799, 979, 997, 889, 898 si 988). Celelalte sume ale cifrelor (4, 5, . . . , 24) apar ecare pe mai mult de sapte cartona se. In concluzie, num arul minim de cartona se extrase trebuie s a e 2 1+2 3+2 6+7 = 27.

Clasa a VI-a
VI.158. Determinat i perechile de numere ntregi (x, y ) care veric a simultan condit iile x + y = 6 si 2xy 3x 3y + 2 = 0. Eugeniu Bl ajut , Bac au 3x 2 Solut ie. Din a doua relat ie rezult a c a y = . Cum y Z, se impune 2x 3 condit ia 2x 3|3x 2 si atunci 2x 3|2(3x 2) 3(2x 3), adic a 2x 3|5, prin urmare x {1, 1, 2, 4}. A doua relat ie este vericat a de perechile (1, 1), (1, 1), (2, 4) si (4, 2) si, dintre ecestea, doar ultimele dou a veric a si prima condit ie. In concluzie, S = {(2, 4); (4, 2)}. VI.159. Determinat i numerele ntregi a, b, c, d, dac a 3a + 3b + 3c + 3d = 10. Vasile Chiriac, Bac au Solut ie. Pentru nceput, s a presupunem c a a b c d. Dac a a 2, nu exist a b, c, d Z pentru care suma 3a + 3b + 3c + 3d s a e num ar ntreg, iar dac a 139

a = 1, trebuie n mod necesar s a avem b = c = 1, altfel 3a + 3b + 3c + 3d Q\Z. Deducem c a singura solut ie av and componente negative ( n ipoteza asumat a) este a = b = c = 1, d = 2. Dac a a 0, atunci 3a + 3b + 3c 1 + 1 + 1 = 3, deci 3d 7; rezult a c ad=1 si, apoi, se arat a u sor c a a = 0, b = c = d = 1. In concluzie, solut iile ecuat iei date sunt (1, 1, 1, 2), (0, 1, 1, 1) si nc a sase care se obt in din acestea prin permut ari circulare. VI.160. Imp art ind 2013 la a, obt inem c atul b si restul c. Determinat i numerele naturale a, b si c, stiind c a reprezint a lungimile laturilor unui triunghi isoscel. Ioana Maria Popa, elev a, Ia si Solut ie. Avem c a 2013 = a b + c, cu c < a; din ipoteza problemei, rezult a c a a = b sau b = c. In primul caz, 2013 = a2 + c, cu a2 < a2 + c < a(a + 1). Cum 442 = 1936, 44 45 = 1980 < 2013 si 452 = 2025 > 2013, aceast a situat ie nu conduce la nicio solut ie. In al doilea caz, 2013 = c(a + 1), cu c < a; obt inem c a (a, c) {(2012, 1); (670, 3); (182, 11); (60, 33)}. Se veric a inegalitatea triunghiului doar dac a a = 60, b = c = 33. VI.161. Determinat i valorile naturale ale lui n pentru care exist a num arul natural x astfel nc at (2x + 9, 3x + 1) = 5n . Gheorghe Iacob, Pa scani Solut ie. Dac a 5n |2x + 9 si 5n |3x + 1, atunci 5n |3(2x + 9) 2(3x + 1), adic a 5n |25; rezult a c a n {0, 1, 2}. Dac a x = 0, avem c a (2x + 9, 3x + 1) = (9, 1) = 1 = 50 . Dac a x = 3, atunci (2x + 9, 3x + 1) = (15, 10) = 51 . Pentru x = 8, avem c a (2x + 9, 3x + 1) = (25, 25) = 52 . In concluzie, toate cele trei valori posibile ale lui n sunt convenabile. VI.162. Un calculator defect mai face doar patru operat ii: poate nmult i un num ar cu 2 sau cu 5 sau poate mp art i un num ar la 2 sau la 5, dac a mp art irea se efectueaz a exact. Un copil pleac a de la num arul 20 si face aleator 2013 astfel de operat ii. Este posibil ca rezultatul nal s a e tot 20? Petre B atr anet u, Galat i Solut ie. Observ am c a 20 = 22 5, cu suma exponent ilor num ar impar. La ecare operat ie, suma exponent ilor se m are ste sau se mic soreaz a cu 1, deci si schimb a paritatea. Dup a un num ar impar de operat ii (2013), suma exponent ilor va par a, prin urmare rezultatul nal nu poate tot 20. VI.163. Fie M si N mijloacele laturilor AB, respectiv AC ale triunghiului ABC. Cercul de diametru AB intersecteaz a dreapta M N n punctele D si E . Demonstrat i c a BD si BE sunt bisectoarele (interioar a si exterioar a) ale unghiului ABC . Ioan S ac aleanu, H arl au Solut ie. Presupunem, ca n gur a, c a D IntABC . Triunghiul M BD este A isoscel, ntruc at M B si M D sunt raze ale cercului; rezult a c a M BD M DB . Pe de alt a parte, cum M N BC, avem c a M DB DBC (alterne interne). E M N D Deducem c a M BD DBC , adic a BD este bisectoarea interioar a a unghiului ABC . Analog se arat a c a BE B C este bisectoarea exterioar a a lui ABC . 140

VI.164. Pe laturile AC si AB ale triunghiului ABC se consider a punctele E , respectiv F astE F fel nc at m(ABE ) < m(CBE ) si m(ACF ) < H m(BCF ). Fie D un punct pe latura BC si G K (AB ), H (AC ) pentru care DG BE si DH H CF. Demonstrat i c a AG + AH > AB + AC BC. G L G Petru Asaftei, Ia si Solut ie. Not am {L} = DG BE, {K } = B C D DH CF si e G , H simetricele punctelor G si H fat a de L, respectiv fat a de K . Cum m(G BE ) = m(ABE ) < m(CBE ), rezult a c a G (DL) si atunci BG = BG < BD; analog se arat a c a CH < CD. Astfel, AG + AH = AB + AC (BG + CH ) > AB + AC (BD + DC ) = AB + AC BC.

Clasa a VII-a
VII.158. Determinat i numerele naturale m cu proprietatea c a m(m +17) se poate scrie ca produs de dou a numere naturale consecutive. Lucian Tut escu si Petri sor Roc soreanu, Craiova Solut ie. Fie n N cu proprietatea c a m(m + 17) = n(n + 1); atunci 4m2 + 68m + 289 288 = 4n2 + 4n + 1, deci (2m + 17)2 (2n + 1)2 = 288. Obt inem c a (m + n + 9)(m n + 8) = 72 si, cum m n + 8 < m + n + 9 si cele dou a numere sunt naturale, de parit a ti diferite, avem de studiat trei posibilit a ti: (m n + 8, m + n + 9) {(1, 72); (3, 24); (8, 9)}. Solut iile problemei sunt m {0, 5, 28}. VII.159. Dac nc at m2 +n2 +p2 = 3, demonstrat i a m, n, p sunt astfel F numere reale 2m2 + 5 + 2n2 + 5 + 2p2 + 5 3 7. Mihai Cr aciun, Pa scani F Solut ie. Not am x = 2m2 + 5, y = 2n2 + 5 si z = 2p2 + 5; atunci x2 + y 2 + z 2 = 2(m2 + n2 + p2 ) + 15 = 21. Folosind inegalitatea dintre media aritmetic a si cea p atratic a, obt inem c a (x + y + z )2 3(x2 + y 2 + z 2 ) = 63, deci x + y + z 3 7. Egalitatea se atinge pentru m = n = p = 1. VII.160. Determinat i triunghiurile dreptunghice cu lungimile laturilor exprimate prin numere naturale, care au aria egal a cu 24. Neculai Stanciu, Buz au Solut ie. Dac a laturile a, b, c ale unui triunghi dreptunghic sunt numere naturale, atunci a = x2 + y 2 , b = x2 y 2 si c = 2xy , unde x, y N , x > y. Aria triunghiului bc este A = = xy (x y )(x + y ) > y y 1 y , prin urmare y 3 < 24, deci y {1, 2}. 2 Dac a y = 1, atunci x(x + 1)(x 1) = 24, de unde x = 3. Dac a y = 2, obt inem c a 2x(x 2)(x + 2) = 24, f ar a solut ii n numere naturale. In concluzie, x = 3 si y = 1, a sadar triunghiul c autat are laturile a = 10, b = 8, c = 6. VII.161. Se consider a trapezul ABCD cu baza mare AB egal a cu diagonala AC . Fie {E } = AD BC, {O} = AC BD, iar P (AD) este astfel nc at OP AB. Demonstrat i c a CP si CE sunt bisectoarele interioar a, respectiv exterioar a ale unghiului ACD. Claudiu-S tefan Popa, Ia si c a 141

DP DO Solut ie. Din asem an ari evidente avem c a = = PA OB DP DC obt inem c a = deci, conform reciproPA AC cei teoremei bisectoarei aplicat a n triunghiul ACD, rezult a c a CP este bisectoarea (interioar a) unghiului ACD. T in and seama de faptul c a triunghiul ABC este isoscel, avem: P m(ABC ) + m(BCD) = 180 m(ABC ) + m(ACB ) + m(ACD) = 180 2m(ACB ) + 2m(ACP ) = 180 m(ACB ) + m(ACP ) = 90 m(BCP ) = 90 , prin urmare CP CE. A De aici rezult a c a CE este bisectoarea exterioar a a unghiului

DC . Cum AB = AC, AB E

D O

BCD.

VII.162. Demonstrat i c a diagonala unui trapez isoscel este mai lung a dec at linia mijlocie a acestuia. Nicolae Bourb acut , Sarmizegetusa Solut ia 1. Fie P proiect ia v arfului C pe baza mare AB ; n triunghiul dreptunghic P AC , ipotenuza AC este mai lung a dec at D C cateta AP . Pe de alt a parte, P N este median a n O triunghiul dreptunghic P BC , a sadar P N = BN , de M N >. Atunci AM P N unde N P B B A si, cum AP M N, rezult a c a AP N M este paralelogram. Deducem c a M N = AP si, de aici, urmeaz a concluzia A B problemei. P Solut ia 2 (Adina Onofrei, elev a, Ro siori (Bac au)). S tim c a ntr-un triunghi mediana este mai mic a dec at semisuma laturilor care pleaca din acela si v arf. Folosind si inegalitatea triunghului, avem: M N < OM + ON < OD + OA OC + OB AC + BD + = = AC. 2 2 2

VII.163. Fie D un punct pe latura BC a triunghiului ABC. Paralela prin D la AC taie AB n E , iar paralela prin D la AB taie AC n F . Dac a {M } = CE BF, ar atat i c a suprafet ele AEM F si BM C sunt echivalente. Dan Popescu, Suceava Solut ie. Not am {N } = AD CE, {P } = AD BF. In trapezele ABDF si ACDE A au loc egalit a tile ABP D = AAP F , respectiv ADN C = AAN E . Adun and aceste egalit a ti si reduc and din ambii membri AM N P , obt inem c a F ABM N D +ADN C = AAP F +AAEM P , deci ABM C = P AAEM F . Remarc am c a aria triunghiului M N P M E poate si nul a, n cazul n care D este mijlocul N lui BC . Not a. Rezultatul constituie o generalizare a B D C

.. .

142

problemei 26567 din Gazeta Matematic a 2/2012, n care D este piciorul bisectoarei din A. VII.164. Fie ABCD un patrulater convex, ale c arui diagonale se intersecteaz a n O. Dac a oricare ar punctele E (AB si F (CD cu AE = CF , avem c a O EF , demonstrat i c a ABCD este paralelogram. Gheorghe Iurea, Ia si Solut ie. Presupunem, prin absurd, c a AB CD si e B BC pentru care C AB CD. Consider am punctele E si F ca n enunt , iar F F D {M } = EF AB . Din AOM COF (U.L.U.) rezult a c a AM = CF si, cum CF = AE , triunghiul AM E este isoscel. Proced am analog pentru alte dou a O puncte E , F ca n enunt si obt inem c a AM E este 1 B isoscel. Astfel, m(AM E ) = m(AM E ) = (180 M M 2 m(BAB )), prin urmare M E M E , ceea ce contrazice A E E B faptul c a M E M E = {O}. R am ane c a AB CD. Pentru o pereche de puncte E si F ca n enunt , avem c a AECF este paralelogram (AE = CF, AE CF ), deci O va mijlocul diagonalei AC a patrulaterului ABCD cu AB CD. De aici, deducem imediat c a ABCD este paralelogram.

Clasa a VIII-a
VIII.158. Rezolvat i n R ecuat ia 27x2 + 54x + 15 147x2 126x + 24 = . 98x2 84x + 10 18x2 + 36x + 16 Constantin Dragomir, Pite sti

2 Solut ie. Inmult ind cu ambii membri ai ecuat iei si apoi sc az and 1 din ambii 3 2 2 40x 60x 40x + 60x = . De aici, membri ai ecuat iei obt inute, deducem c a 2 49x 42x + 5 9x2 + 18x + 8 3 40x2 60x = 0 cu solut iile x1 = 0 si x2 = sau 49x2 42x + 5 = 9x2 18x 8, 2 adic a 58x2 24x + 10 = 0, ecuat ie care nu are solut ii reale. Observ and c a valorile lui 3 . x obt inute nu anuleaz a numitorii fract iilor, rezult a c a S = 0, 2 1 VIII.159. Demonstrat i c a 1 + 3x + 6x2 + 9x3 + 9x4 > , x R. 4 Ionel Tudor, C alug areni Solut ia 1. Membrul st ang se descompune n factori ca (1 + 3x2 )(1 + 3x + 3x2 ). 1 (cu Observ am c a 1 + 3x2 1 (cu egalitate pentru x = 0), iar 1 + 3x + 3x2 4 1 egalitate c and x = ). Inmult ind membru cu membru aceste inegalit a ti si tin and 2 seama de faptul c a egalit a tile nu se ating simultan, obt inem cerint a problemei. Solut ia 2 (Adina Onofrei, elev a, Ro siori (Bac au)). Avem succesiv: 36x4 + Q 1 + 36x3 +24x2 +12x+4 > 1 12x4 +12x3 +4x2 +4x2 +4x+1 > 0 12x2 x2 + x + 3 143

Q

(2x + 1) > 0 12x


2

1 x+ 2

1 + + (2x + 1)2 > 0, ultima ind evident a. 12

VIII.160. Se consider a dreapta x a d, punctul x A /d si planul variabil care cont ine dreapta d. Not am cu M proiect ia punctului A pe planul . Determinat i locul geometric al lui M . Aida-Andreea Iacob, Ia si Solut ie. Fie planul care cont ine punctul A si este perpendicular pe dreapta d si {B } = d ; evident, B este un punct x. Dreapta AM este inclus a n planul , iar triunghiul ABM este dreptunghic n M . Rezult a c a M apart ine cercului C de diametru AB , inclus n planul . Reciproc, se observ a u sor c a, pentru orice punct M al lui C , exist a un plan = (d, M B ) astfel nc at M este proiect ia lui A pe . In concluzie, locul geometric dorit este cercul C . VIII.161. Se dau zece cutii cubice cu muchiile de 1 cm, 2 cm, . . ., 10 cm, ecare ind umplut a cu cuburi de muchie 1 cm. Spunem c a p cutii formeaz a un bicub dac a una dintre ele cont ine tot at atea cuburi c ate cont in celelalte p 1 la un loc. Putem forma un bicub folosind c ateva dintre cutiile date? Geanina H av arneanu, Ia si Solut ie. R aspunsul este armativ: 13 + 33 + 43 + 53 + 63 = 73 . Mai mult, o analiz a (plicticoas a!) arat a c a aceasta este singura solut ie a problemei. VIII.162. Determinat i ultimele trei cifre ale num arului natural A = 3733 +3743 + 5 3 375 + . . . + 628 . Mihai Haivas, Ia si Solut ie. Dac a a, b Z, atunci a3 + b3 = M (a + b). Astfel, A = (3733 + 6273 ) + (3743 + 6263 ) + . . . + (4993 + 5013 ) + 5003 + 6283 = M1000 + 6283 = M1000 + . . . 172 = . . . 172. 4m + 3 VIII.163. Determinat i valorile ntregi ale lui m pentru care num arul m5 este rat ional. Bogdan Chiriac, Bac au 4m + 3 4 m + 3 a Solut ie. Fie = , cu a, b N , (a, b) = 1 (evident c a m5 b m5 5a2 3b2 23b2 este strict pozitiv pentru m Z). Obt inem c a m = = 5 2 . 4b2 a2 4b a2 2 2 2 2 2 2 2 Cum (4b a , b ) = (a , b ) = 1, rezult a c a 4b a divide 23, deci (2b a)(2b + a) {1, 23}. Dintre sistemele care se pot forma, singurul cu solut ii naturale este 2b a = 1, 2b + a = 23, prin urmare a = 11, b = 6, adic a m = 31. VIII.164. Fie a, b, c, d patru numere reale strict pozitive astfel nc at ab(c + d) (a + b)cd si ab + cd (a + b)(c + d). Comparat i numerele a + b si c + d. Dan Nedeianu, Drobeta Tr. Severin Solut ie. Vom demonstra c a a + b > c + d; pentru aceasta, s a presupunem, prin absurd, c a a + b c + d. Inmult ind membru cu membru inegalit a tile din enunt , obt inem c a ab(c + d)(ab + cd) (a + b)2 cd (c + d). Ins a (a + b)2 4ab si atunci ab(c + d)(ab + cd) 4abcd(c + d), deci ab + cd 4cd, a sadar ab 3cd. Pe de alt a 144

parte, ab + cd (a + b)(c + d) (a + b)2 4ab, de unde cd 3ab. Ar rezulta c a cd 3ab 9cd, imposibil; r am ane astfel adev arat a armat ia init ial a.

Clasa a IX-a
IX.136. Fie a, b, c (0, ) cu a b c = 1. Demonstrat i c a ab bc ca 9 + 2 + 2 3 . b2 + c2 c + a2 a + b2 a + b3 + c3 + a + b + c Sven Cortel si Kinga Rat iu, elevi, Satu Mare Solut ie. Conform inegalit a t ii lui Bergstr o m, membrul st ang al inegalit a tii din ( ab + bc + ca)2 . Din inegalitatea mediilor, avem c a enunt este cel put in egal cu 2(a2 + b2 + c2 ) 3 ab + bc + ca 3 abc = 3, iar 2a2 a + a3 , 2b2 b + b3 , 2c2 c + c3 , de unde rezult a cerint a problemei. Egalitatea se atinge pentru a = b = c = 1. IX.137. Fie ABCD un patrulater inscriptibil. Not am cu H1 , H2 , H3 si H4 ortocentrele triunghiurilor DAB, ABC , BCD respectiv CDA si cu G1 , G2 , G3 si G4 centrele de greutate ale triunghiurilor AH1 B, BH2 C , CH3 D respectiv DH4 A. Demonstrat i c a patrulaterele ABCD si G1 G2 G3 G4 au acela si centru de greutate dac a si numai dac a ABCD este dreptunghi. Florin St anescu, G ae sti Solut ie. In raport cu centrul O al cercului circumscris patrulaterului ABCD, = + + = vectorii de pozit ie ai punctelor din problem a sunt: rH r r r B, rH D A 1 2 + + , = + + , = + + ; = 1 (2 + 2 + ), r r r r r r r r r r r r r r r A B C H3 B C D H4 C D A G1 A B D 3 1 1 1 rG2 = (2rB +2rC + rA ), rG3 = (2rC +2rD + rB ), rG4 = (2rD +2rA + rC ). Centrul 3 3 3 1 + de greutate al patrulaterului ABCD are vectorul de pozit ie r 1 = ( rA + r B rC + rD ), 4 iar centrul de greutate al patrulaterului G1 G2 G3 G4 are vectorul de pozit ie r2 = 5 (rA + rB + rC + rD ). Evident c a r 1 = r2 rA + rB + rC + rD = 0 ABCD 12 este dreptunghi; propunem cititorului s a demonstreze riguros ultima echivalent a. IX.138. Fie M un punct variabil n interiorul sau pe laturile triunghiului ascu titunghic ABC. Ar atat i c a M B 2 + M C 2 M A2 AB 2 + AC 2 . Ovidiu Pop, Satu Mare Solut ie. Fie D simetricul punctului A fat a de mijlocul O al segmentului BC . A Folosind teorema medianei n triunghiurile M BC si 2 2 2 2( M B + M C ) BC M AD, obt inem c a M O2 = = 4 2(M A2 + M D2 ) AD2 M , de unde M B 2 + M C 2 4 2 2 C B BC AD O M A2 = M D 2 + . Cum unghiul A este 2 ascut it, cea mai mare distant a dintre dou a puncte situate pe laturile sau n interiorul paralelogramului ABDC
D

145

este diagonala AD. Rezult a c a M D2 AD2 = 4AO2 = 2(AB 2 + AC 2 ) BC 2 . AstBC 2 AD2 BC 2 + AD2 fel, M B 2 + M C 2 M A2 AD2 + = = AB 2 + AC 2 , ceea 2 2 ce trebuia demonstrat. >) = 90 IX.139. Se consider a triunghiul dreptunghic ABC cu m(A si m(C ) = 30 . Fie O si F picioarele medianei, respectiv n alt imii din A, iar E este un punct astfel nc at F (AE ), F E = F B. Not am cu M simetricul lui A fat a de E si cu D intersect ia dreptelor M O si AC . Demonstrat i c a CO = CD. Eugeniu Bl ajut , Bac au Solut ia 1. Fie AB = 2a; se observ a imediat c a OF = F B = F E = a , AF = a 3, M E = a(1 + 3), F M = a(2 + 3). In triunghiul C MF = dreptunghic F OM, avem c a tg M OF = OF M a(2 + 3) = 2 + 3, prin urmare m(M OF ) = 75 . N a O Rezult a c a m(COD) = 75 , de unde m(CDO) = 180 30 75 = 75 . Astfel, triunghiul COD este D E isoscel, cu CO = CD. F Solut ia 2 (a autorului). Not am cu N mijlocul segmentului OM ; atunci EN este linie mijlocie n 1 B AOM, deci N E = OA = a si N E AO. De- A 2 ducem c a m(N EF ) = 180 m(OAF ) = 150 , iar triunghiul EF N este isoscel, cu EF = N E = a. Obt inem c a m(N F E ) = 15 , a sadar m(OF N ) = 75 . Pe de alt a parte, N F este mediana ipotenuzei n F OM, prin urmare N F = N O, de unde m(N OF ) = 75 si solut ia continu a ca mai sus. IX.140. Determinat i numerele naturale nenule n pentru care (x + y )n (xn + y n ) = nxy (x + y )(x2 + xy + y 2 )
n 3 2

, x, y R.

Ionel Tudor, C alug areni n 3 Solut ie. Pentru x = y = 1, obt inem c a 2n 2 = 2n 3 2 . Aceast a egalitate este fals a dac a n este par, ntruc at membrul st ang este num ar rat ional, iar ce drept este irat ional. Fie deci n = 2k + 1, k N, astfel nc at 22k+1 2 = 2(2k + 1)3k1 , adic a 4k = 1 + (2k + 1)3k1 ; aceast a egalitate se veric a pentru k {1, 2, 3} si nu este adev arat a pentru k = 0 si k 4, ntruc at se dovede ste u sor, prin induct ie matematic a, inegalitatea 4k > 1 + (2k + 1)3k1 , k 4. Inseamn a c a singurele valori ale lui n care ar putea solut ii ale problemei sunt n {3, 5, 7}. Identit a tile: (x + y )3 (x3 + y 3 ) = 3xy (x + y ); (x + y )5 (x5 + y 5 ) = 5xy (x + y )(x2 + xy + y 2 ); (x + y )7 (x7 + y 7 ) = 7xy (x + y )(x2 + xy + y 2 )2 ind adev arate pentru orice x, y R, rezult a c a valorile c autate ale lui n sunt 3, 5 si 7. 146

Clasa a X-a
X.136. Fie numerele naturale nenule a1 , a2 , . . ., an , n2. Determinat i numerele n n 2 2 ai ai |=2n 2. complexe z1 , z2 , . . ., zn , de modul 1, cu proprietatea c a |1 zi | + |1+ zi
i=1 i=1

Sven Cortel, elev, Satu Mare Solut ie. Fie a N si z = cos 2t + i sin 2t un num ar complex de modul 1; atunci 2 a 2 |1 z | = (1 cos 2at) + sin 2at = 2 2 cos 2at = 2| sin at| si, analog, |1 + z a | = H 2 sin at + cos2 at 2| cos at|, deci |1 z a | + |1 + z a | = 2(| sin at| + | cos at|) 4 = 2 2. 2 k Egalitatea se atinge c and sin at = cos at, adic a pentru t = + , k Z sau 4a a 3 k t= + , k Z. 4a a n 2 ai Din cele de mai sus rezult a c a, n general, are loc inegalitatea |1 zi |+ (4k + 1) 2n 2. Intruc at se atinge egalitatea, obt inem c a zi = cos + 2ai i=1 (4k + 1) (4k + 3) (4k + 3) i sin , k Z sau zi = cos + i sin , k Z, oricare ar 2ai 2ai 2ai i {1, 2, . . . , n}. |1 +
ai zi | n 2

i=1

X.137. Fie a, b N. Rezolvat i n C ecuat ia |z + a| + |z b| + |a b| = 2. Gheorghe Iurea, Ia si Solut ie. Dac a z C veric a relat ia din enunt , atunci 2 = |z + a| + |z b| + |a b| |z + a z + b| + |a b| = |a + b| + |a b|. Din a, b N si |a + b| + |a b| 2, rezult a c a (a, b) {(0, 0); (1, 0); (0, 1); (1, 1)}. Dac a a = b = 0, ecuat ia |z | = 1 admite solut iile z = cos t + i sin t, t R. Dac a a = 1, b = 0, ecuat ia devine |z + 1| + |z | = 1, cu mult imea solut iilor S = [1, 0]. Dac a a = 0, b = 1, ecuat ia devine |z | + |z 1| = 1, cu mult imea solut iilor S = [0, 1]. In sf ar sit, dac a a = b = 1, ecuat ia |z + 1| + |z 1| = 2 are solut iile S = [1, 1]. Pentru alte valori ale numerelor a si b, ecuat ia nu are solut ii. X.138. Rezolvat i ecuat ia
4x log2 + 22x ) + 24x + 22x = 4 + log5 (28x + 23x+3 + 242x ). 5 (2

Dan Nedeianu, Drobeta Tr. Severin Solut ie. Deoarece (24x + 22x )2 = 28x + 23x+3 + 242x , ecuat ia se poate scrie sub forma [log5 (24x + 22x ) 1]2 = 5 (24x + 22x ) si atunci 24x + 22x 5. Ins a, din 1 1 1 1 inegalitatea mediilor, avem c a 24x + 22x = 24x + x + x + x + x 5, cu egalitate 2 2 2 2 c and 24x = 2x , i.e. x = 0. Se veric a faptul c a x = 0 este solut ie a ecuat iei, prin urmare S = {0}. X.139. i ecuat iile: Rezolvat n a) [ n n + n2 ] = n; n 1 b) [ n1 n + n2 ] = n. Ionel Tudor, C alug areni 147

Solut ie. a) Pentru a avea sens radicalii, se impune ca n N, n 2. Prin induct ie 3 n 3 n matemtic a se arat a c a 3 n , n N , de unde n 3, n N , n 2 . Rezult a n n a ecuat ia c a n2 3 9, prin urmare n n + n2 3 3 + 3 9 < 2 + 3 = 5. Deducem c dat a nu are solut ii n 5. Veric and direct valorile n {2, 3, 4}, g asim c a unica solut ie a ecuat iei este n = 3. b) Se impune condit ia n N, n 3. Se veric a faptul c an=3 si n = 5 nu veric a ecuat ia, iar n = 4 este solut ie; vom ar ata c a nu exist a solut ii n 6. Prin induct ie n1 3 n 1 matematic a se demonstreaz a c a3 > n , n 6, deci n < 3 3, n N, n 6. n 1 n 1 Atunci n2 < 3 9, a sadar n1 n + n2 < 3 3 + 3 9 < 2 + 3 < 6. R am ane c a ecuat ia dat a are unica solut ie n = 4. X.140. Dac a n triunghiul ABC , cu notat iile uzuale, are loc egalitatea a2 + b2 + 2 2 2 c = p + 9r , atunci triunghiul este echilateral. C at alin Calistru, Ia si S2 2 3 2 Solut ie. Observ am c a pr = = (p a)(p b)(p c) = p (a + b + c)p + (ab + p bc + ca)p abc = p3 + (ab + bc + ca)p 4Rrp, de unde ab + bc + ca = p2 + r2 + 4Rr. Pe de alt a parte, ab + bc + ca a2 + b2 + c2 = p2 + 9r2 si atunci p2 + r2 + 4Rr p2 + 9r2 , prin urmare R 2r. Rezult a c a se atinge egalitatea n inegalitatea lui Euler R 2r, adic a triunghiul este echilateral.

Clasa a XI-a
XI.136. Fie A, B M2 (R) astfel nc at det A = 9, det B = 4 si det (A + B ) = 1. Determinat i x R pentru care num arul det(A + xB ) este minim. R azvan Ceuc a, student, Ia si Solut ie. Fie f (x) = det(A + xB ) = det A + mx + (det B )x2 = 4x2 + mx + 9. Cum f (1) = det(A + B ) = 1, obt inem c a 4 + m + 9 = 1, deci m = 12. Atunci 3 2 2 f (x) = 4x 12x + 9 = (2x 3) 0, valoarea minim a 0 ating andu-se pentru x = . 2 XI.137. Fie a (0, 1) [2 , ) si sirul (xn )n1 denit prin x1 = a, xn+1 = 2a xn+1 a + xn , n 1. Not am yn = , n N . a xn 2 a) Demonstrat i c a yn > , n N . 2 2 a + 1 + 1 + 4a 1 b) Dac a a (0, 1), ar atat i c a, n plus, yn < , n N . 2a + a Gheorghe Costovici, si Ia 1 + 1 + 4a Solut ie. a) Se demonstreaz a, prin induct ie matematic a, faptul c a xn < , 2 2a a + xn a xn 1 n N . Atunci yn = = = > a xn (a xn )( 2a + a + xn ) 2a + xn+1 1 2 = , n N . 1+ 1+4a 2 2 a + 1 + 1 + 4 a 2a + 2 b) Se demonstreaz a prin induct ie c a, pentru a (0, 1), xn > a, n N . Atunci 1 1 < , n N . yn = 2a + xn+1 2a + a 148

XI.138. Fie (xn )n1 un sir de numere reale monoton si convergent. Ar atat i c a sirul (yn )n1 , yn =
n 2

(xk+1 xk )2013 are aceea si monotonie cu (xn )n1 si este, de

asemenea, convergent.

k=1

Silviu Boga, Ia si Solut ie. Cum yn+1 yn = (xn+2 xn+1 )2013 , diferent a xn+2 xn+1 are semn constant pentru n N si 2013 este num ar impar, rezult a c a (yn )n1 are aceea si monotonie cu (xn )n1 . Datorit a convergent ei lui (xn )n1 , sirul zn = (xn+1 xn )2012 este m arginit superior de M ; atunci |yn | =
n 2

|xk+1 xk |zk

n 2

|xk+1 xk |M =

|xn+1 xn |M (am tinut seama de faptul c a toate diferent ele xk+1 xk au acela si semn). Deoarece (xn )n1 este m arginit, rezult a c a (yn )n1 este m arginit, deci convergent. XI.139. Dac a p N, p 2, calculat i lim n e n+1 + n+2 +...+ pn p .
n

k=1

k=1

1 1 1 + + ... + = n + 1 n + 2 pn _Q Q b 1 1 1 1 lim 1 + + ... + ln pn 1 + + . . . + ln n + ln pn ln n = c n 2 pn 2 n c + ln p = ln p, a sadar avem de-a face cu o nedeterminare de tipul 0. Not and xn = 1 1 1 xn +ln p + +. . .+ ln p, limita din enunt devine: L = lim n(e eln p ) = n n+1 n+2 pn exn 1 xn xn+1 xn lim n p(exn 1) = lim np xn = lim p 1 = p lim 1 1 = p n n n n xn n n+1 n Solut ie. Observ am c a lim
p 2

Lucian Tut escu si Liviu Smarandache, Craiova Q

lim

i=1

1 np+i

p p(n+1)

p P 2

p 1 Q 2 i=1

n(n1 +1)
n

= p lim = p

i=1

1 np+i

1 np+p

n p 1 2 i=1

1 n(n+1)

= p lim

p 1 2

n i=1

(p i)n(n + 1) = (np + i)(np + p)

lim

n(p i) np2 + pi

pi p p(p 1) 1p = 2 = . p2 p 2 2

XI.140. Dac a A, B M2 (R), ar atat i c a 2(det A)2 + det (AB + BA) + 2(det B )2 2 2 det (A B ) + 4det AB. Mih aly Bencze, Bra sov Solut ie. Dac a U, V M2 (C), are loc egalitatea () det(U + V ) + det(U V ) = 2det U + 2det V.

Consider and n (*) U = A2 + B 2 si V = i(AB BA), unde i este unitatea imaginar a, obt inem c a 2det(A2 + B 2 ) 2det(AB BA) = det(A2 + iAB iBA + B 2 )+ + det(A2 iAB + iBA + B 2 ) = det(A iB )det(A + iB )+ + det(A + iB )det(A iB ) = 2det(A + iB )det(A + iB ) = = 2|det(A + iB )|2 0, 149

prin urmare det(A2 +B 2 ) det(AB BA). Ins a, conform (*), avem c a det(A2 +B 2 ) = 2 2 2 2 2det A +2det B det (A B ), iar det(AB BA) = 2det AB +2det BA det(AB + BA) = 4det AB det(AB + BA); nlocuind, obt inem concluzia problemei.

Clasa a XII-a
x2013 + ax1006 dx, x (0, ), a, b > 0. (x1007 + b)2013 C at alin Cristea, Craiova x2013 + ax1006 x1006 (x1007 + b) x1006 (a b) Solut ie. dx = dx + dx = (x1007 + b)2013 (x1007 + b)2013 (x1007 + b)2013 1 (x1007 + b) ab (x1007 + b) 1 1 dx+ dx = 1007 2012 1007 (x + b) 1007 (x1007 + b)2013 2011 1007 (x1007 + b)2011 ab 1 + C. 1007 2012 (x1007 + b)2012 D cos 2011x XII.137. Determinat i primitivele funct iei f : 0, R, f (x) = . 2 cos2013 x Liviu Smarandache si Lucian Tut escu, Craiova cos(2012x x) cos 2012x cos x Solut ie. Avem: f (x)dx = dx = dx + 2013 x cos cos2013 x Q sin 2012x cos 2012x 1 sin 2012x sin x dx = dx + dx = 2012 x cos2013 x cos2012 x cos 2012 Q cos 2012x cos 2012x sin 2012x 1 sin 2012x dx = dx + dx + cos2012 x 2012 cos2012 x cos2012 x 2012 cos2012 x sin 2012x cos 2012x sin 2012x dx = + C. 2012 cos2012 x cos2012 x 2012 cos2012 x XII.136. Calculat i XII.138. Ar atat i c a r ad acinile polinomului X 3 + aX 2 + bX + c sunt n progresie geometric a dac a si numai dac a a3 c = b3 . Temistocle B rsan, Ia si Solut ie. T in and cont de relat iile lui Vi` ete, avem: a3 c = b3 (x1 + x2 + x3 )3 3 3 3 3 3 3 3 3 x1 x2 x3 = (x1 x2 + x2 x3 + x3 x1 )3 x1 x2 x3 (x3 1 + x2 + x3 ) = x1 x2 + x2 x3 + x1 x3 2 2 2 2 3 3 3 2 2 (x1 x2 x3 )(x2 x3 x1 x3 x1 x2 + x1 x2 x3 ) = 0 (x1 x2 x3 )(x2 x1 x3 )(x3 x1 x2 ) = 2 2 a r ad acinile polinomului dat sunt 0 x2 1 = x2 x3 sau x2 = x1 x3 sau x3 = x1 x2 , adic n progresie geometric a. XII.139. Pe mult imea nevid a G se consider a operat ia asociativ a ,, n raport cu care are loc regula de simplicare la st anga si astfel nc at exist a a G cu axa = x3 , x G. Demonstrat i c a (G, ) este grup abelian. D.M. B atinet u-Giurgiu, Bucure sti si Neculai Stanciu, Buz au 3 Solut ie. Avem: a(ax)a = (ax) a(axa) = a x (axa) x axa = x (axa) x x3 = x x3 x x = x3 , x G. In particular, a3 = a si atunci a3 x = ax, x G, deci 2 a x = x, x G. Inlocuind pe x cu xa, deducem c a a2 xa = xa a (axa) = xa 3 2 ax = xa ax = xa, x G. Atunci a x = a (ax) = a (xa) = (ax)a = (xa)a = xa2 si, cum a2 x = x, x G, rezult a c a a2 x = xa2 = x, x G, adic a a2 este elementul neutru al operat iei ,,; not am e = a2 . 150

Am v azut c a x3 = x = xa2 , x G. Simplic and la st anga cu x, obt inem c a x = e, x G. Aceast a egalitate arat a c a orice x G este inversabil, cu x1 = x. In plus, xy = (xy )1 = y 1 x1 = yx, x, y G, a sadar (G, ) este grup abelian. XII.140. Fie a > 0 si cercurile de ecuat ii C1 : (x a)2 + y 2 = a2 si 2 2 2 C2 : (x + a) + y = a . Determinat i aria minim a a unei elipse care are drept axe de simetrie axele de coordonate si este tangent a exterior n c ate dou a puncte la ecare dintre cercurile date. Adrian Corduneanu, Ia si x2 y 2 Solut ie. Fie A(a, 0) centrul lui C1 , E : 2 + 2 = 1 ecuat ia unei elipse ca n enunt , y P (x0 , y0 ) punctul de tangent a situat n primul cadran, iar d tangenta comun a n P la cele dou a curbe. Panta 1 a x0 tangentei la C1 n P este md = = < mAP y0 0, iar panta tangentei la elips a n P se obt ine prin d 2y y 2x derivare, din 2 + = 0 , pentru x = x , deci 0 P 2 2 x0 este y (x0 ) = 2 . Egal and cele dou a pante, x Q y0 a O 2 2 A obt inem c a = 1 . Cum P E , avem c a x0 x2 y2 0 2 = a2 . Din aceste trei egalit a ti, + 0 = 1, iar din P C1 deducem c a (x0 a)2 + y0 2 2 3 /2 ax2 ax0 0 rezult a c a 2 = , iar 2 = ax0 . Aria elipsei va A = = . x0 a (x0 a)1/2 Minimul ariei se atinge odat a cu p atratul s au, deci avem de determinat minimul x3 , cu a < x < 2a. G asim imediat c a punctul de minim este funct iei f (x) = xa 3a 3 3 x0 = , iar valoarea minim a a ariei este Amin = a2 . 2 2
2

Solut iile problemelor pentru preg atirea concursurilor propuse n nr. 1/2013
A. Nivel gimnazial
G236. Determinat i numerele naturale a, b, c, d si e, strict mai mari ca 1, cu proprietatea c a a + b + c + d + e = abcde 95. Titu Zvonaru, Com ane sti Solut ie. Presupunem c a a b c d e 2; atunci 95 + 5a 95 + a + b + c + d + e = abcde a 2 2 2 2 = 16a, de unde a 8. Dac a a = 8, rezult a c a 103 + b + c + d + e = 8bcde si, proced and ca mai nainte, obt inem c a 103 + 4b 64b, deci 60b 103 si nu avem solut ii n acest caz. La fel se arat a c a nu avem solut ii c and a = 7. Dac a a = 6, deducem c a 101+b+c+d+e = 6bcde, deci 101 + 4b 48b, prin urmare b = 2. Rezult a c ac=d=e=2 si, nlocuind n 151

ecuat ia init ial a, obt inem o contradict ie. Analog, nu vor conveni nici valorile a = 5 si a = 4. Deoarece 95 + 5 2 = 25 , r am ane doar posibilitatea a = 3. Atunci, unul, dou a, trei, patru sau cinci dintre numerele c autate sunt egale cu 3, iar celelalte sunt 2. Prin veric ari directe, solut iile sunt (3, 3, 3, 2, 2) si permut arile acesteia. G237. Ar atat i c a exist a n numere naturale distincte a1 , a2 , . . . , an pentru care 2 2 suma a1 + a1 + . . . + an este p atrat perfect, iar a2 1 + a2 + . . . + an este cub perfect. Gheorghe Iurea, Ia si Solut ie. C aut am numere de forma ai = k i, i = 1, 2, . . . , n, unde k N ; atunci n(n + 1) 2 2 2 n(n + 1)(2n + 1) S = a1 + a2 + . . . + an = k , iar T = a2 . 1 + a2 + . . . + an = k 2 6 n(n + 1) 2 u , u N . In acest Pentru ca S s a e p atrat perfect, vom considera k = 2 3 3 n (n + 1) 2n + 1 fel, T devine egal cu u3 u si, aleg and u = 3(2n + 1)2 , vom avea T 8 3 9 cub perfect. In concluzie, putem considera ai = k i, unde k = n(n + 1)(2n + 1)4 . 2 G238. Se consider a numerele reale x, a1 , a2 , . . . , a100 . Dac a 51 dintre numerele a1 + a2 a1 + a2 + . . . + a100 a1 , ,..., sunt egale cu x, ar atat i c a m acar dou a dintre 2 100 numerele ai , i = 1, 100, sunt egale cu x. C at alin Budeanu, Ia si b1 + b2 Solut ie. Fie b1 = a1 x, b2 = a2 x, . . . , b100 = a100 x; atunci = 2 b1 + b2 + . . . + b100 a1 + a2 + . . . + a100 a1 + a2 x, . . . , = x. Din ipotez a rezult a 2 100 100 b1 + b2 b1 + b2 + . . . + b100 c a 51 dintre numerele b1 , ,..., sunt nule, prin urmare 51 2 100 dintre sumele s1 = b1 , s2 = b1 + b2 , . . . , s100 = b1 + b2 + . . . + b100 , n num ar de 100, sunt nule. M acar dou a perechi dintre cele 51 de sume nule au indici consecutivi: sm = sm+1 = 0 si sn = sn+1 = 0, cu m, n {1, 2, . . . , 99}, m = n. Atunci bm+1 = sm+1 sm = 0 si bn+1 = sn+1 sn = 0, prin urmare am+1 = an+1 = x. G239. Determinat i valorile num arului real k , dac a
3 a3 1 + . . . + a2013 + 4026 k (a1 + . . . + a2013 ), ai [2, ).

Lucian Tut escu, Craiova si Marian Voinea, Bucure sti Solut ie. Consider and a1 = . . . = a2013 = 1, obt inem k 3. Pentru a1 = . . . = a2013 = 2, g asim k 3 si, de aici, k = 3. Demonstr am n continuare c a valoarea k = 3 este convenabil a.Cum x3 3x + 2 = (x + 2)(x 1)2 0, x [2, +), rezult a c a x3 + 2 3x, x [2, ). D and lui x valorile a1 , a2 , . . . , a2013 si sum and relat iile 3 obt inute, deducem c a a3 1 + . . . + a2013 + 4026 3(a1 + . . . + a2013 ), ai [2, ). G240. Se consider a E (x1 , x2 , . . . , xn ) = R, n 16, cu
n 2 i=1 n 2

xi +2

i=1

1i<j n

xi xj , unde x1 , x2 , . . . , xn

x2 i valorile extreme ale acestei expresii. i = 1. Determinat Petru Asaftei, Ia si 152

Solut ie. Observ am c a E = 1 5 = 4 4


Q
n 2 i=1

n 2 i=1

xi +

n 2 i=1

xi

n 2 i=1

x2 i =

n 2 i=1

xi

n 2 i=1

xi +

xi +

1 2

x3 = x5 = . . . = x15 5 urmare Emin = . 4

5 5 1 . Pentru x1 = x2 = x4 = x6 = . . . = x16 = , 4 4 4 1 5 = , xk = 0 pentru k > 16, obt inem pentru E valoarea , prin 4 4
Q
n 2 i=1

Utiliz and inegalitatea (n 1) 1+2


2
i<j

x2 i

2
n 2 i=1

2
i<j

xi xj , deducem c a n si atunci E

n 2 i=1

xi

xi xj 1 + (n 1) 1 = n, deci

xi

n + (n 1).

1 Pentru x1 = x2 = . . . = xn = se atinge egalitatea, a sadar Emax = n + n 1. n G241. In triunghiul ABC se consider a cevienele concurente AA , BB si CC astfel nc at A B = A C, iar dreptele BC si B C se intersecteaz a n M . 2 1 1 a) Demonstrat i c a = . M A AB AC b) Determinat i lungimea segmentului M A n funct ie de laturile triunghiului, atunci c and AA este bisectoare, respectiv n alt ime. Neculai Roman, Mirce sti (Ia si) 2 Solut ie. a) Presupunem, ca n gur a, c a A B < A C ; atunci = M A 1 1 A B AC + M A A B = M A A A B A C A C A B A C A B M C = A C M B MB A B B = (). Aplic and teorema lui Menelaus MC AC n ABC cu transversala M C B , obt inem M B BC C A C c a = 1. Ins a, din teorema lui M C BA C B AB BC CA Ceva, = 1 si, de aici, rezult a c a AC BA CB AB MB B A C M = , adic a (*) este adev arat a. MC AC ac ab 2 b) Dac a AA este bisectoare, atunci BA = si A C = , deci = b+c b+c M A 2 2 abc b+c b+c |b c | , iar M A = 2 . = ac ab abc |b c2 | a2 + c2 b2 a2 + c2 b2 Dac a AA este n alt ime, atunci A B = c cos B = c = , 2ac 2a Q 1 a2 + b2 c2 1 1 iar A C = . Obt inem M A = a 2 . 2a a b2 + c2 a2 + b2 c2 Not a. In cazul n care triunghiul este ascut iunghic si cele trei ceviene sunt n alt imile sale, reg asim problema 25480 din Gazeta Matematic a 2/2006. 153

triunghiul ABC latura AB este x G242. In a, iar lungimea laturii AC este constant a. Fie AA bisectoarea unghiului BAC , cu A BC. Ar atat i c a dreapta perpendicular a n A pe AA trece printr-un punct x. Claudiu-S tefan Popa, Ia si Solut ie. Fie d perpendicular a n A pe AA , {D} = d AB, {E } = d AC, A A M AC, M AB si A N AB, N AC. Vom ar ata c a D este punctul x c autat; cum [AB ] este x, ar sucient s a ar at am c a lungimea AD este constant a. Evident N c a triunghiul ADE este isoscel, deci A este mijlocul lui M DE . Atunci A N este linie mijlocie n ADE, prin E d urmare AD = 2A N = 2AM (deoarece AM A N este romb). Aplic and de dou a ori teorema lui Thales n tri- B C A AB AC unghiul ABC , obt inem c a AM = . In con- D AB + AC 2AB AC cluzie, AD = = constant, deci d trece prin punctul x D. AB + AC G243. Fie O intersect ia diagonalelor trapezului ABCD, cu baza mare CD. Punctele M si N sunt astfel nc at AD separ a M si O, BC separ a N si O, iar M AD N CB OAB. Ar atat i c a BD AC > AB M N. Cosmin Manea si Drago s Petric a, Pite sti Solut ie. Cum AM D AOB , rezult a c a patrulaterul AM DO este inscriptibil, prin urmare AM O ADO. In plus, m(M AO) = m(M AD)+ m(DAO) = m(OAB )+ AO MO m(DAO) = m(DAB ), a sadar M AO DAB . Rezult a c a = , deci AB BD AO BD = AB M O. Analog, din asem anarea N OC BDC obt inem c a BD CO = N O DC. Adun and aceste relat ii, deducem c a BD AC = AB M O + DC N O > AB M O + AB N O = AB (M O + N O) AB M N. Observat ie. De fapt, punctele M, O si N sunt coliniare: m(M ON ) = m(M OA)+ m(AOB ) + m(BON ), iar M OA M DA ABO, BON BCN BAO, a sadar m(M ON ) = m(ABO) + m(AOB ) + m(BAO) = 180 . G244. Fie V ABC un tetraedru, iar M, N, P mijloacele muchiilor V A, V B respectiv V C . Demonstrat i c a 2(AM BC + AN CA + AP AB ) < AV BC + AV CA + AV AB + 3AABC . Mih aly Bencze, Bra sov Solut ie. Complet am tetraedrul V ABC la prisma triunghiular a V BCV1 AC1 si V e P1 mijlocul muchiei V1 C1 . Vom demonstra c a 1 2AP AB < AV AB + AABC (1). Inmult ind cu 2 ambii membri ai inegalit a tii, avem: V (1) 2AABP P1 < AABV V1 + AABCC1 2 P P1 d(A, P P1 ) < CC1 d(A, CC1 )+ + V V1 d(A, V V1 ) 2d(A, P P1 ) < d(A, CC1 ) + d(A, V V1 ) 2 AP2 < AC2 + AV2 , (2)
B P 1 C1 A P

154

unde P2 , C2 , V2 sunt intersect iile dintre planul si P P1 , CC1 respectiv V V1 , iar este planul ce cont ine punctul A si este perpendicular pe planul (V CC1 ). Se observ a u sor c a AP2 este median a n triunghiul AC2 V2 si atunci are loc inegalitatea (2), deci si (1). Scriem cele dou a inegalit a ti analoage lui (1) si, prin adunarea lor, rezult a concluzia. G245. Un triunghi echilateral are v arfurile n interiorul sau pe laturile unui hexagon regulat de latur a1 si nu cont ine centrul hexagonului n interiorul s au. Care este lungimea maxim a posibil a a laturii triunghiului? Marian Tetiva, B arlad Solut ie. R aspunsul este 2/ 3. Dac a hexagonul este ABCDEF, triunghiul AM N pentru care latura M N are ca dreapt a suport mediatoarea comun a a segmentelor BC si EF realizeaz a acest maxim ( n alt imea sa are lungimea 1). Pentru a vedea c a, ntr-adev ar, aceasta este maximul c autat, s a consider am un triunghi echilateral XY Z ale c arui v arfuri sunt situate n interiorul sau pe laturile hexagonului regulat ABCDEF si astfel nc at centrul O al hexagonului nu apart ine interiorului triunghiului XY Z . Interiorul triunghiului este intersect ia benzilor deschise delimitate de c atre o latur a a triunghiului si paralela la acea latur a prin v arful opus, de aceea centrul hexagonului nu apart ine m acar uneia dintre cele trei benzi. S a admitem c a O nu apart ine benzii delimitate de latura Y Z a triunghiului si paralela prin X la Y Z si s a consider am paralela la Y Z prin O. Aceast a paralel a intersecteaz a dou a laturi opuse ale hexagonului si, tot f ar a a restr ange generalitatea, putem considera c a aceast a paralel a intersecteaz a pe AB n P si pe DE n Q. De asemenea, datori a simetriei, putem presupune c a XY Z este inclus n pentagonul P BCDQ. (La limit a, c and paralela prin O la Y Z trece prin v arfuri ale hexagonului, pentagonul devine patrulater.) E clar c a n alt imea din X a triunghiului XY Z este cel mult egal a cu cea mai mare dintre distant ele de la punctele B, C, D ls dreapta P Q, iar la r andul ei, aceast a distant a (oricare ar ea) este cel mult egal a cu 1. De exemplu, dac a ne referim la distant a de la C la P Q, aceasta e cel mult egal a cu CO = 1 (perpendiculara e mai scurt a dec at oblic a). In concluzie, lungimea n alt imii oric arui triunghi echilateral care ndepline ste condit iile din enunt este cel mult egal a cu 1, ceea ce nseamn a c a lungimea laturii unui astfel de triunghi este cel mult egal a 2 cu si cu asta solut ia este complet a. 3

B. Nivel liceal
L236. Pe sfera de centru si raz a 7 se consider a punctele A, B, C astfel nc at BC = 4, CA = 5 si AB = 6. Perpendiculara pe planul triunghiului ABC n centrul cercului nscris n acest triunghi intersecteaz a sfera n punctele M si N . Determinat i lungimea segmentului M N . Temistocle B rsan, Ia si Solut ie. Not am cu O si I centrele cercurilor circumscris, respectiv nscris ale triunghiului ABC . Evident, O este proiect ia lui pe planul (ABC ). In 155

M W J C I A N O B

a+b+c 15 ABC cunoa stem lungimile laturilor, prin urmare p = = , S = 2 2 7 15 7 S abc 8 p(p a)(p b)(p c) = , r = = , R = = , iar OI 2 = R2 4 p 2 4S 7 8 2Rr = . Dac a J este mijlocul segmentului M N , atunci OIJ este dreptunghi, deci 7 8 335 J = . Din triunghiul dreptunghic JM obt inem c a JM 2 = M 2 J 2 = , 7 7 355 . a sadar M N = 2JM = 2 7 L237. Fie A1 , B1 , C1 mijloacele laturilor BC, CA respectiv AB ale triunghiului ascut itunghic ABC . Coarda comun a a cercurilor de diametre BB1 si CC1 intersecteaz a B1 C1 n A2 ; construim analog punctele B2 si C2 . Demonstrat i c a dreptele A1 A2 , B1 B2 si C1 C2 sunt concurente. Neculai Roman, Mirce sti (Ia si) Solut ie. Fie E al doilea punct de intersect ie dintre cercul de diametru BB1 A si dreapta B1 C1 , iar D al doilea punct de intersect ie dintre cercul de diametru CC1 si dreapta B1 C1 . Din B1 C1 A2 B1 C1 BC si m(B1 EB ) = m(C1 DC ) = 90 , rezult a D E c a BCDE este drepuntghi. Folosind triunghiurile ctg B dreptunghice BEC1 si CDB1 , obt inem c a = ctg C B A1 C DB1 EC1 EC1 : = . Punctul A2 ind pe axa BE DC DB1 radical a a cercurilor de diametre BB1 si CC1 , avem A2 E A2 B1 = A2 C1 A2 D A2 C1 A2 B1 + EC1 A2 B1 = A2 C1 A2 B1 + A2 C1 DB1 EC1 A2 C1 ctg B A2 C1 EC1 A2 B1 = A2 C1 DB1 = . Astfel, = si, analog, DB1 A2 B1 ctg C A2 B1 ctg C B2 A1 ctg A C2 B1 A2 C1 C2 B1 B2 A1 = , iar = . De aici, =1 si reciproca ctg A B2 C1 ctg B C2 A1 A2 B1 C2 A1 B2 C1 teoremei lui Ceva conduce la concluzia problemei. Not a. D-l. Titu Zvonaru observ a c a punctul de concurent a a dreptelor A1 A2 , B1 B2 si C1 C2 este, de fapt, izotomicul ( n A1 B1 C1 ) ortocentrului triunghiului A1 B1 C1 . A L238. Fie R un punct n interiorul triunghiului ABC , iar {M } = AR BC, {N } = BR AC , {P } = CR AB. Not am cu , si m asurile unghiurilor AM B , BN C , respectiv CP A. Dac a + C + A, P g + + B + C s i + A + B se a a n interN D E R b atat i c a R este centrul cercului nscris valul 0, , ar 2 n triunghiul ABC . a Marius Dr agan, Bucure sti B M Solut ie. Din teorema lui Ceva (forma trigonomeC sin( + B ) sin( + C ) sin M AB sin N BC sin P CB = 1 tric a) obt inem: sin M AC sin N BA sin P CA sin( C ) sin( A) 156

sin( + A) = 1 sin(+B )sin( +C ) sin( +A) = sin(C ) sin( A) sin( B ) sin( B ) cos( + B A) sin( + C ) cos( + B + + A) sin( + C ) = cos( C B ) sin( A) cos( C + B ) sin( A). Dup a transformarea produselor n sume si efectuarea calculelor, g asim c a sin A cos( + + B + C ) + sin B cos( + + A C ) + sin C cos( + A + B ) = 0 de unde, tin and seama de ipoteza problemei, rezult a c a + + A C = + + B + C = + + B A = . 2 1 1 1 Solut ia acestui sistem este = A + C, = A + C , = C + B, prin urmare R 2 2 2 este intersect ia bisectoarelor triunghiului ABC . L239. Fie ABCD patrulater circumscriptibil cu AB CD, iar A , B , C si D sunt punctele de tangent a ale cercului nscris cu laturile AB, BC, CD respectiv DA. Se noteaz a cu A , B , C , D simetricele punctelor A , B , C respectiv D fat a de mijloacele laturilor pe care se a a. Demonstrat i c a: a) SA B C D SA B C D ; b) SA B C D + SA B C D SABCD . Marius Olteanu, Rm. V alcea Solut ie. Not am AA = AD = A B = DD = u, BB = A B = A A = B C = v, CC = B C = C D = BB = w si DC = DD = D A = CC = t; e nc a O centrul cercului si r raza acestuia. Aplic and teorema n alt imii n triunghiurile dreptunghice AOD si BOC, obt inem c a ut = vw = r2 . Apoi, avem SDC D =
r 2 tg D uw sin D uw uwtr 2 t = = 2 = uw r2 2 2 1 + tg2 D r + t2 1 + t2 2

si nc a trei relat ii similare. Astfel,


Q

A B C D

= SABCD r

vwt uvw uvt uwt + 2 + 2 + 2 r 2 + t2 r + w2 r + v2 r + u2

Mai observ am c a SDC D = S


A B C D

t2 sin D t3 =r 2 , prin urmare 2 r + t2


= SABCD r

t3 w3 v3 u3 + + + r 2 + t2 r2 + w2 r2 + v2 r 2 + u2

a) T in and seama de cele de mai sus, avem: SA B C D SA B C D


: uwt : t3 : uwt t3 r2 + t2 r2 + t2 ut + t2 ut + t2 v 2 + w2 uw + vt uw + vt t2 + u 2 + + t+u v+w u+t v+w (t2 + u2 )(v + w) + (t + u)(v 2 + w2 ) (uw + vt)(u + v + w + t) :

(tw uv )(w + t u v ) 0

r2 r2 t v v t

r2 r2 +t v v t

(t2 v 2 )(r2 t r2 v + t2 v v 2 t) 0 (t v )2 (t + v )(r2 + tv ) 0, 157

relat ie adev arat a. Se constat a c a egalitatea se atinge n cazul n care ABCD este romb. b) Cum SABCD = r(u + v + t + w), inegalitatea de demonstrat revine la
: : t3 uwt + u+v+t+w r 2 + t2 r 2 + t2 v2 + w2 uw + vt uw + vt t2 + u 2 + + + u+v+t+w t+u v+w t+u v+w (t2 + u2 )(v + w) + (t + u)(v 2 + w2 ) (uw + vt)(u + v + t + w)

(tv uw)(t + v u w) 0 (t w)2 (t + w)(r2 + tw) 0,

r2 r2 w w t

t+

r2 r2 w w t

inegalitate adev arat a. Egalitatea se atinge n cazul n care ABCD este trapez isoscel. Not a. S-a primit solut ie corect a de la prof. Nela Ciceu, Bac au. L240. Demonstrat i c a n orice triunghi are loc inegalitatea
: c 2r : bc + 3. R ab + ac a+b

Marian Tetiva, B arlad Solut ia 1 (a autorului). Inegalitatea rezult a n mod evident din identitatea ()
: c 2r : abc + = 3. 2 2 R (b + c)(a (b c) ) a+b

Pentru a vedea de unde se obt ine aceasta, observ am nt ai c a avem cos B + cos C = 2 cos B+C BC A BC cos = 2 sin cos = 2 2 2 2 C B C A B C cos B r a+b 2 cos 2 + sin 2 sin 2 = 2 sin sin sin = , B C 2 2 2 Ra+bc sin 2 sin 2
H

A (p b)(p c) A unde, pentru utlima egalitate, am folosit formulele sin = , cos = 2 bc 2 H p(p a) si cele analoage. Pe de alt a parte, cu teorema cosinusului, avem imediat bc cos B + cos C = (b + c)(a2 (b c)2 ) . 2abc
:

Folosind acestea, precum si formulele obt inute prin permut ari circulare, deducem c a
: : c 1 R = 3 cos B + cos C r a+b Q

=2

(b +

c)(a2

abc , (b c)2 )

de unde rezult a identitatea (*) si, de aici, inegalitatea din enunt . 158

Solut ia 2 (Daniel V acaru, Pite sti). Vom demonstra c a () bc 2(p a) 2r ; R ab + ac b+c

odat a demonstrat a aceast a inegalitate, scriindu-le si pe celelalte dou a similare si sum and, obt inem cerint a problemei. Inegalitatea (**) se rescrie, succesiv, astfel: () r bc r abc pa pa R a R a2 S 4S 4p(p a)(p b)(p c) 2 pa pa p a pa2 4(p b)(p c) a2 (a + c b)(a + b c) a2 .

Aceast a ultim a inegalitate rezult a imediat din inegalitatea mediilor M G M A si, astfel, solut ia este complet a. 2r 2S 4S Solut ia 3 (Titu Zvonaru, Com ane sti). Deoarece = = R p abc 8(p a)(p b)(p c) 8p(p a)(p b)(p c) = , avem: pabc abc
: c : Q 8(p a)(p b)(p c) 2r : ab c + 3= + 1 = R bc + ca a+b c2 (a + b) a+b Q : 2(p c)[4(p a)(p b) c2 ] : 8(p a)(p b)(p c) 2(p c) = = = c 2 (a + b ) a+b c2 (a + b) : 2(p c)(a b)2 : 2(p c)[(c a + b)(c + a b) c2 ] = 0. c2 (a + b) c2 (a + b)

Avem egalitate dac a si numai dac a a = b = c. Not a. S-a primit solut ie corect a de la elevul Ovidiu P auc a, Tru se sti (Boto sani). 3 3 cos x 2 2 sin x leg + , x R. (In atur a cu L241. Ar atat i c a 2 2 x)2 2 (1 + cos 9 (1 + sin x) problema L211 din RecMat-2/2011.) Dumitru Barac, Sibiu a3 b3 Solut ie. Not and sin x = a, cos x = b, avem de ar atat c a + (1 + a2 )2 (1 + b2 )2 1 2 2 , a, b R cu a2 + b2 = 1. Observ am c a pentru a = b = are loc egali9 2 t3 tatea. Folosim metoda parabolei tangente: c aut am A, B R astfel nc at (1 + t2 )2 2 At + B, t R (de fapt, este sucient pentru t [1, 1]). Impunem condit iile 3 3 t t = (At2 + B ) si = (At2 + B ) ; obt inem 2 )2 (1 + t2 )2 t= (1 + t 1 1 1 1 t= t= t=
2 2 2 2

159

2 A 5 2 2 = + B , respectiv A = , deci B = . Demonstr am acum c a 9 2 27 54 t3 2 5 2 2 () t + , t R. 2 2 (1 + t ) 27 54 Avem succesiv: () 27t3 2 (10t2 + 1)(t4 + 2t2 + 1) 10t6 + 21t4 27t3 2 + 12t2 + 1 0 (t 2 1)2 [(5t2 + 5t 2 + 16)t2 + (2t2 + 2t 2 + 1)] 0, iar a inegalitate este adev arat a ntruc at trinoamele 5t2 + 5t 2 + 16 si 2t2 + aceast 2t 2 + 1 au discriminant ii negativi. Acum, lu and n (*) t = a si t = b si adun and membru cu membru inegalit a t ile obt inute, rezult a cerint a problemei. 3 3 2 2 este efectiv atins a si este mai bun a dec at estimarea Not am c a marginea 9 16 din problema L211 citat a. L242. Un paralelipiped dreptunghic are dimensiunile x, y, z si diagonala d. Ar atat i c a d4 d4 d4 27 + + , 4 4 4 4 4 4 ad + bx ad + by ad + bz 9a + b oricare ar a, b > 0, 6a 5b. ( In leg atur a cu L231 din RecMat-2/2012.) Titu Zvonaru, Com ane sti d4 9 b(d2 + 3x2 )(d2 3x2 ) Solut ie. Avem: = = ad4 + bx4 9a + b (ad4 + bx4 )(9a + b) 2 2 2 2 2 2 2 2 2 2 b (4x + y + z )(y x ) (4x + y + z )(z x ) + , prin urmare 9a + b ad4 + bx4 ad4 + bx4
:

d4 27 b : (4x2 + y 2 + z 2 )(y 2 x2 ) (4x2 + y 2 + z 2 )(z 2 x2 ) = + 4 4 ad + bx 9a + b 9a + b ad4 + bx4 ad4 + bx4 b : (4x2 + y 2 + z 2 )(y 2 x2 ) (x2 + 4y 2 + z 2 )(x2 y 2 ) = + = 9a + b ad4 + bx4 ad4 + by 4 2 x + 4y 2 + z 2 4x2 + y 2 + z 2 b : 2 (x y 2 ) = = 9a + b ad4 + by 4 ad4 + bx4 b : (x2 y 2 )2 [(b 3a)(x4 + y 4 ) 3az 4 + (5b 6a)x2 y 2 + (b 6a)(x2 z 2 + y 2 z 2 ) = , 9a + b (ad4 + by 4 )(ad4 + bx4 )

iar acest num ar este cel mult egal cu 0 n condit iile din enunt (deoarece 5b 6a implic a b < 3a si b < 6a). Avem egalitate dac a si numai dac a x = y = z. L243. Pentru m, n N si a, b, c R i inegalitatea + , demonstrat m m m n a b c (a + bn + cn )2 3. a3m+2n + b3m+2n + c3m+2n ( In leg atur a cu problema VIII.149 din RecMat-1/2012.) Neculai Stanciu, Buz au 160

Solut ia 1 (Emanuel Necula, elev, C ampulung Muscel, Titu Zvonaru, Com ane sti si Daniel V acaru, Pite sti). Putem presupune c a a b c. Folosind inegalitatea lui Ceb sev, inegalitatea mediilor M A M G si binecunoscuta 3(x2 + 2 2 2 y + z ) (x + y + z ) , obt inem: 3(a3m+2n + b3m+2n + c3m+2n ) = 3(a3m a2n + b3m b2n + c3m c2n ) (a3m + b3m + c3m )(a2n + b2n + c2n ) 3am bm cm (a2n + b2n + c2n ) am bm cm (an + bn + cn )2 . Avem egalitate dac a si numai dac a a = b = c. Solut ia 2 (a autorului). Folosind inegalitatea avem: am bm cm (an + bn + cn )2 = [a 2 b 2 c 2 (an + bn + cn )]2 = = [(am+2n bm cm ) 2 + (am bm+2n cm ) 2 + (am bm cm+2n ) 2 ]2 3(am+2n bm cm + am bm+2n cm + am bm cm+2n ). Din inegalitatea mediilor ponderate obt inem c a am+2n bm cm m + 2n 3m+2n m m a + b3m+2n + c3m+2n 3m + 2n 3m + 2n 3m + 2n
1 1 1 m m m

a+

b+

a+b+c , 3

si nc a dou a inegalit a ti similare. Adun andu-le membru cu membru, rezult a cerint a problemei. i c a are loc inegalitatea L244. Fie a, b, c R + , a b c. Demonstrat (a2 + c2 )(ab + ac + bc) 2ac(a2 + b2 + c2 ) 2c(a b)(a c)(b c). Gabriel Dospinescu, Paris si Marian Tetiva, B arlad Solut ia 1 (a autorului). Totul rezult a din identitatea (a2 + c2 )(ab + ac + bc) 2ac(a2 + b2 + c2 ) 2c(a b)(a c)(b c) = = c2 (a c)2 + 2c2 (a b)(b c) + 2c(a c)2 (b c) + (a c)3 (b c). Propunem cititorului s a verice aceast a identitate, folosind eventual metoda norm arii (a se vedea, de exemplu, Inegalit a ti -idei si metode de Mihai Onucu Drimbe, Editura GIL, Zal au, 2003, pp. 79-80). Inegalitatea din enunt nt are ste inegalitatea (G. Dospinescu): a2 + c2 a2 + b2 + c2 , a b c > 0. 2ac ab + ac + bc Solut ia 2 (Daniel V acaru, Pite sti). Pornim din membrul st ang: M S = (a2 + c2 )(ab + bc) ac(a2 + c2 ) 2ac b2 (a2 + c2 )(ab + bc ca) (a2 + c2 ) b2 = = (a2 + c2 )(ab + bc ca b2 ) = (a2 + c2 )(a b)(b c). 161

R am ane s a prob am c a a2 + c2 2c(a c); acest lucru revine la a2 + 3c2 2ac, ceea ce este evident, ntruc at a2 + 3c2 a2 + c2 2ac. Not a. Am mai primit solut ie corect a de la d-l. Titu Zvonaru. Q Q k k L245. Fie f, g : [0, 1] [0, 1] dou a funct ii continue astfel nc at f g n n Q Q k 1 f g , k, n N , k n. Demonstrat i c a cele dou a funct ii sunt egale. k n Florin St anescu, G ae sti Solut ie (Moubinool Omarjee, Paris). Obt inem imediat c a 1:k f n n
n k=1

k n

k g n

1 n

1: k f g n n

Q Q

.
Q Q

Funct ia f g ind continu a, deci integrabil a, limita sirului este nit a; atunci lim 1 n n
n

n 1 2 k f g n k=1 n

Q Q

n 1 2 k f g n k=1 n

exist a si

= 0. Deducem c a
1

1:k f n n n lim
k=1 1

k n

k n

=
0

x|f (x) g (x)|dx 0,

de unde
0

x|f (x) g (x)|dx = 0. Ins a funct ia x x|f (x) g (x)| este continu a si

nenegativ a; av and integrala nul a, rezult a c a funct ia este nul a. De aici, obt inem c a f (x) = g (x), x (0, 1]. Faptul c a f (0) = g (0) urmeaz a din continuitatea funct iilor f si g n 0.
1 1

Not a. Autorul problemei arat a mai nt ai c a


0

P (x)f (x)dx =
0

P (x)g (x)dx,

oricare ar polinomul cu coecient ii reali P si folose ste apoi faptul c a orice funct ie continu a pe un interval este limita uniform a a unui sir de funct ii polinomiale. Am primit solut ie corect a si de la d-l Daniel V acaru.

Primul num ar al Colect iei ,,Recreat ii Matematice


1. D. Br anzei, Al. Negrescu

Probleme de pivotare,

Ed. ,,Recreat ii Matematice, Ia si, 2011 (208 pag.) poate procurat printr-o simpl a cerere la adresa: t birsan@yahoo.com si indicarea adresei po stale proprii. Cartea va trimis a cu plata ramburs la adresa indicat a contra sumei de 25 lei (inclusiv taxe po stale). 162

Probleme propuse1
Clasele primare
P.269. S apte forme geometrice sunt a sezate astfel: Mutat i trei forme astfel nc at n sirul obt inut triunghiul s a e la mijloc, iar p atratele s a e de o parte si de alta a lui. (Clasa I ) Mariana Manoli, elev a, Ia si P.270. Afat i numerele naturale a si b astfel nc at a 26 = 4 b. (Clasa I ) Denisa Apetrei, elev a, Ia si P.271. Din cei 28 elevi ai unei clase, 25 ndr agesc fotbalul si 24 tenisul. Fiecare dintre elevi ndr age ste cel put in un sport. C a ti elevi ndr agesc un singur sport? (Clasa I ) Mihaela Buleandr a, elev a, Ia si P.272. In trei cutii sunt 80 de nasturi. C a ti nasturi sunt n ecare cutie, dac a n primele dou a cutii sunt 64 nasturi, iar n ultimele dou a 41? (Clasa a II-a) Maria Racu, Ia si P.273. Exist a 12 numere mai mari ca zero, pare, diferite ntre ele si care s a aib a suma 154? (Clasa a II-a) Dumitrit a Grigoriu, elev a, Ia si P.274. Intr-o urn a sunt 34 de jetoane, pe ecare ind scris un num ar; exist a jetoane cu numere de parit a ti diferite. Dup a ce s-au scos mai mult de 14 jetoane cu numere pare, n urn a au r amas tot at atea jetoane cu numere pare. C ate jetoane cu numere impare sunt n urn a? (Clasa a II-a) Iustina Diaconu, elev a, Ia si P.275. Se d a sirul de numere 0, 3, 6, 9, 12, . . . C ate numere din sir, mai mici dec at 500, au suma cifrelor 18? (Clasa a III-a) Nicoleta Cump at a, elev a, Ia si P.276. S a se ae a si b stiind c a au loc egalit a tile: a : 2 15 = b : 3, a + b = 90. (Clasa a III-a) Alexandra Tololoi, elev a, Ia si P P.277. In desenul al aturat, t aiat i un num ar minim O de segmente, de aceea si lungime cu segmentul AB , astfel K L M N n at s a nu mai r am an a nici un p atrat. (Clasa a III-a) Maria Nastasia, elev a, Ia si J H I G F P.278. Suma a patru numere naturale diferite este S . Dac a ad aug am 2 la primul num ar, sc adem 2 din al doilea num ar, nmult im cu 2 al treilea num ar si mp art im la 2 al A B E D C patrulea num ar, vom obt ine acela si rezultat. Ar atat i c a suma celor patru numere se mparte exact la 9. (Clasa a III-a) Andreea B zd g a, elev a, Ia si P.279. Suma a trei numere naturale nenule este zecimea produsului lor, iar suma ultimelor dou a este zecimea primului num ar. Aat i suma celor 3 numere. (Clasa a IV-a) Amalia Munteanu, elev a, Ia si
1 Se

primesc solut ii p an a la data de 15 ianuarie 2014.

163

P.280. Fie abcde un num ar cu suma cifrelor patru. Ar atat i c a, dac a mp art im acest num ar la trei, obt inem restul 1. (Clasa a IV-a) Mihaela G lc a, elev a, Ia si P.281. Fie un poligon cu 10 laturi astfel nc at oricare trei v arfuri nu se a a pe aceea si dreapt a. C ate segmente se formeaz a prin unirea oric aror dou a v arfuri neal aturate? (Clasa a IV-a) Andreea Simion, elev a, Ia si P.282. In 10 co suri sunt mere, pere si gutui, n total 44 de fructe. In ecare co s sunt fructe de toate felurile. S a se arate c a exist a dou a co suri care au acela si num ar de mere, dou a co suri care au acela si num ar de pere si dou a co suri care au acela si num ar de gutui. (Clasa a IV-a) Petru Asaftei, Ia si

Clasa a V-a
V.165. Ar atat i c a fract ia 3n+2 5n + 3n 5n+2 15n este reductibil a. abcabc Nicolae Iv a schescu, Craiova

V.166. Numere 1, 2, 2, 3, 3, 3, 4, 4, 4, 4, 5, 5, 5, 5, 5, . . . se scriu succesiv n c ate una din 2013 c asut e dispuse circular. In momentul n care ajungem la o c asut a ocupat a, stergem num arul deja existent si scriem noul num ar. Aat i suma numerelor scrise n c asut e imediat dup a ce s-a scris primul 2013. Silviu Boga, Ia si 1 V.167. In tabloul al aturat sunt scrise, n ordine cresc atoare, 4 9 toate p atratele perfecte care nu se divid cu 6. Dac a pe r andul 16 25 49 64 81 100 121 58 al 25-lea num ar este x2 , determinat i valoarea lui x. .............................. Vlad Tuchilu s, elev, Ia si V.168. a) Ar atat i c a num arul 22013 nu poate scris ca sum a de cel put in dou a numere naturale consecutive. b) Scriet i num arul 3 22013 ca sum a de cel put in dou a numere naturale consecutive. C ate asemenea modalit a ti de scriere exist a? Elena Iurea, Ia si V.169. Demonstrat i c a num arul N = 22
2013

+ 27 se divide cu 31. Tamara Culac, Ia si

V.170. Intr-o urn a sunt 68 de bile. Dou a persoane joac a urm atorul joc: ecare, alternativ, scoate din urn a ntre una si cinci bile, p an a c and urna se gole ste. C a stig a cel care a scos ultimul trei bile deodat a. Care dintre cei doi juc atori are strategie de c a stig? Mihai Cr aciun, Pa scani V.171. Consider am num arul n = 1 2 3 . . . 9, unde n ecare p atr a tel se a a unul dintre semnele ,, sau ,,:. S tiind c a n este num ar natural, ar atat i c a este par si cel put in egal cu 70. Radu Miron, elev, Ia si 164

Clasa a VI-a
VI.165. Se consider a un triunghi isoscel ABC , AB = AC , cu proprietatea c a exist a punctul D pe latura BC astfel nc at BD = 2DC si m(BAD) = 90 . Determinat i m asurile unghiurilor triunghiului. Elena Iurea, Ia si VI.166. Se consider a triunghiul isoscel ABC , AB = AC , si e D si E picioarele bisectoarelor din B , respectiv C . Punctul F este astfel nc at DF CE , DF = BD, iar E si F sunt separate de AC. Demonstrat i c a punctele B, C si F sunt coliniare. Ion P atra scu, Craiova VI.167. Ar atat i c a nu exist a numere prime de trei cifre, av and produsul acestor cifre egal cu 210. Mirela Marin, Ia si 1 1 1 VI.168. Demonstrat i c a num arul A = + +...+ nu este natural. 1007 1008 2013 Doina Stoica si Mircea Stoica, Arad VI.169. Fie a N si num arul n = a2 + a3 + . . . + a2013 . Demonstrat i c a num arul a+n n este natural nenul. 1 + a + a2 1 + a + a2 + a3 Ionel Tudor, C alug areni VI.170. Stabilit i care este cel mai mare num ar de numere prime care pot g asite printre 15 numere naturale consecutive. Constantin Dragomir, Pite sti VI.171. Ar atat i c a exist a o innitate de numere naturale a cu proprietatea c aa si a + 1 sunt, ecare, suma a c ate trei p atrate perfecte nenule. Nicolae Iv a schescu, Craiova

Clasa a VII-a
VII.165. Fie a, x, y numere reale strict pozitive cu proprietatea c a x(y a) y (a x). Demonstrat i c a a2 xy. Gheorghe Iacob, Pa scani VII.166. Dac a x, y, z sunt numere reale pozitive, ar atat i c a x2 + 4x + 4 + 8y y 2 + 4y + 4 + 8z z 2 + 4z + 4 + 8x + + 48. z x y Bogdan Chiriac, Bac au VII.167. Dac a ecuat ia |x 1| + |x 2| + . . . + |x 2013| = y (y + 1) are o singur a solut ie (x0 , y0 ) R R , calculat i x + y . 0 0 + Liviu Smarandache, Craiova VII.168. Fie n N xat. Ar atat i c a exist a p N astfel nc at num arul n2 + k 2 s a nu e p atrat perfect, oricare ar k N, k > p. Marian Pant iruc, Ia si 165

VII.169. Fie D piciorul bisectoarei din A n triunghiul ABC . Cercul de diametru AD intersecteaz a a doua oar a laturile AB si AC n mijloacele lor. Ce particularitate are triunghiul ABC ? Dar raza cercului? Temistocle B rsan, Ia si VII.170. Se consider a triunghiul ABC , dreptunghic n A. Punctele D, E, P si Q se a a pe segmentele AC, AB, CE , respectiv BD, astfel nc at ABD ACE , DP CE si EQ BD. a) Demonstrat i c a punctele A, D, E, P si Q sunt conciclice. b) Ar atat i c a DEQP este trapez dac a si numai dac a AB = AC. Petru Asaftei, Ia si
> VII.171. Fie M un punct pe latura BC a triunghiului ABC , cu unghiul A ascut it. Perpendiculara n M pe BC taie dreapta AC n punctul N . Demonstrat i c a M N sin A + M C cos A CN. C and se atinge egalitatea? C at alin Calistru, Ia si

Clasa a VIII-a
VIII.165. Fie N si P centrele fet elor ABB A , respectiv ADD A ale unui para lelipiped dreptunghic ABCDA B C D . Dac a exist a un punct M pe diagonala (AC ), diferit de mijlocul acesteia, cu proprietatea c a M N AB si M P AD , ar atat i c a ABCDA B C D este cub. S tefan Dominte, elev, Ia si VIII.166. Piramida patrulater a regulat a V ABCD are muchia bazei AB = a si n alt imea V O = h. Determinat i raza sferei nscris a n piramid a. Adrian Corduneanu, Ia si VIII.167. Determinat i numerele ntregi k pentru care num arul n = k 4 + 8k 3 2 35k 24k + 161 este natural prim. Mihai Haivas, Ia si VIII.168. Se consider a sistemul de ecuat ii ax + by = cx2 + dy 2 = , a, b, c, d, , R . a2 b2 2 + = . Aat i solut ia c d Temistocle B rsan, Ia si VIII.169. Pentru a, b, c R +, demonstrat i inegalitatea

Ar atat i c a sistemul are solut ie unic a dac a si numai dac a sistemului n acest caz.

a b c 3(a + b + c) + 2 + 2 . b2 + c2 c + a2 a + b2 2(a2 + b2 + c2 ) Marian Cucoane s, M ar a se sti 166

2m+1

VIII.170. Fie m N si numerele reale a si b cu a + b 0. Ar atat i c a a2m+1 + m m a b (a + b). Ovidiu Pop, Satu Mare

VIII.171. Demonstrat i c a nu exist a numere naturale nenule a, b si c pentru care a(a2 b2 + 1) = c2 . Cosmin Manea si Drago s Petric a, Pite sti

Clasa a IX-a
IX.141. Dac a a, b, c, d sunt numere reale pozitive, ar atat i c a a + b2 + c4 + 4 b + c2 + d4 + 4 c + d2 + a4 + 4 d + a2 + b4 + 4 + + + 28. d a b c C at alin Cristea, Craiova IX.142. Determinat i funct iile f : R R cu proprietatea c a f (x4 y 4 ) = (x y )(x2 + y 2 )(f (x) + f (y )), x, y R. Lucian Tut escu, Craiova si Ion Nedelcu, Ploie sti IX.143. Rezolvat i n R& sistemul x + y = 1 x2 + y 2 + 2x2 y 2 sin z = 1 x3 + y 3 x6 y 6 sin3 z = 1.
3

Vasile Chiriac, Bac au IX.144. Se consider a triunghiul ADE , dreptunghic n D, si triunghiul DAP , dreptunghic n A, astfel nc at (AE ) (DP ) = {G} si 2DE = AP. a) Ar atat i c a exist a o innitate de triunghiuri ABC n care AE este median a, iar AD este n alt ime. b) Demonstrat i c a cercurile circumscrise tuturor triunghiurilor ABC trec prin punctul P . Romant a Ghit a si Ioan Ghit a Blaj 2r ha IX.145. Demonstrat i c a n orice triunghi ABC are loc inegalitatea , ia R unde ha si ia reprezint a lungimile n alt imii, respectiv bisectoarei din A. Constantin Dragomir, Pite sti

Clasa a X-a
X.141. Rezolvat i ecuat ia 2013x + log2013 x = 2013x . Lucian Tut escu, Craiova si Aurel Chirit a, Slatina X.142. Fie z1 , z2 , z3 trei numere complexe distincte, de modul 1, astfel nc at z1 z2 z3 = (z1 + z2 + z3 )(z1 z2 + z1 z3 + z2 z3 ). Ar atat i c a cele trei numere sunt axele v arfurilor unui triunghi dreptunghic. Ion Nedelcu, Ploie sti si Dimitru S avulescu, Bucure sti 167
3 2

D X.143. Determinat i numerele reale x , + cu proprietatea c a 2013x +1 = 2 2012x + | sin x cos x|. Sven Cortel, elev, Satu Mare X.144. Fie H ortocentrul triunghiului ABC si M N BC , P QAC, RS AB astfel nc at M N P Q RS = {H }, iar punctele M, N, P, Q, R si S sunt situate pe laturile triunghiului. Ar atat i c a: PQ RS MN + + = 4R; a) sin A sin B sin C 3 8R b) M N P Q RS . 3 3 Bogdan Victor Grigoriu, F alticeni > X.145. Se consider a triunghiul ABC cu m(A) = 90 si m(B ) = 60 , iar punctele D si E de pe latura BC sunt astfel nc at AE este bisectoarea unghiului BAD, iar AD = CE . Determinat i m asura unghiului CAD. Titu Zvonaru, Com ane sti

Clasa a XI-a
XI.141. Fie matricele A, B M2 (R) cu AB = BA si det (A + iB ) = 0. Calculat i determinantul matricei A5 + A4 B + AB 4 + B 5 , funct ie de a = det A. Dan Popescu, Suceava XI.142. Consider am ecuat ia X 2 + Y 2 + Z 2 + In = XY + XZ + Y Z , unde X, Y, Z sunt matrice din Mn (R), n 2, care comut a dou a c ate dou a. a) Ar atat i c a ecuat ia nu are solut ii pentru n impar. b) Dac a n este par, ar atat i c a ecuat ia are o innitate de solut ii. Dumitru Cr aciun, F alticeni 2 2 XI.143. IntreD unghiurile triunghiului ABC are loc relat ia A = B + C 2 . De . monstrat i c a A ( 2 1), 2 Ioan S ac aleanu, H arl au XI.144. S irul (xn )nN este astfel nc at x0 R\Q, iar xn (xn+1 1) = 1, n N. Determinat i limita sirului. Mih aly Bencze, Bra sov XI.145. Se consider a funct ia derivabil a f : [a, b] R cu f (a) = a si f (b) = b. Demonstrat i c a oricare ar n N , exist a x1 , x2 , . . . , xn (a, b) astfel nc at 1 1 1 + + ... + = n. f (x1 ) f (x2 ) f (xn ) Dan Nedeianu, Drobeta Turnu Severin

Clasa a XII-a
XII.141. Dac a f : R R, f (x) =
1 0

f (x) dx f (x)

1 0

x4 + x + 1 1 , calculat i lim 3 x x x2 + 1

f (t)dt si
0

f (x) f (x)

dx. Constantin Dragomir, Pite sti 168

XII.142. Fie a R si funct ia continu a f : R R a f (x) + + cu proprietatea c a dx . f (x) = 1, x R. Calculat i 2 a (x + 2013)(1 + f (x)) D.M. Batinet u-Giurgiu, Bucure sti si Neculai Stanciu, Buz au
4 XII.143. Calculat i (x sin x + cos x) 1 sin x 1 cos xdx, x 0, . 2 Dan Nedeianu, Drobeta Turnu Severin XII.144. Demonstrat i c a nu exist a numere naturale n si m astfel nc at 19 s a divid a 5n + 7m . Marian Cucoane s, M ar a se sti XII.145. Fie (A, +, ) inel cu 1 = 0, av and un num ar impar de elemente, n care are loc implicat ia: ,,dac a x2 2xy + y 2 = 1 + 1 + 1 + 1, atunci x + y = 1 + 1 + 1 + 1. Dac a 1 + 1 nu este divizor al lui zero, demonstrat i c a A este izomorf cu Z3 . Florin St anescu, G ae sti

Probleme pentru preg atirea concursurilor


A. Nivel gimnazial
G246. Doi copii, A si B , joac a un joc. Acesta se desf a soar a pe un careu format din a b p atr a telele, n care a si b sunt numere naturale impare, propuse ecare de c atre unul dintre cei doi copii. Juc atorii bifeaz a, pe r and, c ate o c asut a din careu, astfel: A ncepe jocul prin bifarea unui p atr a tel (m, n), unde m reprezint a linia, iar n coloana p atr a telului bifat. Apoi, B bifeaz a unul dintre p atr a telele (m 1, n 3) sau (m 3, n 1), aat n interiorul careului. De ecare dat a c and un juc ator vine la r and, el alege o pozit ie (p, q ) deja bifat a si are voie s a bifeze una dintre pozit iile (p 1, q 3) sau (p 3, q 1) care este nc a nebifat a n careu. Pierde juc atorul care, atunci c and i vine r andul, nu mai are ce bifa. Demonstrat i c a A are strategie de c a stig. Silviu Boga, Ia si G247. Fie A = {1, 2, 3, . . . , n}, n 6, si X, Y dou a submult imi disjuncte ale lui A, X Y = A, av and ecare cel put in trei elemente. Demonstrat i c a exist a x, y X , x = y si a, b Y, a = b, astfel nc at x y = a b. Gheorghe Iurea, Ia si G248. Dac a a N ar atat i c a num arul 5a(a2 + 1) nu este p atrat perfect. Gheorghe Iurea, Ia si m 4 G249. Rezolvat i n numere naturale ecuat ia 85 n = 4. Cristinel Mortici, T argovi ste 3 3 G250. Demonstrat i c a a + b 2 ab(a 2b)(b 2a), oricare ar numerele reale pozitive a si b. Gabriel Popa, Ia si G251. Dac a a, b, c sunt numere reale poztive cu ab + bc + ca = 3, ar atat i c a a2 (b + c) + b2 (c + a) + c2 (a + b) 6. Monica Golea, elev a, Craiova 169

G252. Fie n N, n 2 si numerele reale pozitive x1 , x2 , . . . , xn . Dac a S = x1 + x2 + . . . + xn , demonstrat i c a Q 1 1 1 2 max x1 + , x2 + , . . . , xn + . S x1 S x2 S xn n1 Ani Dr aghici si Mariana M arculescu, Craiova G253. Fie M un punct oarecare pe latura AB a p atratului ABCD. Bisectoarea unghiului M DC intersecteaz a BC n N . Ar atat i c a BM + BN < AM + CN. Cecilia Deaconescu, Pite sti G254. Diagonalele trapezului ABCD se intersecteaz a n O. Paralela prin O la baza AB intersecteaz a latura BC n P . Punctul Q este situat n semiplanul opus celui determinat de dreapta AD si punctul B , iar dreptele QB si QC intersecteaz a AD n R, respectiv S . Demonstrat i c a dreptele P Q, BS si CR sunt concurente. Claudiu-S tefan Popa, Ia si G255. Fie AB si AC tangentele din punctul A la un cerc C (B si C ind punctele de tangent a) si R regiunea din plan determinat a de arcul mic BC al cercului C si segmentele AB si AC . Demonstrat i c a M N max(AB, AC ), oricare ar punctele M si N din R. Marian Tetiva, B arlad

B. Nivel liceal
>) 90 , L246. Fie ABC cu m(A nscris n cercul C . Pe latura BC se consider a punctele D si D astfel nc at ABC CAD si ACB BAD . Cercul tangent dreptelor AD, BD si cercului C este tangent segmentului BD n M . Cercul tangent dreptelor MN AD , CD si cercului C este tangent segmentului CD n N . Ar atat i c a 2 1. BC Neculai Roman, Mirce sti (Ia si) L247. Pe laturile BC, CA si AB ale triunghiului ABC se consider a punctele A1 , B1 , respectiv C1 astfel nc at AB + BA1 = AC + CA1 , AB + AB1 = BC + CB1 si AC + AC1 = BC + BC1 . Dac a A2 , B2 si C2 sunt punctele de tangent a ale cercului 2 2 + + B1 C1 nscris n triunghiul ABC cu laturile BC, CA, respectiv AB , ar atat i c a A1 B1 2 2 2 2 C1 A1 A2 B2 + B2 C2 + C2 A2 . Marius Olteanu, Rm. V alcea L248. Demonstrat i c a n orice triunghi are loc inegalitatea 3(ra + rb + rc ) 1 1 1 6 + + . 2 2p ra + rb rb + rc rc + ra 5R + 2r Andi Gabriel Brojbeanu, elev, T argovi ste

L249. Fie ABCD un patrulater at at inscriptibil, c at si circumscriptibil. Dac a p e+f not am cu e si f lungimile diagonalelor, demonstrat i c a . ef 2 2Rr Vasile Jigl au, Arad L250. Stabilit i pentru care dintre numerele 1, 2, . . . , 9 este adev arat a egalitatea 2 4 + tg = 3n. tg tg n n n Ionel Tudor, C alug areni 170

L251. Fie n N, n 2, si numerele reale nenegative x1 , x2 , . . . , xn cu pro2 2 2 prietatea c a x2 + x + . . . + x i c a (x1 + x2 + . . . + xn )3 n = 3n . Demonstrat 1 2 9n(x1 x2 + x1 x3 + . . . + xn1 xn ). Lucian Tut escu si Ionut Iv anescu, Craiova L252. Fie n N, n 5, si numerele reale a1 < a2 < . . . < an . Se calculeaz a toate sumele ai + aj , i = j , obt in and t rezultate distincte. Demonstrat i c a t 2n 3 si c a t = 2n 3 dac a si numai dac a a1 , a2 , . . . , an este progresie aritmetic a. Titu Zvonaru, Com ane sti L253. Fie a, b, c trei numere reale pozitive cu a c si x, y, z [a, c] astfel nc at 1 1 1 1 1 1 atat i c a numerele x, y si z coincid x+y+z = a+b+c si + + = + + . Ar x y z a b c ntr-o anumit a ordine, cu a, b si c. Marian Tetiva, B arlad L254. Determinat i numerele reale x, y, z din intervalul [1, 3] astfel nc at x2 + y 2 + 3 3 3 z = 14 si x + y + z = 36. Marian Tetiva, B arlad
2

L255. Se consider a numerele reale a < c < b si sirul (xn )n1 . Orice sub sir convergent al sirului (xn ) are limita a sau limita b. Not am An = {k N|k n si xk c} si Bn = {k N|k n si xk > c}. Dac a exist a si este nit a si nenul a limita card An x1 + xn + . . . + xn L = lim , ar atat i c a sirul yn = este convergent si aat i n card Bn n limita sa (funct ie de a, b si L). Studiat i si cazurile L = 0 si L = +. Cristinel Mortici, T argovi ste

Training Problems for Mathematical Contests


A. Junior Highschool Level
G246. Two children, A and B , play a game. This takes place on a rectangular square consisting of a b small squares, where a and b are odd natural numbers, each of them proposed by one of the two children. The players mark, successively and once at a time, a cell in the table as it follows : A begins the game by marking a small square (m, n), where m represents the row and n the column of the marked cell. Then, B marks one of the cells (m 1, n 3) or (m 3, n 1), situated inside the big table. Every time when a player comes to his turn, he chooses an already marked position (p, q ) and he is allowed to mark one of the positions ( p 1, q 3) or ( p 3, q 1) whick is still blank on the table. The player who has no more cell to mark when its turn comes up loses the game. Prove that A has a strategy for winning. Silviu Boga, Ia si G247. Let A = {1, 2, 3, . . . , n}, n 6, and X, Y be two disjoint subsets of the set A, X Y = A, each of them consisting of at least three elements. Prove that four elements x, y X , x = y and a, b Y, a = b exist such that x y = a b. Gheorghe Iurea, Ia si 171

G248. If a N show that the number 5a (a 2 + 1) is not a perfect square. Gheorghe Iurea, Ia si m 4 G249. Solve in natural numbers the equation 85 n = 4. Cristinel Mortici, T argovi ste 3 3 G250. Prove that a + b 2 ab(a 2 b)(b 2 a), any would be the positive real numbers a and b. Gabriel Popa, Ia si G251. If a, b, c are positive real numbers such that ab + bc + ca = 3, show that a2 (b + c) + b2 (c + a) + c2 (a + b) 6. Monica Golea, elev a, Craiova G252. Let n N, n 2 and consider the positive real numbers x1 , x2 , . . . , xn . If S = x1 + x2 + . . . + xn , prove that
Q

max x1 +

1 1 1 , x2 + , . . . , xn + S x1 S x2 S xn

2 . n1

Ani Dr aghici si Mariana M arculescu, Craiova G253. Let M be an arbitrary point on the side AB of the square ABCD. The angle bisector of M DC intersects the side BC at point N . Show that BM + BN < AM + CN. Cecilia Deaconescu, Pite sti G254. The diagonals of the trapezium ABCD cut each other at point O. The line through O that is parallel to the base AB intersect the side BC at P . The point Q is situated in the opposite half-plane to the one determine by line AD and point B , and the lines QB and QC intersect AD at R, respectively at S . Prove that the lines P Q, BS and CR are concurrent. Claudiu-S tefan Popa, Ia si G255. Let AB and AC be the tangents from point A to a circle C (B and C being the points of contact) and let R be the plane region closed by the small arc BC of circle C and the line segments AB and AC . Prove that M N max(AB, AC ), any would be the points M and N in R. Marian Tetiva, B arlad

B. Highschool Level
>) 90 , which is inscribed in L246. Let us consider the triangle ABC with m(A the circle C . The points D and D are taken on the side BC such that ABC CAD and ACB BAD . The tangent circle to the lines AD, BD and to the circle C is also tangent to the line segment BD at M . The circle tangent to the lines AD , CD MN and to circle C is tangent to the line segment CD at N . Show that 2 1. BC Neculai Roman, Mirce sti (Ia si) L247. Three points are consideered on the sides BC, CA and AB of the triangle ABC, respectively denoted A1 , B1 and C1 , such that AB + BA1 = AC + CA1 ,

172

AB + AB1 = BC + CB1 and AC + AC1 = BC + BC1 . If A2 , B2 and C2 are the contact points of the circle inscribed in triangle ABC with the sides BC, CA and 2 2 2 2 2 respectively AB , show that A1 B1 + B1 C1 + C1 A2 1 A2 B2 + B2 C2 + C2 A2 . Marius Olteanu, Rm. V alcea L248. Prove that the following inequality holds in any triangle : 1 1 1 6 3(ra + rb + rc ) + + . 2 2p ra + rb rb + rc rc + ra 5 R + 2r Andi Gabriel Brojbeanu, elev, T argovi ste L249. Let ABCD be a bicentric quadrilateral, and let e and f denote the lengths p e+f of its diagonals. Prove that . ef 2 2Rr Vasile Jigl au, Arad L250. Establish which numbers n among 1, 2, . . . , 9 satisfy the equation tg n 4 2 + tg = 3n. tg n n Ionel Tudor, C alug areni L251. Let n N, n 2 and consider the nonnegative numbers x1 , x2 , . . . , xn 2 2 3 2 with the property that x2 1 + x2 + . . . + xn = 3n . Prove that (x1 + x2 + . . . + xn ) 9 n (x1 x2 + x1 x3 + . . . + xn1 xn ). Lucian Tut escu si Ionut Iv anescu, Craiova L252. Let n N, n 5 and the real numbers a1 < a2 < . . . < an . All the sums ai + aj , i = j are calculated, giving distinct results. Prove that t 2 n 3 and that t = 2 n 3 if and only if a1 , a2 , . . . , an is an arithmetic progression. Titu Zvonaru, Com ane sti L253. Let a, b, c be positive real numbers with a c and x, y, z [a, c] such 1 1 1 1 1 1 that x + y + z = a + b + c and + + = + + . Show that the numbers x, y x y z a b c and z respectively coincide, in a certain order, with a, b and c. Marian Tetiva, B arlad L254. Find the real numbers x, y, z in [1, 3] such that x2 + y 2 + z 2 = 14, and x3 + y 3 + z 3 = 36. Marian Tetiva, B arlad L255. The real numbers a < c < b and the sequence (xn )n 1 are considered. Any convergent subsequence of sequence (xn ) has limit a or limit b. Let us denote An = {k N | k n and xk c} and Bn = {k N | k n and xk > c}. If card An there exists the limit L = lim and it is nite and nonzero, show that the n card Bn x1 + xn + . . . + xn converges and nd its limit (as a function of a, b sequence yn = n and L). Study the cases L = 0 and L = + as well. Cristinel Mortici, T argovi ste

173

Pagina rezolvitorilor
CAMPULUNG MUSCEL Colegiul Nat ional ,,Dinicu Golescu. Clasa a X-a (prof. PETRIS OR Constantin). NECULA Emanuel: IX(137,139), X(136-140), XI(136-138), G(239,243), L.243. GRAJDURI (IAS I) AnaS coala Gimnazial a ,,Valea Satului. Clasa a IV-a (prof. nv.primar URMA ANDROI L acr amioara). CAS Gabriela: P(257,258,260,262,264); PETIEANU IonelaVasilica: P(257,258,260,262,264); RUSU Alexandra: P(257,258,260,262,264); STOICA Mariana-Ionela: P(257,258,260,262,264). IAS I S coala nr. 3 ,,Al. Vlahut a. Clasa a VI-a (prof. MARIN Mirela). ENEA Codrut : P(266,267), VI(158,161,163); POPOVICI Teodor-Andrei: P(266-268), VI(158, 160-163). Clasa a VII-a (prof. MARIN Mirela). CIOCOIU Alexandra: VI(158,163), VII(160-162); CONDURACHE Alexandra: VI(158,163), VII(160-162); COSTEA Ale xandru: VI(158,163), VII(160-162); MARIN Marius: VI(158,161), VII(160-162); MANZAT Rebecca: VI(158,163), VII(160-162); TROFIN Ana: VI(158,163), VII(160-162); VLAD Ioana: VI(158,163), VII(160-162). S coala Gimnazial a nr. 26 ,,G. Co sbuc. Clasa a IV-a (prof. nv.primar RACU Sebastian-Andrei: P(255,261,263,265,266); GROSU Victor-AleMaria). CIOPEICA ssandru: P(255,256,260-262,265); LUCHIAN Maria-Clara: P(255-258,260-262,265); MANOLE Alexandra-Georgiana: P(255-262,265); PASNICU Cosmin-Constantin: P(255-257,259-261,265); POPESCU Andrei-Eduard: P(255,259-261,265); RAILEANU Ana-Maria: P(255-262,265); RAILEANU R azvan-Constantin: P(255-262,265); VASILE Raluca-Andreea: P(255-257,259-262). S coala nr. 33 ,,M. Kog alniceanu. Clasa a III-a ( nv. S IRBU Lenut a). CIOCOIU Alexandru Boris: P(261-266). Liceul Economic ,,Virgil Madgearu. Clasa a IX-a (prof. OLENIUC Claudia). JOLDESCU Petronela Lavinia: VII(159,160), VIII(158,159,162); LUNGU Adelina Veronica: VII(159,160), VIII(158,159,162); PLOP Cosmin Alexandru: VII(159,160), VIII(158,159,162). Clasa a X-a (prof. OLENIUC Claudia). GEORGESCU R azvan: VIII(158,159,162), X.138, XI.136; GHERGHEL Petrut a: VIII(158,159,162), X.138, C XI.136; MARIN Ileana: VIII(158,159,162), X.138, XI.136; PLACINT A at alinaAlexandra: VIII(158,159,162), X.138, XI.136; S CRAB Bianca-Maria: VIII(158,159, 162), X.138, XI.136. Clasa a XI-a (prof. OLENIUC Claudia). S IRGHI Nicoleta: X(138,140), XI.136, XII(136,138); SOFRONEA Daniela: X(138,140), XI.136, XII(136,138). Colegiul Nat ional ,,Emil Racovit a. Clasa a V-a (prof. BUDEANU C at alin). OLENIUC Iulian: P(266,267), V.159, VI(158,160). Cristian). POPA Ioana Colegiul Nat ional Ia si. Clasa a VI-a (prof. LAZAR Maria: VI(158-164), VII(158-163). Clasa a VIII-a (prof. POPA Gabriel). DOMINTE S tefan: VII(158-164), VIII(158-164). 174

Colegiul Nat ional ,,M. Eminescu. Clasa a V-a (prof. BLENDEA Gheorghe). ALISTAR S tefan-Daniel: P(265-268), V.159; VASILIU R azvan Andrei: P(265-268), V.159; ZAMCANU Andrei-Rare s: P(265-268), V.159. Liceul Teoretic ,,Miron Costin. Clasa a IV-a. LUCHIAN Denisa Alexandra: P(260,263-268), V.161. ROS IORI (BACAU) S coala Gimnazial a nr. 1. Clasa a VIII-a (prof. CICEU Nela). ONOFREI Adina: VI(158,163), VII(158-163), VIII(159,161,162). TARGOVIS TE S coala ,,Vasile C arlova. Clasa I ( nv. ION Daniela). BACIU Maria Briana: P(255-257,259,260). TRUS ES TI (BOTOS ANI) Liceul Teoretic ,,Demostene Botez. Clasa a XII-a (prof. CULIDIUC C at alin). Ovidiu: X.138, XI.136, XII(136-139), G.240, L.240. PAUC A T IGANAS I (IAS I) S coala Gimnazial a cu clasele I-VIII ,,M. Kog alniceanu. Clasa a II-a (prof. nv. primar VOICU Ana). GANEANU Andreea: P(255-258,260); PIU Eduard-Narcis: P(255-258,260); PIU Iuliana: P(255-258,260); RUSU Alexandru-Gabriel: P(255-258, 260); SANDU Ruben: P(255-258,260). Clasa a III-a ( nv. SAMSON Daniel-Mihai). A DUCA Daria-C at alina-Stela: P(263-267). Clasa a IV-a (prof. nv.primar BADIT Aurica). DUCA Anamaria: P(261,263-267); DUCA Ema-S tefania: P(261,263-267); Alexandra-Ionela: P(261, DUCA Roxana-Georgiana: P(261,263-267); GHIOANCA 263-267); SANDU Marta: P(261,263-267); S TEFANACHE Anamaria: P(261,263267); TICAN David-Petru: P(261,263-267). Clasa a V-a (prof. IACOB Aida Andreea). DUCA Adriana: P(265-267), V(159,160); DUCA Denis-Alexandru: P(265 267), V(159,160); GANEANU Maria-Teodora: P(265-267), V(159,160); ROMANIUC Georgiana: P(265-267), V(159,160). Clasa a VI-a (prof. IACOB Aida Andreea). CAZADOI Ioana Cristina: P.265, V(159,163), VI.164, VII.158.

Elevi rezolvitori premiat i


S coala nr. 3 ,,Al. Vlahut a , Ia si MARIN Marius (cl. a VII-a): 2/2012(5pb), 1/2013(5pb), 2/2013(5pb). S coala nr. 26 ,,G. Co sbuc, Ia si VASILE Raluca Andreea (cl. a IV-a): 2/2012(5pb), 1/2013(7pb), 2/2013(7pb). S coala nr. 33 ,,M. Kog alniceanu CIOCOIU Alexandru Boris (cl. a III-a): 2/2012(9pb), 1/2013(6pb), 2/2013(6pb). S coala Gimnazial a ,,Valea Satului, Grajduri (Ia si) 175

RUSU Alexandra (cl. a IV-a): 2/2012(5pb), 1/2013(5pb), 2/2013(5pb). STOICA Mariana Ionela (cl. a IV-a): 2/2012(5pb); 1/2013(5pb), 2/2013(5pb). S coala Gimnazial a cu clasele I-VIII ,,M. Kog alniceanu, T ig ana si (Ia si) Alexanda-Ionela (cl. a IV-a): 2/2012(11pb); 1/2013(7pb), 2/2013(6pb). GHIOANCA S TEFANACHE Anamaria (cl. a IV-a): 2/2012(11pb), 1/2013(7pb), 2/2013(6pb). TICAN David-Petru (cl. a IV-a): 2/2012(11pb), 1/2013(7pb), 2/2013(6pb). DUCA Anamaria (cl. a IV-a): 1/2012(6pb), 1/2013(7pb), 2/2013(6pb). Liceul Teoretic ,,Miron Costin, Ia si LUCHIAN Denisa Alexandra (cl. a IV-a): 2/2012(7pb), 1/2013(10pb), 2/2013(8pb). Liceul Economic ,,Virgil Madgearu, Ia si SIRGHI Nicoleta (cl. a XI-a): 1/2012 (5pb), 2/2012(5pb), 2/2013(5pb). SOFRONEA Daniela (cl. a XI-a): 1/2012 (5pb), 2/2012(5pb), 2/2013(5pb). Colegiul Nat ional ,,Emil Racovit a, Ia si OLENIUC Iulian (cl. a V-a): 2/2012(7pb), 1/2013(6pb), 2/2013(5pb).

IMPORTANT
In scopul unei leg aturi rapide cu redact ia revistei, pot fi utilizate urm atoarele adrese e-mail: t birsan@yahoo.com si profgpopa@yahoo.co.uk . Pe aceast a cale colaboratorii pot purta cu redact ia un dialog privitor la materialele trimise acesteia, procurarea numerelor revistei etc. Suger am colaboratorilor care trimit probleme originale pentru publicare s a le numeroteze si s a- si ret in a o copie xerox a lor pentru a putea purta cu u surint a o discut ie prin e-mail asupra accept arii/neaccept arii acestora de c atre redact ia revistei. La problemele de tip L se primesc solut ii de la orice iubitor de matematici elementare (indiferent de preocupare profesional a sau v arst a ). Fiecare dintre solut iile acestor probleme - ce sunt publicate n revist a dup a jum atate de an - va fi urmat a de numele tuturor celor care au rezolvat-o. Adres am cu insistent a rug amintea ca materialele trimise revistei s a nu fie (s a nu fi fost) trimise si altor publicat ii. Rug am ca materialele tehnoredactate s a fie trimise pe adresa redact iei A nsot ite de sierele lor (de preferint a n L TEX). Pentru a facilita comunicarea redact iei cu colaboratorii ei, autorii materialelor sunt rugat i s a indice adresa e-mail.

176

Revista semestrial RECREAII MATEMATICE este editat de ASOCIAIA RECREAII MATEMATICE. Apare la datele de 1 martie i 1 septembrie i se adreseaz elevilor, profesorilor, studenilor i tuturor celor pasionai de matematica elementar. n atenia tuturor colaboratorilor Materialele trimise redaciei spre publicare (note i articole, chestiuni de metodic, probleme propuse etc.) trebuie prezentate ngrijit, clar i concis; ele trebuie s prezinte interes pentru un cerc ct mai larg de cititori. Se recomand ca textele s nu depeasc patru pagini. Evident, ele trebuie s fie originale i s nu fi aprut sau s fi fost trimise spre publicare altor reviste. Rugm ca materialele tehnoredactate s fie nsoite de fiierele lor. Problemele destinate rubricilor: Probleme propuse i Probleme pentru pregtirea concursurilor vor fi redactate pe foi separate cu enun i demonstraie/rezolvare (cte una pe fiecare foaie) i vor fi nsoite de numele autorului, coala i localitatea unde lucreaz/nva. Redacia va decide asupra oportunitii publicrii materialelor primite. n atenia elevilor Numele elevilor ce vor trimite redaciei soluii corecte la problemele din rubricile de Probleme propuse i Probleme pentru pregatirea concursurilor vor fi menionate n Pagina rezolvitorilor. Elevii menionai de trei ori vor primi o diplom i un premiu n cri. Elevii rezolvitori vor ine seama de regulile: 1. Pot trimite soluii la minimum cinci probleme propuse n numrul prezent i cel anterior al revistei (pe o foaie va fi redactat o singur problem). 2. Elevii din clasele VI-XII au dreptul s trimit soluii la problemele propuse pentru clasa lor, pentru orice clas mai mare, din dou clase mai mici i imediat anterioare. Cei din clasa a V-a pot trimite soluii la problemele propuse pentru clasele a IV-a, a V-a i orice clas mai mare, iar elevii claselor I-IV pot trimite soluii la problemele propuse pentru oricare din clasele primare i orice clas mai mare. Orice elev poate trimite soluii la problemele de concurs (tip G i L). 3. Vor fi menionate urmtoarele date personale: numele i prenumele, clasa, coala i localitatea, precum i numele profesorului cu care nva. 4. Plicul cu probleme rezolvate se va trimite prin pot (sau va fi adus direct) la adresa Redaciei: Prof. dr. Temistocle Brsan Str. Aurora, nr. 3, sc. D, ap. 6, 700 474, Iai Jud. IAI E-mail: t_birsan@yahoo.com

CUPRINS
Profesorul CONSTANTIN CORDUNEANU la aniversarea a 85 de ani (T. Brsan) ............. 89

ARTICOLE I NOTE
M. TRNUCEANU Grupuri finite cu proprietatea (P) .......................................................... 92 I. PTRACU Cercurile mixtliniare adjuncte nscrise asociate unui triunghi .......................... 96 D.M. BTINEU-GIURGIU, N. STANCIU Cteva aplicaii ale inegalitii Ionescu- Weitzenbck ..................... 100 M. BENCZE Cteva generalizri i rafinri ale inegalitii x2 +y2 +z2 xy+yz+zx ................. 103

NOTA ELEVULUI
A.G. BROJBEANU Caracterizarea unor proprieti de perpendicularitate n care sunt implicate punctele O, I, H, G, O9 ................... 106

CORESPONDEN E
A. REISNER La cissode, podaire de la parabole ................................. 110 F. SMARANDACHE Another proof of the Ptracus theorem .. 114

CHESTIUNI METODICE
M. MONEA, S. MONEA Comentarii pe marginea unor probleme .......................................... 116 D. POPESCU Asupra unei clase de ecuaii/ inecuaii ............................................................... 119 F. STNESCU Rezolvarea unor ecuaii i inecuaii integrale .................................................. 122

CUM CONCEPEM ... CUM REZOLV M


M. TETIVA Merge i aa! ....................................................................................................... 126

COLI I DASCLI
A. NEICU Colegiul Naional tefan cel Mare Hrlu ........................................................ 129

CONCURSURI I EXAMENE
Concursul de matematic "Florica T. Cmpan", ed. a XIII-a, 2013 ..................................... 132

PROBLEME I SOLU II
Soluiile problemelor propuse n nr. 1/2013 ............................................................................. 136 Soluiile problemelor pentru pregtirea concursurilor propuse n nr. 1/2013 ........................... 151 Probleme propuse ......................................................................................................................... 163 Probleme pentru pregtirea concursurilor ................................................................................... 169 Training problems for mathematical contests ............................................................................ 171 Pagina rezolvitorilor .................................................................................................................... 174 Elevi rezolvitori premiai ............................................................................................................ 175
ISSN 1582 1765 8 lei

S-ar putea să vă placă și